Sie sind auf Seite 1von 77

COLEO ELEMENTOS

DA MATEMTICA

VOLUME 1

Marcelo Rufino de Oliveira


Com formao pelo Instituto Tecnolgico de Aeronutica (ITA)
Coordenador das Turmas Militares do Colgio Ideal
Professor de Matemtica das Turmas Militares do Colgio Ideal
Coordenador Regional da Olimpada Brasileira de Matemtica

Mrcio Rodrigo da Rocha Pinheiro


Com formao pela Universidade Federal do Par (UFPa)
Professor de Matemtica das Turmas Militares do Colgio Ideal

COLEO ELEMENTOS
DA MATEMTICA
Marcelo Rufino de Oliveira
Mrcio Rodrigo da Rocha Pinheiro

CONJUNTOS
FUNES
ARITMTICA

3 edio (2010)

Copyright 2009 by marcelo rufino de oliveira

Todos os direitos desta edio esto reservados


Marcelo Rufino de Oliveira
Belm Par Brasil
E-mail: marcelorufino@hotmail.com

Ilustrao da Capa
Maximiliano / Zeef
Modificaes em 2010
Annysteyne M. Chaves
LOUDES PACHECO
Ficha Catalogrfica
Editora VestSeller
Impresso

F48c.........Oliveira, Marcelo Rufino de


Coleo elementos da matemtica, 1 : conjuntos, funes, aritmtica /
Marcelo Rufino de Oliveira, Mrcio Rodrigues da Rocha Pinheiro. 3 ed.
Fortaleza Editora VestSeller - 2010.
p. 309
ISBN: 978-85-89171-22-9
1. Matemtica (Ensino Mdio) 2. Matemtica (Ensino Mdio) conjuntos.
3. Matemtica (Ensino Mdio) funes 3 Matemtica (Ensino Mdio) - aritmtica I
- Pinheiro, Mrcio Rodrigues da Rocha. II. Ttulo. III: Ttulo: Conjuntos. IV. Ttulo:
Funes.V. Ttulo: Aritmtica..
CDD: 510.7

APRESENTAO 3 EDIO
Este o primeiro volume da Coleo Elementos da Matemtica, programada para
apresentar toda a matemtica elementar em seis volumes:
Volume 1 Conjuntos, Funes, Exponencial, Logaritmo e Aritmtica
Autor: Marcelo Rufino de Oliveira e Mrcio Rodrigo da Rocha Pinheiro
Volume 2 Geometria Plana
Autores: Marcelo Rufino de Oliveira e Mrcio Rodrigo da Rocha Pinheiro
Volume 3 Seqncias, Combinatria, Probabilidade, e Matrizes
Autor: Marcelo Rufino de Oliveira, Manoel Leite Carneiro e Jefferson Frana
Volume 4 Nmeros Complexos, Polinmios e Geometria Analtica
Autores: Marcelo Rufino de Oliveira e Jefferson Frana
Volume 5 Geometria Espacial
Autor: Antonio Eurico da Silva Dias
Volume 6 Clculo
Autor: Mrcio Rodrigo da Rocha Pinheiro
A linguagem de conjuntos essencialmente a base de toda a linguagem matemtica.
Algo similar s regras gramaticais, aprendidas em vrios nveis de profundidade, durante toda
a vida escolar. No possvel expressar-se corretamente em certa linguagem sem antes
conhecer o bsico da estrutura lingstica. Analogamente, no conveniente buscar um
conhecimento mais detalhado de matemtica elementar abrindo mo das noes
fundamentais de conjuntos.
Como uma verdadeira alfabetizao matemtica, a linguagem dos conjuntos e das
funes deve tornar-se muito clara ao aluno, ainda que no haja total dela domnio, para que
a compreenso dos fatos cotidianos torne-se mais eficientes. As idias de agrupamento de
entes que tm em comum alguma propriedade importante, dos tipos e importncia dos
nmeros, de correspondncia entre elementos de conjuntos, distintos ou no, so
fundamentais para a formao do cidado consciente e analista do mundo que o rodeia. A
associao entre a linguagem gramatical e a linguagem matemtica d-se no mbito da
linguagem dos conjuntos e das funes. Praticamente toda a Matemtica pode ser (e de fato
) desenvolvida a partir destes conceitos.
A aritmtica trata do estudo particular de um conjunto numrico importante: o dos
nmeros inteiros. Sua importncia reside em vrios aspectos, tais quais estruturar os sistemas
de bases numricas (alicerces de todas as reas da matemtica), servir de base para a
informtica, aplicaes na cinemtica, quantizao da carga eltrica em condutores, suporte
na formulao de calendrios (ciclo anual de 365 dias, ciclo lunar de 28 dias, etc), alm de
aplicao direta em outras reas da matemtica: nmeros complexos (1 e 2 Leis de Moivre),
diviso de polinmios, geometria (polgonos estrelados), etc.
Finalmente, deseja-se deixar claro o carter experimental desta obra. Por meio dela,
busca-se reunir o melhor que h no que diz respeito preparao de alunos aos concursos
vestibulares mais difceis do Brasil. No entanto, embora se busque a perfeio, natural que
haja erros ou imprecises. Deseja-se manter um relacionamento atencioso com alunos e
outros professores, a fim de aprimorar este trabalho.

Os autores

ndice
Captulo 1. Conjuntos
1. Introduo Noes Primitivas . . . . . . . . . . . . . . . . . . . . . . . . . . . . . . . . . . . . . . . .
2. Representaes de Conjuntos . . . . . . . . . . . . . . . . . . . . . . . . . . . . . . . . . . . . . . . . .
3. Conjuntos Notveis . . . . . . . . . . . . . . . . . . . . . . . . . . . . . . . . . . . . . . . . . . . . . . . . .
4. Relao de Incluso Subconjuntos . . . . . . . . . . . . . . . . . . . . . . . . . . . . . . . . . . . .
5. Igualdade entre Conjuntos . . . . . . . . . . . . . . . . . . . . . . . . . . . . . . . . . . . . . . . . . . . .
6. Operaes entre Conjuntos . . . . . . . . . . . . . . . . . . . . . . . . . . . . . . . . . . . . . . . . . . .
7. Cardinalidade da Unio de Conjuntos Princpio da Incluso-Excluso . . . . . . . . .
8. Par Ordenado . . . . . . . . . . . . . . . . . . . . . . . . . . . . . . . . . . . . . . . . . . . . . . . . . . . . . .
9. Produto Cartesiano . . . . . . . . . . . . . . . . . . . . . . . . . . . . . . . . . . . . . . . . . . . . . . . . . .
10. Principais Conjuntos Numricos . . . . . . . . . . . . . . . . . . . . . . . . . . . . . . . . . . . . . . . .
Exerccios . . . . . . . . . . . . . . . . . . . . . . . . . . . . . . . . . . . . . . . . . . . . . . . . . . . . . . . . .

1
2
2
3
5
6
13
15
15
17
36

Captulo 2. Funes
1. Definies Iniciais . . . . . . . . . . . . . . . . . . . . . . . . . . . . . . . . . . . . . . . . . . . . . . . . . . . 53
2. Igualdade de Funes . . . . . . . . . . . . . . . . . . . . . . . . . . . . . . . . . . . . . . . . . . . . . . . 57
3. Funes Implcitas . . . . . . . . . . . . . . . . . . . . . . . . . . . . . . . . . . . . . . . . . . . . . . . . . . 59
4. Grfico de uma Funo . . . . . . . . . . . . . . . . . . . . . . . . . . . . . . . . . . . . . . . . . . . . . . 60
5. Imagem Direta e Imagem Inversa . . . . . . . . . . . . . . . . . . . . . . . . . . . . . . . . . . . . . . 64
6. Tipologia . . . . . . . . . . . . . . . . . . . . . . . . . . . . . . . . . . . . . . . . . . . . . . . . . . . . . . . . . . 67
7. Composio de Funes . . . . . . . . . . . . . . . . . . . . . . . . . . . . . . . . . . . . . . . . . . . . . 75
8. Inverso de Funes . . . . . . . . . . . . . . . . . . . . . . . . . . . . . . . . . . . . . . . . . . . . . . . . 80
9. lgebra de Funes . . . . . . . . . . . . . . . . . . . . . . . . . . . . . . . . . . . . . . . . . . . . . . . . . 84
85
10. Monotonicidade . . . . . . . . . . . . . . . . . . . . . . . . . . . . . . . . . . . . . . . . . . . . . . . . . . . .
91
11. Paridade . . . . . . . . . . . . . . . . . . . . . . . . . . . . . . . . . . . . . . . . . . . . . . . . . . . . . . . . .
94
12. Funes Peridicas . . . . . . . . . . . . . . . . . . . . . . . . . . . . . . . . . . . . . . . . . . . . . . . . .
13. Algumas Transformaes Geomtricas Bsicas . . . . . . . . . . . . . . . . . . . . . . . . . . . 98
14. Funo Afim . . . . . . . . . . . . . . . . . . . . . . . . . . . . . . . . . . . . . . . . . . . . . . . . . . . . . . . 107
15. Funes Quadrticas . . . . . . . . . . . . . . . . . . . . . . . . . . . . . . . . . . . . . . . . . . . . . . . . 116
Exerccios . . . . . . . . . . . . . . . . . . . . . . . . . . . . . . . . . . . . . . . . . . . . . . . . . . . . . . . . . 127
Captulo 3. Representao Decimal
1. Introduo . . . . . . . . . . . . . . . . . . . . . . . . . . . . . . . . . . . . . . . . . . . . . . . . . . . . . . . . . 167
Exerccios . . . . . . . . . . . . . . . . . . . . . . . . . . . . . . . . . . . . . . . . . . . . . . . . . . . . . . . . . 171
Captulo 4. Critrios de Divisibilidade
1. Introduo . . . . . . . . . . . . . . . . . . . . . . . . . . . . . . . . . . . . . . . . . . . . . . . . . . . . . . . . . 177
Exerccios . . . . . . . . . . . . . . . . . . . . . . . . . . . . . . . . . . . . . . . . . . . . . . . . . . . . . . . . . 181
Captulo 5. Propriedades da Divisibilidade
1. Propriedades . . . . . . . . . . . . . . . . . . . . . . . . . . . . . . . . . . . . . . . . . . . . . . . . . . . . . . 184
2. Algoritmo da Diviso Euclidiana . . . . . . . . . . . . . . . . . . . . . . . . . . . . . . . . . . . . . . . . 188
Exerccios . . . . . . . . . . . . . . . . . . . . . . . . . . . . . . . . . . . . . . . . . . . . . . . . . . . . . . . . . 191
Captulo 6. Nmeros Primos
1. Definio . . . . . . . . . . . . . . . . . . . . . . . . . . . . . . . . . . . . . . . . . . . . . . . . . . . . . . . . . .
2. Propriedades . . . . . . . . . . . . . . . . . . . . . . . . . . . . . . . . . . . . . . . . . . . . . . . . . . . . . .
3. Teorema Fundamental da Aritmtica . . . . . . . . . . . . . . . . . . . . . . . . . . . . . . . . . . . .
4. Dois Teoremas Clssicos Sobre Nmeros Primos . . . . . . . . . . . . . . . . . . . . . . . . .
5. A Infinitude dos Nmeros Primos . . . . . . . . . . . . . . . . . . . . . . . . . . . . . . . . . . . . . . .
6. Divisores Primos de um Inteiro Composto . . . . . . . . . . . . . . . . . . . . . . . . . . . . . . . .
7. Crivo de Eratstenes . . . . . . . . . . . . . . . . . . . . . . . . . . . . . . . . . . . . . . . . . . . . . . . .
8. Primos Gmeos . . . . . . . . . . . . . . . . . . . . . . . . . . . . . . . . . . . . . . . . . . . . . . . . . . . .

199
199
202
204
205
205
206
206

9. Seqncias de Inteiros Consecutivos Compostos . . . . . . . . . . . . . . . . . . . . . . . . . . 207


10. Postulado de Bertrand . . . . . . . . . . . . . . . . . . . . . . . . . . . . . . . . . . . . . . . . . . . . . . . 207
Exerccios . . . . . . . . . . . . . . . . . . . . . . . . . . . . . . . . . . . . . . . . . . . . . . . . . . . . . . . . 208
Captulo 7. MDC e MMC
MDC
1. Definio . . . . . . . . . . . . . . . . . . . . . . . . . . . . . . . . . . . . . . . . . . . . . . . . . . . . . . . . . .
2. Existncia e Unicidade do MDC . . . . . . . . . . . . . . . . . . . . . . . . . . . . . . . . . . . . . . . .
3. Inteiros Primos Entre Si . . . . . . . . . . . . . . . . . . . . . . . . . . . . . . . . . . . . . . . . . . . . . .
4. Propriedades . . . . . . . . . . . . . . . . . . . . . . . . . . . . . . . . . . . . . . . . . . . . . . . . . . . . . .
5. Clculo do MDC a partir das fatoraes cannicas . . . . . . . . . . . . . . . . . . . . . . . . .
6. Algoritmo de Euclides . . . . . . . . . . . . . . . . . . . . . . . . . . . . . . . . . . . . . . . . . . . . . . . .

212
212
212
212
213
216

MMC
7. Definio . . . . . . . . . . . . . . . . . . . . . . . . . . . . . . . . . . . . . . . . . . . . . . . . . . . . . . . . . .
8. Clculo do MMC a partir das fatoraes cannicas . . . . . . . . . . . . . . . . . . . . . . . . .
9. Relao Entre MDC e MMC . . . . . . . . . . . . . . . . . . . . . . . . . . . . . . . . . . . . . . . . . . .
Exerccios . . . . . . . . . . . . . . . . . . . . . . . . . . . . . . . . . . . . . . . . . . . . . . . . . . . . . . . . .

217
217
219
220

Captulo 8. Divisores
1. Definio . . . . . . . . . . . . . . . . . . . . . . . . . . . . . . . . . . . . . . . . . . . . . . . . . . . . . . . . . .
2. Nmero de Divisores Positivos . . . . . . . . . . . . . . . . . . . . . . . . . . . . . . . . . . . . . . . . .
3. Soma dos Divisores Positivos . . . . . . . . . . . . . . . . . . . . . . . . . . . . . . . . . . . . . . . . .
4. Produto dos Divisores . . . . . . . . . . . . . . . . . . . . . . . . . . . . . . . . . . . . . . . . . . . . . . .
5. Nmeros Perfeitos . . . . . . . . . . . . . . . . . . . . . . . . . . . . . . . . . . . . . . . . . . . . . . . . . .
6. Nmeros Amigos . . . . . . . . . . . . . . . . . . . . . . . . . . . . . . . . . . . . . . . . . . . . . . . . . . .
7. Nmeros Deficientes e Abundantes . . . . . . . . . . . . . . . . . . . . . . . . . . . . . . . . . . . . .
Exerccios . . . . . . . . . . . . . . . . . . . . . . . . . . . . . . . . . . . . . . . . . . . . . . . . . . . . . . . . .

228
228
231
233
234
236
236
237

Captulo 9. Congruncias
1. Propriedades . . . . . . . . . . . . . . . . . . . . . . . . . . . . . . . . . . . . . . . . . . . . . . . . . . . . . . 241
2. Sistemas Completos de Restos . . . . . . . . . . . . . . . . . . . . . . . . . . . . . . . . . . . . . . . . 246
Exerccios . . . . . . . . . . . . . . . . . . . . . . . . . . . . . . . . . . . . . . . . . . . . . . . . . . . . . . . . . 247
Captulo 10. Funo Mximo Inteiro
1. Propriedades . . . . . . . . . . . . . . . . . . . . . . . . . . . . . . . . . . . . . . . . . . . . . . . . . . . . . .
2. Expoente de um Primo na Fatorao de n! . . . . . . . . . . . . . . . . . . . . . . . . . . . . . . .
3. Nmero de Dgitos de um Inteiro Positivo . . . . . . . . . . . . . . . . . . . . . . . . . . . . . . . .
Exerccios . . . . . . . . . . . . . . . . . . . . . . . . . . . . . . . . . . . . . . . . . . . . . . . . . . . . . . . . .

250
253
255
256

Captulo 11. Equaes Diofantinas Lineares


1. Definio . . . . . . . . . . . . . . . . . . . . . . . . . . . . . . . . . . . . . . . . . . . . . . . . . . . . . . . . . .
2. Condio de Existncia de Soluo . . . . . . . . . . . . . . . . . . . . . . . . . . . . . . . . . . . . .
3. Solues da Equao ax + by = c . . . . . . . . . . . . . . . . . . . . . . . . . . . . . . . . . . . . . .
Exerccios . . . . . . . . . . . . . . . . . . . . . . . . . . . . . . . . . . . . . . . . . . . . . . . . . . . . . . . . .

258
258
258
260

Apndices
1. Bases de Numerao
2. Tringulos Pitagricos . . . . . . . . . . . . . . . . . . . . . . . . . . . . . . . . . . . . . . . . . . . . . . .
3. Teoremas de Euler e Fermat . . . . . . . . . . . . . . . . . . . . . . . . . . . . . . . . . . . . . . . . . .
4. Teorema de Wilson
5. Equaes Diofantinas no lineares
6. Representaes de nmeros naturais como somo de potncias inteiras

262
265
269
275
278
287

Gabaritos . . . . . . . . . . . . . . . . . . . . . . . . . . . . . . . . . . . . . . . . . . . . . . . . . . . . . . . . . . . . 290

Captulo 1. Conjuntos
1. CONJUNTOS
1.1. INTRODUO NOES PRIMITIVAS
So aquelas aceitas sem definio matemtica formal, de modo que a experincia cotidiana e
exemplos ilustrativos sejam suficientes para repassar suas principais caractersticas. A rigor, utilizam-se
tambm espcies de barreiras ou limitaes lgicas das propriedades relativas a tais noes, papel
realizado pelos postulados ou axiomas, o que, no entanto, no cabe num curso deste nvel.
Neste estudo da linguagem de conjuntos, aceitar-se-o trs noes primitivas:
a) A prpria idia de CONJUNTO. Intuitivamente, consiste nas idias usuais de coleo ou
agrupamento de objetos quaisquer bem definidos, que, entretanto, no convm como definies, uma
vez que fogem ao senso comum as noes de coleo de apenas uma coisa (conjunto unitrio) ou, mais
acentuadamente, agrupamento de nada (conjunto vazio). Qual seria, por exemplo, a coleo de Ferraris
do professor Mrcio (formada, pelo menos por enquanto, por zero elementos)?
Usualmente, representa-se um conjunto por uma letra maiscula do nosso alfabeto. Assim, falase usualmente nos conjuntos A, B, M, X, Y1, Y2, etc.
b) ELEMENTO. Quando algum deseja iniciar uma coleo, de um modo geral no lhe
limitado o gnero (tipo) de coisas que pode colecionar. Pode-se, teoricamente, colecionar qualquer
coisa. Analogamente, a natureza dos membros (entes) formadores de um conjunto totalmente
arbitrria. Tais coisas (objetos) que constituem um conjunto (no vazio) que so denominadas
elementos do conjunto.
imprescindvel notar que at mesmo um conjunto pode ser (funcionar como) membro de outro
conjunto. Assim, por exemplo, o conjunto das selees de uma copa do mundo de futebol formado por
vrias equipes, as quais, por sua vez, podem ser consideradas como conjuntos de vrios jogadores, que
podem ser encarados como conjuntos de clulas, e assim por diante. Pode-se pensar no conjunto O dos
rgos de um determinado ser humano. O corao pertence a O. Por sua vez, o corao pode ser visto
tambm como um conjunto, C, formado por clulas especficas. Da, tem-se C funcionando como
elemento de O. Com igual propriedade, note-se que cada um dos alunos do 3 Militar pode ser encarado
como elemento do conjunto M, que representa tal turma, a qual, por sua vez, pode tambm ser vista
como elemento do conjunto I das turmas do Ideal Militar, o qual, a seu tempo, tambm pode ser visto
como elemento do conjunto E das escolas de Belm, e assim por diante.
Comumente, representa-se um membro genrico de um conjunto por uma letra minscula.
c) A noo de PERTINNCIA DE UM ELEMENTO A UM CONJUNTO. Corresponde
resposta a perguntas do tipo: tal ente (coisa) ou no elemento daquele conjunto? Admitir-se-
intrnseca a capacidade de responder, de forma nica, positiva ou negativamente a esta questo.
Igualmente, todo conjunto deve possuir tacitamente a capacidade de ter seus elementos bem
determinados (caracterizados). Assim, a gula, por exemplo, pertence ao conjunto dos pecados capitais,
ao passo que o professor Mrcio no pertence nem ao conjunto dos alunos do Ideal Militar, nem ao dos
homens que j pisaram em Marte (pelo menos de acordo como o que se sabe, atualmente).
A pertinncia ou no de um elemento a um determinado conjunto indicada pelos smbolos
(pertence) ou (no pertence), respectivamente.
Obs.: Como regra fundamental, a ser aceita sem demonstrao (axioma) tem-se que um conjunto
(ou qualquer objeto, de um modo mais geral) no pode ser elemento dele mesmo, ou seja, qualquer que
seja o conjunto A, impe-se A A. Entretanto, h outros axiomas (que no sero trabalhados aqui), cujo
principal objetivo consiste em evitar os denominados paradoxos: idias aparentemente perfeitas, mas
que levam a concluses contraditrias. Por exemplo, o mais famoso deles brinca com o axioma acima:
o paradoxo de Russel, que considera o conjunto X formado pelos conjuntos que no so elementos deles
mesmos (X = {AA A}, conforme notao a ser vista em seguida). A pergunta : X X? Em caso
afirmativo, X, por pertencer a X, no pode pertencer a X, o que uma contradio. Ento X no pode
pertencer a X. Mas, por definio X pertenceria a X (!?). Uma forma bem popular de apresentar este
paradoxo a seguinte: numa certa cidadezinha, existe um barbeiro que s faz a barba dos homens que
no barbeiam a si prprios (e de todos eles). Tente responder pergunta: quem faz a barba do barbeiro?
Tais paradoxos, entretanto, bem como uma apresentao mais rigorosa da Teoria dos Conjuntos
1

Captulo 1. Conjuntos
(desenvolvida formalmente a partir do final do sculo XIX, notadamente pelo matemtico russogermnico Georg Cantor), no cabem num curso deste nvel, muito embora as noes elementares aqui
desenvolvidas serem de grande utilidade para uma linguagem matemtica mais padronizada, utilizada
em praticamente todos os ramos da Matemtica (e, conseqentemente, em muitas reas do
conhecimento).
1.2. REPRESENTACES DE CONJUNTOS
De um modo geral, representa-se um conjunto por meio de chaves ou de uma linha fechada,
qualquer um dos quais deixando os elementos do conjunto, e somente eles, em seu interior, de maneira
explcita ou no.
a) Utilizando chaves:
a.1) Forma analtica ou tabular ou por enumerao: explicita elementos do conjunto, podendo
ser todos ou alguns, nesse ltimo caso sendo possvel notar diretamente quais so os elementos
subentendidos. Exemplos:
A = {a, e, i, o, u}; B = {1, 2, 3, ...}; C = {2, 3, 5, 7}.
a.2) Forma sinttica (caracterizao por meio de propriedade): expressa uma propriedade comum
a todos os elementos do conjunto e somente a eles. Exemplos.:
A = {aa vogal}; B = {bb um nmero natural positivo}; C = {cc primo menor que 10}.
b) Utilizando diagramas:
Consiste no uso de uma linha simples e fechada qualquer (em geral, uma circunferncia)
contornando os elementos do conjunto. Comumente, os elementos so indicados por pontos do interior
da linha. Tais diagramas so freqentemente chamados de diagramas de (Euler ) Venn. Exemplos:
B
C
A
u
a
2
1
i
3
2
e
o
5
3
7
m

1.3. CONJUNTOS NOTVEIS


a) Conjunto unitrio: possui um nico elemento. Exemplos:
P = {x Zx primo e par} = {2}.
L = {xx atual presidente eleito do Brasil} = {Lula}.
S = {x Zx + 1 = 0} = { 1}.
D = {a Ra = ei + 1} = {0}.
b) Conjunto Vazio: no possui elemento algum. Como possvel? De um modo geral, o conjunto
vazio definido por meio de uma propriedade contraditria, isto , uma afirmao que sempre falsa,
no podendo ser satisfeita por objeto algum. Exemplos:
X = {x R x2 < 0} = { } (no existe nmero real cujo quadrado seja negativo).
Y = {yy y} = (tudo igual a si prprio).

Captulo 1. Conjuntos
S = {x R x + x + 1 = 0} = .
O interesse em adotar esses dois casos especiais de conjuntos (vazio e unitrio) a
generalizao. Assim, por exemplo, quando se fala no conjunto das razes de uma equao polinomial
do 2 grau, no necessrio afirmar que equaes como x2 + x + 1 = 0 no possui tal conjunto em R.
Diz-se, simplesmente, que tal conjunto existe e vazio.
Obs.: o conjunto {} unitrio.
2

c) Conjunto Soluo (S): tambm denominado conjunto verdade (de uma sentena aberta), o
conjunto das respostas a uma pergunta. Mais rigorosamente (como ser visto em Lgica Matemtica),
o conjunto dos valores que podem ser atribudos a variveis, de modo a transformar uma sentena aberta
em uma proposio verdadeira. Tambm chamado conjunto verdade. Exemplos:
O conjunto soluo da equao x2 5x + 6 = 0 S = {2,3}, uma vez que os elementos desse conjunto (e
somente eles) tornam a igualdade verdadeira.
O conjunto verdade da inequao x2 < x V = {x Rx2 < x} = {x R 0 < x < 1}.
d) Conjunto Universo (U): o maior conjunto do qual podem ser retiradas as respostas a um
certo problema. Noutras palavras, um conjunto fundamental a partir do qual saem as solues de uma
certa classe de questionamentos (mais precisamente, o conjunto do qual todos os conjuntos em estudo
so subconjuntos). Note-se que no permitido aqui que U U, ou seja, no existe um conjunto
formado por todos os conjuntos. O que o universo faz , apenas, limitar uma discusso. Exemplo:
Suponha-se que dado um certo ponto O. Qual a resposta pergunta: quais so os pontos que esto a
1cm de O? A resposta : depende! Se os pontos tiverem que estar sobre uma determinada reta que passe
por O, ento h exatamente dois pontos convenientes, que deixam O como ponto mdio do segmento de
extremidade neles. Se os pontos puderem situar-se sobre um plano que contm O, ento a soluo a
circunferncia de centro em O e raio 1cm. Se o ponto puder estar em qualquer lugar (espao), ento a
resposta a superfcie de uma esfera de centro em O e raio 1cm.

B
O

OBSERVAO IMPORTANTE: Quando no se explicita o conjunto universo de uma questo, deve-se


considerar o maior possvel. Assim, a resposta pergunta acima deveria ser, como de praxe, a
superfcie esfrica descrita.
e) Conjuntos Finitos e Infinitos: Intuitivamente, um conjunto finito quando o processo de
contagem (enumerao) dos seus elementos chega a um fim, e infinito em caso contrrio. Apesar de ser
suficiente para certas situaes, tal concepo no pode ser tomada como uma definio matemtica,
uma vez que se utilizou a idia de finitude para definir fim (trechos em itlico), ou seja, em verdade no
se definiu nada. Foi exatamente esse tipo de impasse que fez com que o Cantor embasasse um estudo
revolucionrio de alto nvel, a Teoria dos Conjuntos, que elucidou dvidas clssicas pr e ps-crists,
como os famosos paradoxos de Zeno e outras brincadeiras no antes sombrio reino do infinito.
1.4. RELAO DE INCLUSO SUBCONJUNTOS
Diz-se que um conjunto A est contido em um conjunto B quando todo elemento de A for,
tambm, elemento de B. Com o mesmo significado, diz-se ainda que A um subconjunto (ou uma
parte) de B ou, ainda, que B contm A. Simbolicamente:
A B (ou B A) (
a A a B)
Exemplos:
3

Captulo 1. Conjuntos
a) Sejam U = {u Zu divisor de 120} e V = {v Zv divisor de 24}. Percebe-se que V
U, uma vez que todo nmero que divide 24 tambm capaz de dividir 120.
b) Sejam P = {2, 3, 5, 7} e Q = {q Z1 < q < 9 e q mpar}. Note-se que, apesar de que 3 P
e 3 Q, 5 P e 5 Q, bem como 7 P e 7 Q, tem-se 2 P, mas 2 Q. Dessa forma, P no pode
ser subconjunto de Q, por no cumprir a definio para tal. Escreve-se, ento, que P Q.
c) Quando se diz que todo paraense brasileiro, deve-se entender que um fato (ser paraense)
implica, acarreta, ocasiona outro (ser brasileiro). Diz-se, ainda, que ser paraense condio suficiente
para ser brasileiro, ou que ser brasileiro condio necessria para ser paraense. Tudo isso se
generaliza para qualquer sentena do tipo HIPTESE TESE. Simplesmente, o que se faz na
linguagem de conjuntos ler a proposio inicial como o conjunto dos paraenses est contido no
conjunto dos brasileiros, isto , qualquer que seja o paraense, ele tambm brasileiro.
de fundamental importncia observar no penltimo exemplo que no importa quantos
elementos de um conjunto X pertenam a Y: se pelo menos um elemento do primeiro no estiver no
segundo, ocorre X Y. Noutros termos, para verificar que um conjunto X no um subconjunto de Y,
deve-se (e basta) exibir (pelo menos) um elemento de X que no seja elemento de Y.
Alm disso, tem-se que ou X Y ou X Y, no havendo uma terceira opo nem podendo
ocorrer as duas situaes simultaneamente (isto , os dois casos excluem-se mutuamente).
Estas duas ltimas observaes servem para demonstrar um fato considerado esquisito para
muitos alunos, que muitas vezes o aceitam sem saber o porqu: o de que o vazio subconjunto de
qualquer conjunto. Com efeito, se existisse algum conjunto A do qual o conjunto vazio no fosse
subconjunto, ento dever-se-ia ser capaz de exibir pelo menos um elemento do vazio que no
pertencesse a A. Mas isso impossvel, e esta contradio nasce do fato de supor que A. Portanto, a
nica possibilidade A, qualquer que seja o conjunto A. Alm desta, h outras propriedades
importantes da relao de incluso, a saber:
I. Para qualquer conjunto A, A A (REFLEXIVIDADE).
DEMONSTRAO:
De fato, a implicao x A x A trivialmente verdadeira ( bvio que todo elemento de A
est em A).
II. Se A B e B C, ento A C (TRANSITIVIDADE).
DEMONSTRAO:
Sabe-se que todo elemento de A est em B e que todo elemento de B est em C. Logo, qualquer
elemento de A, por ser tambm elemento de C, deve estar em C.
III. Se A um conjunto finito com n elementos, ento A tem, exatamente, 2n subconjuntos.
Exemplo:
Suponha-se que X = {a, b, c}. Formando os subconjuntos de X, tem-se que:
Subconjuntos com exatamente 0 elemento:
Subconjuntos com exatamente 1 elemento: {a}, {b} e {c}
Subconjuntos com exatamente 2 elementos: {a, b}, {a, c} e {b, c}
Subconjuntos com exatamente 3 elementos: {a, b, c} = X.
Como possvel perceber, h 8 = 23 subconjuntos de A.
Por que no foram contados conjuntos como {b, a} e {c, a, b}? Porque, como ser visto no
prximo item, tais conjuntos so respectivamente iguais a {a, b} e a {a, b, c} e, assim, j foram
computados.

Captulo 1. Conjuntos
Essa ltima propriedade admite muitas demonstraes interessantes e distintas, utilizando alguns
argumentos combinatrios. Duas das mais simples so as seguintes:
1 DEMONSTRAO de III: Se A vazio, isto , se n = 0, o nico subconjunto possvel o
prprio vazio. A frmula vlida uma vez que 20 = 1. Se n > 0, formar um subconjunto X qualquer de A
consiste em tomar n decises consecutivamente:
Deciso 1: pr ou no o primeiro elemento de A no subconjunto X.
Deciso 2: pr ou no o segundo elemento de A no subconjunto X.
...
Deciso n: pr ou no o ensimo (e ltimo) elemento de A no subconjunto X.
Portanto, de acordo com o teorema fundamental da contagem, j que cada deciso pode ser feita
de 2 modos (pr ou no pr), h um total de 2.2...2 = 2n subconjuntos distintos possveis de A.
2 DEMONSTRAO de III: Pode-se proceder como no exemplo acima, s que modo mais
geral. Dado o conjunto A = {a1, a2, ..., an}, formam-se
Subconjuntos com exatamente 0 elemento, em nmero de Cn,0: .
Subconjuntos com exatamente 1 elemento, em nmero de Cn,1: {a1}, {a2}, ..., {an}.
Subconjuntos com exatamente 2 elementos, em nmero de Cn,2: {a1, a2}, ..., {a1, an}, ..., {an 1,
an}.
...
Subconjuntos com exatamente n elementos, em nmero de Cn,n: o prprio A.
Portanto, h um total de Cn,0 + Cn,1 + Cn,2 + ... + Cn,n subconjuntos de A. Finalmente, pelo
teorema do desenvolvimento binomial:
(1 + 1)n = Cn,0.1n.10+ Cn,1.1n 1 .11 + Cn,2.1n 2 .12 + ... + Cn,n.1n n .1n. Portanto, a quantidade
exata de subconjuntos de A Cn,0 + Cn,1 + Cn,2 + ... + Cn,n = (1 + 1)n = 2n.
Denomina-se subconjunto prprio de A qualquer subconjunto X de A, tal que:
XeXA
Quando um subconjunto no prprio dito imprprio ou trivial (so os subconjuntos bvios,
de qualquer conjunto A: e A).
Obs.: Alguns autores definem subconjunto prprio de A como qualquer subconjunto de A, que
no o mesmo (ou seja, admitem a possibilidade de o conjunto vazio ser prprio). Outros insistem numa
diferenciao entre os conceitos de subconjuntos imprprios e o de subconjuntos triviais. Isto no ser
feito aqui, mas deve sempre ser deixado claro por quem o utiliza.
Denomina-se conjunto das partes de um conjunto A o conjunto de todos os subconjuntos (ou
partes) de A. representado por (A). Assim, se A = {a, b, c}, tem-se (A) = {, {a}, {b}, {c}, {a,b},
{a,c}, {b,c}, A}. Note-se que nunca (A) vazio, j que sempre possui, pelo menos, como elemento.
1.5. IGUALDADE ENTRE CONJUNTOS
Por definio, dois conjuntos so iguais quando possurem os mesmos elementos. Assim,
equivalentemente, ocorre igualdade entre dois conjuntos quando todo elemento do primeiro for tambm
elemento do segundo e, reciprocamente, qualquer elemento do segundo pertencer da mesma forma ao
primeiro.
Noutras palavras, pode-se garantir que dois conjuntos so iguais quando:
- O primeiro estiver contido no segundo (todo elemento do primeiro for tambm elemento do
segundo) e
- O segundo estiver contido no primeiro (qualquer elemento do segundo pertencer da mesma forma ao
primeiro).
Em smbolos:
5

Captulo 1. Conjuntos
A=BABeBA
(Propriedade Anti-Simtrica da Incluso de Conjuntos)
exatamente devido a essa definio que nem a ordem e nem a repetio dos elementos
diferencia conjuntos. Exs.:
a) Sejam os conjuntos X o conjunto das letras da palavra AMOR, Y o conjunto das letras da
palavra ROMA, Z o conjunto das letras da palavra AMORAS e W o conjunto das letras da palavra
MARASMO. Desse modo so vlidas, dentre outras, as seguintes relaes:
X Y; Y X; X = Y (a ordem das letras no distingue os conjuntos X e Y).
X Z; Z X (pois S Z, mas S X). Logo, X Z.
Z W; W Z; Z = W (nem ordem nem repetio dos elementos interessa diretamente em
linguagem de conjuntos).
b) Suponha-se a pergunta: quantos elementos tem o conjunto A em que os cinco primeiros
nmeros inteiros positivos so escritos uma quantidade de vezes igual ao seu valor absoluto?
De acordo com a definio de A, pode-se afirmar que:
A = {1, 2, 2, 3, 3, 3, 4, 4, 4, 4, 5, 5, 5, 5, 5}.
Entretanto, pela definio de igualdade de conjuntos, fcil ver que:
A = {1, 2, 3} e, portanto, tem 3 elementos.
1.6. OPERAES ENTRE CONJUNTOS
1.6.1. UNIO (ou REUNIO)
Dados dois conjuntos quaisquer A e B, chama-se de unio de A com B ao conjunto representado
por A B que consiste em todos os elementos que pertencem a A ou a B, podendo pertencer a ambos.
Formalmente:
A B = {x
x A ou x B}
A

Convm ressaltar que a simbologia x A ou x B exige que pelo menos uma das duas
afirmaes, a saber, x A, x B, seja verdadeira, podendo ser, eventualmente, as duas. Embora na
linguagem cotidiana seja freqente a interpretao do conectivo ou apenas no sentido exclusivo, o seu
significado matemtico correto inclusivo. Assim, para exemplificar, esto corretas afirmaes como: 2
5 (dois menor que ou igual a cinco); 7 7 (sete maior que ou igual a sete), apesar de 2 = 5 e
7 > 7 serem ambas afirmaes falsas. Naturalmente a afirmao 2 4 falsa, uma vez que nem 2 > 4,
nem 2 = 4. Da mesma forma, a sentena Este ano no bissexto ou 2005 verdadeira, ainda que
ambas as afirmaes sejam verdadeiras (incluso). Exs.:
a) Sejam A = {1, 2, 3} e B = {3, 4, 5, 6}. Ento A B = {1, 2, 3, 4, 5, 6}
b) Sendo P = {p Np par} e I = {i Ni = 2k +1, k N}, tem-se que P I = N.

Captulo 1. Conjuntos
1.6.2. INTERSEO (ou INTERSECO)
A interseo entre A e B (A B) o conjunto dos elementos que pertencem a A e a B
(simultaneamente). Matematicamente:
A B = {x
x A e x B}
A

Exemplos:
a) Se A = {1, 2, 3} e B = {3, 4, 5, 6}, conclui-se que A B = {3}.
b) Sendo X = {x Nx mltiplo de 12} e Y = {y Ny mltiplo de 10}, tem-se que X Y
= {x Nx mltiplo de 60}.
c) Supondo P o conjunto dos nmeros primos e C o conjunto dos nmeros compostos, tem-se
que P C = , pois no h inteiros que sejam primos e compostos, ao mesmo tempo.
Quando dois conjuntos tm interseo vazia so chamados de disjuntos.
A

ALGUMAS PROPRIEDADES DA UNIO E DA INTERSEO


Quaisquer que sejam os conjuntos A, B e C so vlidos os seguintes resultados.
I.

IDEMPOTNCIA
A A = A; A A = A.

II.

COMUTATIVIDADE
A B = B A; A B = B A.

III.

ASSOCIATIVIDADE
(A B) C = A (B C);
(A B) C = A (B C).

IV. DISTRIBUTIVIDADE
A (B C) = (A B) (A C);
A (B C) = (A B) (A C).
7

Captulo 1. Conjuntos
V.

EXISTNCIA DO ELEMENTO NEUTRO


A = A; A U = A.

VI. EXISTNCIA DO ELEMENTO ABSORVENTE


A U = U; A = .
VII. A A B e A B B.
VIII. Se A B, ento A B = B e A B = A.
As demonstraes das sete primeiras propriedades acima (ou as idias nelas envolvidas) sero
melhor esclarecidas num momento posterior (em Lgica), embora seja possvel aplicar os conceitos
explanados at o momento. De um modo geral, uma ferramenta muito til para provar que dois
conjuntos so iguais consiste em usar a propriedade anti-simtrica da incluso de conjuntos.
Por exemplo, para provar, em IV, que A (B C) = (A B) (A C), pode-se proceder do
seguinte modo: Seja x um elemento qualquer de A (B C). Ento, por definio de unio, x A ou x
B C. Pela definio de interseo, pode-se escrever ento que x A ou x B e x C. Dessa
maneira, possvel afirmar que, ao mesmo tempo, x A ou x B e x A ou x C, isto , que x (A
B) (A C). Logo, qualquer elemento de A (B C) tambm um elemento de (A B) (A
C), ou seja, A (B C) (A B) (A C). Paralelamente, seja y um elemento qualquer de (A
B) (A C). Tem-se, por definio de interseo, que y A B e que y A C. Pela definio de
unio, pode-se escrever que y A ou y B, ao mesmo tempo em que y A ou y C. Em particular
(para evitar redundncias), basta dizer que y A ou y B e y C. Da, pode-se garantir que y um
elemento de A (B C). Conseqentemente, qualquer elemento de (A B) (A C) tambm deve
estar em A (B C). Noutros termos, (A B) (A C) A (B C). Finalmente, j que A (B
C) (A B) (A C) e (A B) (A C) A (B C), conclui-se que A (B C) = (A
B) (A C).
Como se pode verificar na demonstrao acima, as tcnicas envolvidas so simplesmente
manipulaes convenientes das palavras e das oraes. s vezes, por exemplo, escrever coisas
redundantes ou bvias til. Noutras, procura-se eliminar tais excessos. Por exemplo, na demonstrao
precedente, afirma-se que a sentena y A ou y B, ao mesmo tempo em que y A ou y C pode
ser substituda por y A ou y B e y C. Outro exemplo poder escrever, equivalentemente, que
x A e x B ou que x B e x A. Ou ainda, dizer que x A a mesma coisa que dizer (de
modo prolixo, mas eventualmente til) x A ou x A, ou ainda x A e x A. Depende da
convenincia (e, obviamente, de muita prtica para perceber). As regras matemticas formais de
manipulao como as acima sero vistas em Lgica Matemtica, a qual pode ser entendida como um
modo de pensar, escrever e falar matematicamente.
O raciocnio empregado na demonstrao de ltima propriedade anlogo. Suponha-se que A
seja uma parte de B. Para provar que A B = B, a tcnica ser, novamente, a anti-simetria. Se x A
B, por definio tem-se que x A ou x B. Como, por hiptese, A B, conclui-se que x B ou x
B, ou, mais simplesmente, que x B. Da, todo elemento de A B (na hiptese de A B) tambm
deve ser um elemento de B, isto , A B B. Seja, agora, um elemento b qualquer de B. Ento a
afirmao b A ou b B verdadeira, mesmo que b no esteja em A (uma vez que a definio de
reunio de dois conjuntos no exige que um elemento da unio esteja em ambos os conjuntos basta
pertencer a um deles). Portanto, b A B. Ou seja, todo elemento de B tambm elemento de A B:
B A B. Logo, caso A B, deve-se ter A B = B. Note-se que, mesmo que A no seja um
subconjunto de B, continua valendo B A B, pois esse resultado independe da hiptese adotada.
A demonstrao de que, na mesma hiptese, A B = A anloga e fica como exerccio.
8

Captulo 1. Conjuntos
1.6.3. DIFERENA
Dados os conjuntos quaisquer A e B, a diferena entre A e B consiste no conjunto dos elementos
de A que no so elementos de B, ou seja, aqueles elementos que esto, apenas, em A. Em smbolos:
A B = {x
x A e x B}
A

Exemplos:
a) Sendo A = {1, 2, 3} e B = {3, 4, 5, 6}, conclui-se que A B = {1, 2} e que B A = {4, 5, 6}. Notese que A B B A, e, portanto, a diferena no comutativa, de um modo geral.
b) Tomando X = {x Nx divisor de 12} e Y = {y Ny divisor de 20}, conclui-se que X Y =
{3, 6, 12} e Y X = {5, 10, 15, 20}.
c) Sendo Mn o conjunto dos mltiplos do inteiro n, pode-se garantir que M5 M10 o conjunto dos
inteiros terminados em 5. J M10 M5 = , uma vez que impossvel encontrar um mltiplo de 10
que no seja mltiplo de 5.
d) Supondo P o conjunto dos nmeros pares e I o conjunto dos nmeros mpares, tem-se que P I = P,
pois todos os nmeros pares no so mpares. Analogamente, I P = I.
ALGUMAS PROPRIEDADES DA DIFERENA
Para quaisquer que sejam os conjuntos A e B so vlidas as seguintes propriedades.
I.
II.
III.
IV.

A A = .
A = A (ELEMENTO NEUTRO).
A (A B) = A B.
Se A B = , ento A B, e vice-versa, isto , se A B, ento A B = . Formalmente: A B
= se, e somente se, A B, ou ainda, A B = A B.
DEMONSTRAES:

I.

Basta notar que a afirmao x A e x A sempre falsa, o que define o conjunto vazio. Ou seja:
A A = {x x A e x A} = . Note-se que no interessante utilizar a anti-simetria (pelo
menos por enquanto sem Lgica), visto que o conjunto vazio no tem elemento.

II.

Suponha-se que x A . Ento, por definio, x A e x . Particularmente, x A. Logo, A


A (de uma forma bem mais geral, fcil ver que A B A, qualquer que seja o conjunto B).
Inversamente, suponha-se que x A. A idia que a afirmao x auto-evidente (bvia),
podendo ser considerado uma espcie de redundncia. Isto significa que a sentena x A (sem
dvida, mais simples) pode ser re-escrita x A e x (mais prolixa, no entanto,
particularmente til). De fato, qualquer que seja o elemento x, pode-se sempre afirmar que x .
Portanto, A A . Assim, A = A.

III.

Suponha-se que x A (A B). Ento, x A e x A B. Como x deve ser um elemento de A,


mas no de A B, tem-se que x B. Assim, x A e x B, ou seja, x A B, do que A (A
B) A B. Agora, suponha-se que x A B. Ento, x A e x B. Se x no elemento de B,
9

Captulo 1. Conjuntos
47) (EPCAr-2004) Dados os conjuntos A, B e C
tais que [A (A B)] B = C , pode-se afirmar,
necessariamente, que
a) C (AXB)
b) n(A B) < n(B)
c) n(A C) > n(A B) n(B)
d) n(B C) = n(C)

aprovadas e 40 reprovadas, por conterem plulas


de farinha. No teste de quantidade, 74 foram
aprovadas e 26 reprovadas, por conterem um
nmero menor de plulas que o especificado. O
resultado dos dois testes mostrou que 14 caixas
foram reprovadas em ambos os testes. Quantas
caixas foram aprovadas em ambos os testes?
44) (UFRN-95) Dados os conjuntos A = {x IR;
x > 2} e B = {x IR; x < 4}, assinale a sentena
correta:
A) A B = {x / R ;2 < x < 4}
B) A B = / R
C) A B = {3}
D) A B = {x / R ;2 x 4}
E) A B =

48) (EPCAr-2005) Analise as afirmativas abaixo:


I Sejam A, B e C trs conjuntos no vazios. Se
ABe
C A , ento, (A C) B.
II Se A e B so dois conjuntos no vazios tais
que A B = {x IN | 1 x 8}, A B = {1, 3,
6, 7} e B A = {4, 8}, ento A B = .
III Dados os nmeros reais x tais que: x {x
IR | 1 < x 2}, {x IR | x < 0} e {x IR | x
3}; ento, a unio de todos os nmeros reais x o
conjunto {x IR | x 1 ou x 3}.
correto afirmar que
a) apenas II verdadeira.
b) apenas I falsa.
c) todas so falsas.
d) II e III so falsas.

45) (UFRN-96) De dois conjuntos A e B, sabe-se


que:
) O nmero de elementos que pertencem a
A B 45;
) 40% destes elementos pertencem a ambos os
conjuntos;
) O conjunto A tem 9 elementos a mais que o
conjunto B.
Ento, o nmero de elementos de cada conjunto :
a) n(A) = 27 e
n(B) = 18
n(B) = 21
b) n(A) = 30 e
c) n(A) = 35 e
n(B) = 26
d) n(A) = 36 e
n(B) = 27
e) n(A) = 38 e
n(B) = 29

49) Se A = {a, b, {a}, {b}, {a, b}}, em que a e b


so nmeros reais quaisquer, ento o (s) possvel
(eis) valor (es) do nmero de elementos de A
(so):
a) 1 ou 5.
b) 2 ou 5.
c) 3 ou 5.
d) 2 ou 3.
e) apenas 5.
50) (CBERJ-91) Dado o conjunto A = {; 1; 2;
{1}; {2}} a afirmao FALSA :
b) A
c) {1} A
a) A
d) ( {1} {2} ) A e) 2 A

46) (EPCAr-2003) Numa turma de 31 alunos da


EPCAR, foi aplicada uma Prova de Matemtica
valendo 10 pontos no dia em que 2 alunos
estavam ausentes. Na prova, constavam questes
subjetivas: a primeira, sobre conjuntos; a segunda,
sobre funes e a terceira, sobre geometria plana.
Sabe-se que dos alunos presentes
nenhum tirou zero;
11 acertaram a segunda e a terceira questes;
15 acertaram a questo sobre conjuntos;
1 aluno acertou somente a parte de geometria
plana,
e
7 alunos acertaram apenas a questo sobre
funes.
correto afirmar que o nmero de alunos com
grau mximo igual a 10 foi
a) 4
c) 6
b) 5
d) 7

51) (PUC/SP) Se A = e B = {}, ento:


a) A B
b) A B =
c) A = B
d) A B = B
e) B A
52) (Vunesp-84) Suponhamos que:
A B = {a, b, c, d, e, f, g, h}
A B = {d, e}
A B = {a, b, c}
Ento:
a) B = {f, g, h}
b) B = {d, e, f, g, h}
c) B = {a, b, c, d, e} d) B = {d, e}
e) B =

41

Captulo 1. Conjuntos
53) Determine os conjunto A, B e C que
satisfazem as seguintes seis condies:
1) A B C = {z, x, v, u, t, s, r, q, p}
2) A B = {r, s}
3) B C = {s, x}
4) C A = {s, t}
5) A C = {p, q r, s, t, u, v, x}
6) A B = {p, q, r, s, t, u, x, z}

um portador desta molstia apresenta apenas um


subconjunto no vazio de S.
Assinale a nica alternativa correspondente ao
nmero de subconjuntos de S que podero
apresentar os pacientes desta molstia.
a) 7 b) 8 c) 16 d) 15 e) 14
58) (Escola Naval-89) Considere os conjuntos A
= { x } e B = { x, { A } } e as proposies
I. { A } B
II. { x } A
III. A B
IV.
BA
V. { x, A } B
As proposies FALSAS so
a) I, III e V b) II, IV e V c) II, III, IV e V
d) I, III, IV e V
e) I, III e IV

54) (IME-76) Considere um conjunto E e trs de


seus subconjuntos, A, B, C. Sendo M um
subconjunto de E, represente por ME o seu
complemento em relao a E. Determine E e os
subconjuntos A, B, C, sabendo que A e C so
disjuntos e que:
(A B C)E = {4, 6}
... (1)

59) (Colgio Naval-85) A, B e C so


respectivamente os conjuntos dos mltiplos de 8,
6 e 12, podemos afirmar que o conjunto
A (B C) o conjunto dos mltiplos de:
a) 12 b) 18 c) 24 d) 48 e) 36

B C = {7}
... (2)
A B = {1, 2, 7, 9, 10}
... (3)
A C = {1, 2, 3, 5, 7, 8, 9, 10}
... (4)

60) (Colgio Naval-87) Dados dois conjuntos A e


B tais que:
- o nmero de subconjuntos de A est
compreendido entre 120 e 250.
- B tem 15 subconjuntos no vazios.
O produto cartesiano de A por B tem
a) 8 elementos
b) 12 elementos
c) 16 elementos
d) 28 elementos
e) 32 elementos

BE = {3, 4, 5, 6, 8, 9}
... (5)
55) (Colgio Naval-97) Considere o conjunto A
dos nmeros primos positivos menores do que 20
e o conjunto B dos divisores positivos de 36. O
nmero de subconjuntos do conjunto diferena B
A
a) 32 b) 64 c) 128 d) 256 e) 512

61) (EPCAr-90) Se um conjunto A tiver 4


elementos e um conjunto B tiver 3 elementos,
ento o conjunto de todas as partes do conjunto A
x B (A cartesiano B) ter um nmero de
elementos equivalente a:
a) 23 b) 24 c) 27 d) 212 e) 214

56) (Colgio Naval-87) Considere os conjuntos A


= {1, {1}, 2} e B = { 1, 2, {2}} e as cinco
afirmaes:
I- A B = {1}
II- {2} (B A)
III- {1} A
IV- A B = {1, 2, {1, 2}}
V- B A = {{2}}
Logo,
a) todas as afirmaes esto erradas
b) se existe uma afirmao correta
b) as afirmaes mpares esto corretas
d) as afirmaes III e V esto corretas
e) as afirmaes I e IV so as nicas incorretas

62) (Colgio Naval-00) Dados dois conjuntos A e


B tais que n(A B) = 10, n(A B) = 5 e n(A) >
n(B), pode-se afirmar que a soma dos valores
possveis para n(A B)
a) 10 b) 11 c) 12 d) 13 e) 14
63) (Colgio Naval-87) Representando-se por n
(X) o nmero de elementos de um conjunto X,
considere dois conjuntos A e B tais que n (A B)

57) (UFPE-84) Seja S = {S1, S2, S3} o conjunto de


sintomas de uma determinada molstia. Em geral,
42

Captulo 1. Conjuntos
= 4, n (A B) = 5 e n (A x B) = 36. Podemos
afirmar que n (A B) igual a:
a) 4 b) 6 c) 7 d) 9 e) 10

tero desses alunos, quantos no foram aprovados


em nenhuma das duas universidades?
a) 15 b) 20 c) 21 c) 30 e) 31

64) (Colgio Naval-88) Dados os conjuntos M, N


e P tais que N M, n (M N) = 60% n (M), n (N
P) = 50 % n (N), n (M N P) = 40% n (P) e
n (P) = x% n (M), o valor de x :
a) 80 b) 75 c) 60 d) 50 e) 45

70) (Colgio Naval-84) Num colgio verificou-se


que 120 alunos no tm pai professor; 130 alunos
no tm me professora e 5 tm pai e me
professores. Qual o nmero de alunos do
colgio, sabendo-se que 55 alunos possuem pelo
menos um dos pais professor e que no existem
alunos irmos?
a) 125 b) 135 c) 145 d) 155 e) 165

65) (FGV/SP) Sejam A, B e C conjuntos finitos.


O nmero de elementos de A B 30, o nmero
de elementos de A C 20 e o nmero de
elementos de A B C 15.
Ento o nmero de elementos de A (B C)
igual a:
a) 35 b) 15 c) 50 d) 45 e) 20

71) (FUVEST/SP) Depois de n dias de frias, um


estudante observa que:
a) choveu 7 vezes, de manh ou tarde;
b) quando chove de manh, no chove tarde;
c) houve 5 tardes sem chuva;
d) houve 6 manhs sem chuva.
Podemos afirmar ento que n igual a:
a) 7 b) 8 c) 9 d) 10 e) 11

66) (Colgio Naval-97) Dados os conjuntos A , B


e C , tais que n( B C ) = 20 , n( A B ) = 5 ,
n( A C ) = 4 ,
n( A B C ) = 1
e
n(A B C) = 22 , o valor de n[ A ( B C )] :
a) 10 b) 9 c) 8 d) 7 e) 6

72) (Colgio Naval-95) Num concurso, cada


candidato fez uma prova de Portugus e uma de
Matemtica. Para ser aprovado, o aluno tem que
passar nas duas provas. Sabe-se que o nmero de
candidatos que passaram em Portugus o
qudruplo do nmero de aprovados no concurso;
dos que passaram em Matemtica o triplo do
nmero de candidatos aprovados no concurso; dos
que no passaram nas duas provas a metade do
nmero de aprovados no concurso; e dos que
fizeram o concurso 260. Quantos candidatos
foram reprovados no concurso?
a) 140 b) 160 c) 180 d) 200 e) 220

67) (Colgio Naval-86) Considere os conjuntos M


pares ordenados (x, y) que satisfazem a equao
(a1x + b1y + c1) . (a2x + b2x + c2) = 0 e N dos
pares ordenados (x, y) que satisfazem o sistema
a 1 x + b1 y + c1 = 0

a 2 x + b 2 y + c 2 = 0
sendo a1. b1. c1. a2. b2. c2 0, pode-se afirmar que
a) M = N
b) M N = M c) M N =
d) M N = N
e) M N

73) (AMAN-90) A frmula A B = A B pode


definir a diferena de dois conjuntos usando
somente as operaes de interseo e
complemento. Da mesma forma, A B pode ser
representada por:
a) [A B] [B A] [A B]
b) [A B] B
c) [A B] [B A] [A B]
d) [A B] B
e) [A + B]

68) (IME-75) Em uma pesquisa realizada entre


500 pessoas foram obtidos os seguintes dados:
200 pessoas gostam de msica clssica;
400 pessoas gostam de msica popular;
75 pessoas gostam de msica clssica e de msica
popular.
Verifique a consistncia ou inconsistncia
dos dados desta pesquisa.
69) Trinta e seis alunos de uma determinada
escola prestaram exames vestibulares em duas
universidades, A e B, sendo que, desse grupo de
alunos, todos os aprovados em A tambm foram
aprovados em B e o nmero de aprovados em B
foi o triplo do nmero de aprovados em A. Se
foram aprovados menos da metade e mais de um

74) (FGV-80) Numa pesquisa de mercado, foram


entrevistadas vrias pessoas acerca de suas
preferncias em relao a 3 produtos: A, B e C.
Os resultados da pesquisa indicaram que:
210 pessoas compram o produto A.
43

Captulo 1. Conjuntos
III. 980 filiados votaram a favor de A ou de B,
mas no de C;
IV. 420 filiados votaram a favor de B, mas no de
A ou de C;
V. 1.220 filiados votaram a favor de B ou de C,
mas no de A;
VI. 640 filiados votaram a favor de C, mas no de
A ou de B;
VII. 140 filiados votaram a favor de A e de C,
mas no de B.
Determine o nmero de filiados ao PE que:
a) votaram a favor dos 3 candidatos.
b) votaram a favor de apenas um dos candidatos.

210 pessoas compram o produto B.


250 pessoas compram o produto C.
20 pessoas compram os 3 produtos.
100 pessoas no compram nenhum dos 3
produtos.
60 pessoas compram os produtos A e B.
70 pessoas compram os produtos A e C.
50 pessoas compram os produtos B e C.
Quantas pessoas foram entrevistadas?
a) 670 b) 970 c) 870 d) 610 e) 510
75) Na questo anterior quantas pessoas
compraram exatamente um produto? E quantas
compraram exatamente dois produtos?

79) (Colgio Naval-89) Num grupo de 142


pessoas foi feita uma pesquisa sobre trs
programas de televiso A, B e C e constatou-se
que:
I 40 no assistem a nenhum dos trs
programas;
II 103 no assistem ao programa C;
III 25 s assistem ao programa B;
IV 13 assistem aos programas A e B;
V O nmero de pessoas que assistem somente
aos programas B e C a metade dos que assistem
somente a A e B;
VI 25 s assistem a 2 programas; e
VII 72 s assistem a um dos programas.
Pode-se concluir que o nmero de pessoas que
assistem:
a) ao programa A 30.
b) ao programa C 39.
c) aos 3 programas 6.
d) aos programas A e C 13.
e) aos programas A ou B 63.

76) Um total de 34 estudantes estrangeiro veio ao


Brasil. Todos eles visitaram Manaus, So Paulo
ou Salvador. 16 deles visitaram Manaus; 16
visitaram So Paulo e 11 visitaram Salvador.
Desses estudantes, 5 visitaram Manaus e Salvador
e, desses 5, 3 visitaram tambm So Paulo.
correto, ento, afirmar que
a) 27 estudantes visitaram apenas uma dessas trs
capitais.
b) 29 estudantes visitaram apenas uma dessas trs
capitais.
c) 3 estudantes visitaram exatamente duas
capitais.
d) possvel ter certeza da quantidade de alunos
que visitaram apenas Salvador.
e) os dados da questo so incoerentes entre si.
77) (Colgio Naval-83) Numa cidade constatou-se
que as famlias que consomem arroz no
consomem macarro. Sabe-se que: 40%
consomem arroz; 30% consomem macarro; 15%
consomem feijo e arroz; 20 % consomem feijo
e macarro; 60% consomem feijo.
A porcentagem correspondente s famlias que
no consomem esses trs produtos :
a) 10%
b) 3%
c) 15%
d) 5%
e) 12%

80) (UnB-01) Em uma pesquisa realizada com um


grupo de 100 turistas, constatou-se que 42 falam
ingls, 12 falam ingls e italiano, 18 falam
espanhol e ingls e 16 falam espanhol e italiano.
O nmero de turistas que falam espanhol
precisamente 50% maior do que o nmero
daqueles que falam italiano. Com base nessas
afirmaes, julgue os itens a seguir.
(1)
O nmero de turistas que falam italiano
igual a 2/3 do nmero dos que falam espanhol.
(2)
Se 9 dos turistas consultados falam as trs
lnguas, espanhol, ingls e italiano, enquanto 5
deles no falam nenhuma dessas lnguas, ento
mais da metade dos turistas falam espanhol.
Se 9 dos turistas consultados falam as trs
(3)
lnguas, espanhol, ingls e italiano, enquanto 5

78) (FGV-2004) Numa cidade do interior do


estado de So Paulo, uma prvia eleitoral entre
2.000 filiados revelou as seguintes informaes a
respeito de trs candidatos A, B, e C, do Partido
da Esperana (PE), que concorrem a 3 cargos
diferentes:
I. todos os filiados votaram e no houve registro
de voto em branco, tampouco de voto nulo;
II. 280 filiados votaram a favor de A e de B;
44

Captulo 1. Conjuntos
d) 18

91) (EsPCEx-2003) Quaisquer que sejam o


nmero irracional a e o nmero racional b, podese afirmar que, sempre,
A) a . a irracional.
B) a2 + b racional.
C) a . b racional.
D) b a + 2 irracional.
E) b + 2a irracional.

e) 25

96) (ITA-01) Sejam X, Y e Z subconjuntos


prprios de R, no-vazios. Com respeito s
afirmaes:
I. x {[Y (X Y)C] [X YC)C}
II. Se Z X ento (Z Y) (X (ZC Y)} =
X Y.
C
III. Se (X Y) Z ento ZC X.
temos que:
a) apenas I verdadeira.
b) apenas I e II so verdadeiras.
c) apenas I e III so verdadeiras.
d) apenas II e III so verdadeiras
e) todas so verdadeiras.

92) (EsPCEx-2004) Dados os nmeros a = 3

1, b = 3 +1 e c = 0,1333..., pode-se afirmar que:


a) a.b um nmero irracional.
b) (a b).c um nmero irracional.
c) (a+b).c um nmero racional.
d) b.c um nmero racional.
e) a.b.c um nmero racional.

97) (ITA-03) Sejam U um conjunto no-vazio e A


U, B U. Usando apenas as definies de
igualdade, reunio, interseco e complementar,
prove que:
I Se A B = , ento B AC.
II B \ AC = B A.

93) (ITA-96) Sejam A e B subconjuntos no


vazios de R, e considere as seguintes afirmaes:
I- (A - B)C (B AC)C =
II- (A - BC)C = B - AC
III- [(AC - B)(B - A)]C = A
Sobre essas afirmaes podemos garantir que:
a) Apenas a afirmao I verdadeira.
b) Apenas a afirmao II verdadeira.
c) Apenas a afirmao III verdadeira.
d) Todas as afirmaes so verdadeiras.
e) Apenas as afirmaes I e III so verdadeiras.

98) (ITA-04) Considere as seguintes afirmaes


sobre o conjunto U = {0, 1, 2, 3, 4, 5, 6, 7, 8, 9}:
I - U e n (U) = 10.
II - U e n (U) = 10.
III 5 U e {5} U.
IV {0, 1, 2, 5} {5} = 5
Pode-se dizer, ento, que (so) verdadeira(s).
a) apenas I e III b) apenas II e IV c) apenas II
e III d) apenas IV e) todas as afirmaes

94) (ITA-99) Sejam E, F, G e H subconjuntos no


vazios de R. Considere as afirmaes:
I - Se (E x G) (F x H), ento E F e G H.
II - Se (E x G) (F x H), ento (E x G) (F x
H) = F x H.
III - Se (E x G) (F x H) = F x H, ento (E x G)
(F x H).
Ento:
a) Apenas a afirmao (I) verdadeira.
b) Apenas a afirmao (II) verdadeira.
c) Apenas as afirmaes (II) e (III) so
verdadeiras.
d) Apenas as afirmaes (I) e (II) so verdadeiras.
e) Todas as afirmaes so verdadeiras.

99) (ITA-04) Seja A um conjunto no-vazio.


a) Se n(A) = m, calcule n(P(A)) em termos de m.
b) Denotando P1(A) = P(A) e P k + 1(A) = P
(Pk(A)), para todo nmero natural k 1,
determine o menor k, tal que n(Pk(A)) 65000,
sabendo que n(A) = 2.
100) Suponha-se que sejam verdadeiras as
seguintes afirmaes:
I. Os bebs no so lgicos.
II. Quem consegue amestrar um crocodilo
no desprezado.
III. Pessoas ilgicas so desprezadas.
possvel afirmar que, dentre as proposies a
seguir, a verdadeira deve ser:
a) Nenhum beb desprezado.
b) Existem bebs que sabem amestrar
crocodilos.

95) (ITA-00) Denotemos por n(X) o nmero de


elementos de um conjunto finito X. Sejam A, B e
C conjuntos tais que n(AB) = 8, n(AC) = 9,
n(BC) = 10, n(ABC) = 11 e n(ABC) = 2.
Ento n (A) + n (B) + n (C) igual a :
a) 11
b) 14
c) 15
47

Captulo 1. Conjuntos
c) Bebs no sabem amestrar crocodilos.
d) Pessoas que sabem amestrar crocodilos
podem no ser lgicas.
e) Toda pessoa desprezada um beb.

f)
g)
h)
i)
j)
k)
l)
m)
n)
o)
p)
q)
r)
s)
t)
u)

101) Considerem-se como verdadeiras as


seguintes proposies:
I. Todos os advogados so ricos.
II. Poetas so temperamentais.
III. Carlos um advogado.
IV. Nenhuma pessoa temperamental rica.
Pode-se garantir, nestas condies, que deve ser
correta a afirmao:
a) Todas as pessoas ricas so advogadas.
b) Todas as pessoas temperamentais so
poetas.
c) Existem advogados poetas.
d) Carlos no um poeta.
e) Existem poetas ricos.

X Y (X Z) (Y Z).
X Y (X Z) (Y Z).
X Y = X Y X = Y.
(X Y) Z = X (Y Z) Z X.
X Y X.
X Y = X X Y = .
X Y = X Y.
(X Y) (X Y) = .
(X Y) (X Y) = X.
X (Y Z) = (X Y) Z.
Y (X Y) = .
(X Y) Z = (X Z) (Y Z).
(X Y) Z = (X Z) (Y Z).
(X Y) Z = X (Y Z).
X Z = Y Z X = Y;
X Y = X = Y.

106) Se A B = A C, verdade que B = C? Se


A B = A C, ento B = C? E se A x B = A x
C, ento B = C?

102) Uma pessoa ctica quanto s boas intenes


da humanidade afirma que 70% dos homens so
desonestos, 70% so intolerantes e 70% so
violentos. Se ela estiver certa, numa amostra
perfeita de 100 homens, qual o nmero mnimo
de
pessoas
simultaneamente
desonestas,
intolerantes e violentas?

107) Dados os conjuntos A e B, seja X um


conjunto com as seguintes propriedades:
1 X A e X B.
2 Se Y A e Y B ento Y X.
Prove que X = A B.

103) Numa pesquisa realizada em uma turma


militar, constatou-se que 60% dos entrevistados
desejavam prestar concurso para a ESCOLA
NAVAL, 70% para o IME e 80% para o ITA.
Sabendo que qualquer dos entrevistados almeja
fazer as provas de uma dessas instituies, qual o
percentual mnimo de alunos que querem prestar
os trs concursos?

108) Enuncie e demonstre um resultado anlogo


ao anterior, caracterizando A B.
109) Sendo A e B conjuntos, prove que A B =
se, e somente se, A BC. Prove tambm que A
B = U se, e somente se, AC B.
110) Prove que se A X = e A X = U ento
X = AC .

104) (EN-88) Se 70% da populao gostam de


samba, 75% de choro, 80% de bolero e 85% de
rock, quantos por cento da populao, no mnimo,
gostam de samba, choro, bolero e rock?
a) 5% b) 10% c) 20% d) 45% e) 70%

111) Se A B, ento B (A C) = (B C)
A para todo conjunto C. Por outro lado, se existir
C de modo que a igualdade acima seja satisfeita,
ento A B.

105) Provar que, sendo X, Y, Z e W conjuntos,


valem as seguintes propriedades:
a) X Z e Y Z X Y Z e X Y
Z.
b) X Z e Y W X Y Z W e X
Y Z W.
c) X (Y Z) = (X Y) (X Z).
d) X = X Y Y X.
e) X = X Y X Y.

112) Prove que A = B se, e somente se, (A BC)


(AC B) = .
113) Sejam A, B e C conjuntos quaisquer.
Demonstre as afirmaes verdadeiras e d contraexemplos para as falsas.
a) Se A B e B C, ento A C.
b) (A B)C = AC B.
48

Captulo 1. Conjuntos
c)
d)
e)
f)
g)
h)

A (B C) = A (B C).
(A B) C = (A C) (B C).
(A B) C = (A C) (B C).
Se X Y ento P (X) P (Y).
Se X Y ento P (Y X) = P (Y) P (X).
A B se, e somente se, A BC = .

d) 1 e 4 so verdadeiras
e) 1 e 3 so verdadeiras
117) (IME-87) Dados dois conjuntos A e B,
define-se A B = (A B) (B A)
Prove que dados trs conjuntos arbitrrios X, Y e
Z
X (Y Z) = (X Y) (X Z)

114) (ITA-85) Sejam X um conjunto no vazio; A


e B dois subconjuntos de X. Definimos AC = {x
X tal que x A} e A B = {x A tal que x
B}.
Dadas as sentenas:
1 A B = A BC B AC, onde
significa equivalente e o conjunto vazio;
2 Se X = IR; A = {x IR tal que x3 1 = 0}; B
= {x IR tal que x2 1 = 0} e C = {x IR tal
que x 1 = 0}, ento A = C = B;
3 A = A e A B = A (A B);
4 A B A BC;
podemos afirmar que est (esto) correta (s):
a) as sentenas n 1 e n 3.
b) as sentenas n 1, n 2 e n 4.
c) as sentenas n 3 e n 4.
d) as sentenas n 2, n 3 e n 4.
e) apenas a sentena n 2.

118) (Colgio Naval-88) Sendo a e b nmeros

a
R = x x = , b 0
e
inteiros quaisquer,
b

S = 2;1,3;0,444...; 2 , ento:
a) S R
b) S R = c) S R unitrio d)
S R tem dois elementos e) S R unitrio

119) (Provo-98) Uma das afirmativas abaixo


sobre os nmeros naturais FALSA. Qual ela?
a) Dado um nmero primo, existe sempre um
nmero primo maior do que ele.
b) Se dois nmeros no primos so primos entre
si, um deles mpar.
c) Um nmero primo sempre mpar.
d) O produto de trs nmeros naturais
consecutivos mltiplo de seis.
e) A soma de trs nmeros naturais consecutivos
mltipla de trs.

115) (ITA-87) Sejam F e G dois subconjuntos no


vazios de IR.
Assinale a alternativa CORRETA.
a) Se F G e G F, ento necessariamente F =
F G.
b) Se F G o conjunto vazio, ento
necessariamente F G = IR.
c) Se F G e G F, ento F G = F G.
d) Se F G = F, ento necessariamente G F.
e) Se F G = G e G IR, ento (F G) G =
IR.

120) (Colgio Naval-96) Dadas as operaes:


x * y = x + y; x y = x - y e xy = x y , o valor da
expresso:

[2 * (8 12)] * {[(3 * 2) 5][10 * (2 ( 42))]}

a) No um nmero real
c) igual a 2
e) igual a 4

b) igual a 1
d) igual a 3

121) Um subconjunto X de nmeros naturais


contm 12 mltiplos de 4, 7 mltiplos de 6, 5
mltiplos de 12 e 8 nmeros mpares. Qual o
nmero de elementos de X?

116) (ITA-89) Sejam A, B e C subconjuntos de


IR, no vazios, e A B = {p IR; p A e p
B}. Dadas as igualdades:
( 1 ) (A B) x C = (A x C) (B x C)
( 2 ) (A B) x C = (A x B) (B x C)
( 3 ) (A B) A (B A) B
( 4 ) A (B C) = (A B) (A C)
( 5 ) (A B) (B C) = (A C) (A B)
podemos garantir que
a) 2 e 4 so verdadeiras
b) 1 e 5 so verdadeiras
c) 3 e 4 so verdadeiras

122) (Fuvest-95) Dividir um nmero por 0,0125


equivale a multiplic-lo por
a) 1/125
b) 1/8
c) 8
d) 12,5
e) 80
123) (Escola Naval-90) O 1989 algarismo depois
5
:
da vrgula na expanso decimal de
39
a) 0 b) 1 c) 2 d) 5 e) 8
49

Captulo 1. Conjuntos
a c
a a+c c
<
< <
b d
b b+d d

144) (Provo-2001) Considere os intervalos


fechados A = [1, 3] e B = [2, 4] e as seguintes
afirmaes:
I.
para todo x A, existe y B tal que x
y;
II.
existe x A tal que, para todo y B,
x y;
III.
para todos x A e y B, x y;
existem x A e y B tais que x y.
IV.
Ento:
a) I falsa
b) II falsa c) III falsa
d) IV falsa e) todas so verdadeiras

151) Suponha-se que a, b, c, d sejam nmeros


racionais, que m e n sejam inteiros e que n m seja
irracional. Provar que:
a + b.n m = c + d.n m a = c e b = d .
152) Sejam a, b nmeros racionais positivos.
Prove que a + b racional se, e somente se,
a e

153) Um nmero positivo somado com o seu


inverso. a) Qual o menor valor possvel que pode
ser obtido para tal soma? b) Sob que condio?

145) (Provo-2001) O conjunto das solues da


1+ x
inequao
1
1 x
a) [0, )
b) [0, 1)
c) (1, )
d) (, 0]
e) (, 0] (1, )

154) A rea de um terreno retangular deve ser de


100 m2. Quais devem ser as dimenses do terreno
(comprimento e largura), de modo que o terreno
possua o menor contato possvel com o exterior?

146) a) Mostre, por meio de um exemplo, que


existe um nmero irracional tal que 4 e
6 so nmeros racionais.
b) Mostre que, se 7 e 12 so racionais,
ento racional.

155) a) Sabendo que dois nmeros somam 10,


qual o maior valor possvel para o seu produto?
b) Com 400 metros de arame, deseja-se cercar um
terreno na forma de um retngulo. Qual a maior
rea possvel que pode ser obtida em tais
condies?

147) a) Mostre que 4 + 2 3 = 1 + 3 .


b) Mostre que, sendo a, b e a2 b nmeros
racionais positivos, ento vale a identidade:

156) Sejam a, b, x e y nmeros positivos, com a e


b dados. Prove que, se xy = c (constante
conhecida), a soma ax + by assume seu valor
mnimo quando ax = by = abc .

a + a2 b
a a2 b
a b =

,
2
2
conhecida como frmula do radical duplo.
c) Obter racionais a e b, de modo que

157) Deseja-se cavar um buraco retangular com 1


m de largura de modo que o volume cavado tenha
300 m3. Sabendo que cada metro quadrado de
abertura custa 10 reais e cada metro de
profundidade custa 30 reais, determinar o
comprimento e a profundidade do buraco, a fim
de que seu custo seja o menor possvel.

18 8 2 = a + b 2 .
148) Seja X um conjunto ordenado qualquer.
Provar que, sendo a e b elementos de X:
a2 + b2 = 0 a = b = 0.
149) Provar que: 0 < a < b a
2ab
a+b
<
< ab <
< b. Provar tambm que,
a+b
2
sendo a e b positivos:
ab =

b so ambos racionais.

a+b
2ab a + b
a+b

=
ab =
a =b.
2
a+b
2
2

150) Suponha-se que a, b, c, d sejam elementos de


um conjunto ordenado X, com b e d positivos.
Provar que:

52

Captulo 2. Funes
f(x) = f(x)

(x, f(x))

(x, f(x))

II. Uma funo real de varivel real f: A B, definida num domnio simtrico, mpar se, e somente
se, o grfico de f simtrico em relao origem, o que equivalente a afirmar que o grfico de f possui
os dois eixos ordenados como eixo de simetria. Ou, ainda, que a origem um centro de simetria do
grfico de f.

DEMONSTRAO
A demonstrao anloga do teorema anterior, bastando apenas notar que a origem (0, 0) o
ponto mdio do segmento de extremidades nos pontos (x, f(x)) e (x, f(x)) = (x, f(x)), como
imediato. Assim, pode-se garantir que a funo f mpar se, e somente se, a ocorrncia do ponto (x, f(x))
em G(f) implica o surgimento do ponto (x, f(x)) no mesmo grfico.

f(x)
x

(x, f(x))
0

f(x)
(x, f(x))
OBSERVAO
De maneira similar possvel demonstrar o seguinte resultado para funes inversas.
Seja f uma funo real de varivel real bijetiva. Se os grficos de f e de sua inversa so traados
num mesmo sistema cartesiano, ento um o simtrico do outro em relao bissetriz dos quadrantes
mpares.
Para a demonstrao, s notar que, dado qualquer ponto (x, f(x)) = (x, y) de G(f), o ponto (y, x)
= (y, f 1 (y)) = (f(x), x) pertencer ao grfico de f 1. Dessa forma, j que a reta y = x (bissetriz dos
quadrantes mpares) a mediatriz do segmento de extremidades nos pontos (x, f(x)) e (f(x), x), as
curvas G(f) e G(f 1) so simtricas em relao quela bissetriz, quando representadas num mesmo
sistema cartesiano.
G(f)
y
y=x
(x0, y0)
y0 = f (x0)
G(f 1 )
x0

(y0, x0)

x0 y0 = f (x0) x

92

Captulo 2. Funes

III.
Suponha-se que f1 e f2 sejam funes pares e que g1 e g2 sejam funes mpares, todas reais de
varivel real. Sendo possvel definir as operaes funcionais a seguir em domnios convenientes
(comuns e simtricos), ento:
f
g
a) f1 + f2, f1.f2, g1.g2, 1 e 1 so pares.
f2
g2
f
b) g1 + g2, f1.g1 e 1 so mpares.
g1

DEMONSTRAO
Apenas para exemplificar, prove-se que f1 + f2 e g1.g2 so pares, assim como g1 + g2 e

f1
so
g1

mpares. Com efeito:


(f1 + f2)(x) = f1 (x) + f2 (x) = f1 (x) + f2 (x) = (f1 + f2)( x), ou seja, f1 + f2 par.
(g1.g2)(x) = g1 (x).g2 (x) = [ g1 (x)]. [ g2 (x)] = g1 (x).g2 (x) = (g1.g2)( x), isto , g1.g2 par.
Analogamente:
(g1 + g2)(x) = g1 (x) + g2 (x) = [ g1 (x)] + [ g2 (x)] = [g1 (x) + g2 (x)] = (g1 + g2)( x), ou seja, g1
+ g2 mpar.
f1
f
f ( x )
f (x )
f (x )
( x ) = 1
= 1
= 1
= 1 (x )
g 1 ( x ) g 1 (x )
g 1 (x )
g1
g1
As demonstraes restantes so similares e ficam como exerccio.
importante notar que nada pode afirmar-se sobre a diferena entre uma funo par e uma
funo mpar, como f1 g1, por exemplo. Com efeito:
(f1 g1)(x) = f1 (x) g1 ( x) = f1 (x) + g1 (x), que no obrigatoriamente idntica nem a (f1 g1)(x),
nem a (f1 g1)(x), portanto, nem par, nem mpar, necessariamente. De modo anlogo, a paridade da
soma de uma funo par com uma funo mpar, como f1 + g1 inconclusiva, conforme fcil notar
(inclusive com contra-exemplos).
IV.
Com as mesmas notaes da propriedade anterior, supondo que as seguintes composies
estejam bem definidas, pode-se garantir que:
a) f1 f2 e g1 g2 so pares (isto , a composta de duas funes com a mesma paridade par).
b) f1 g1 e g2 f2 so mpares (ou seja, a composta de duas funes de paridades distintas mpar).

DEMONSTRAO
Apenas para exemplificar, tem-se que:
(g1 g2)(x) = g1(g2(x)) = g1(g2(x)) = [g1(g2(x))] = g1(g2(x)) = (g1 g2)(x), ou seja, a composta de
duas funes mpares uma funo par.
Os demais casos so anlogos e ficam como exerccio.

93

Captulo 2. Funes

2.12. FUNES PERIDICAS


2.12.1. DEFINIES
Uma funo dita peridica quando existe um nmero real T, positivo, de modo que as imagens
se repitam de T em T. Mais precisamente:
f: A B peridica T > 0, f (x + T) = f (x), x A.
O nmero T denominado perodo de f. Perceba-se que a definio exige que T no dependa do
particular valor de x escolhido em A (pois f (x + T) = f (x) deve ser uma identidade, isto , valer em
qualquer x do domnio). Deve-se observar, tambm, a necessidade natural de no somente x, como
tambm x + T serem ambos elementos do domnio de f.
Quando existe um nmero real positivo T0 que o menor dentre todos os demais perodos de f,
T0 dito perodo fundamental de f. Em termos mais precisos, dada uma funo peridica f: A B:
T0 o perodo fundamental de f f (x + T0) = f (x) e 0 < T0 T, T: f (x + T) = f (x).
Exemplos:
1. Qualquer funo constante pode ser considerada como uma funo peridica, em que um
perodo pode ser qualquer nmero real. Com efeito, se f (x) = k, para qualquer x, tem-se inclusive
que f (x + T) = k = f (x), independentemente do valor T escolhido. Note-se que, como no existe o
menor nmero real positivo, nenhuma funo constante (embora peridica) possui um perodo
fundamental.
2. Considere-se uma funo trigonomtrica do tipo f: R R, dada por f (x) = m + n.sen (a.x + b), em
que n.a 0. Suponha-se que f seja peridica. Se isso de fato ocorrer, deve ser possvel encontrar algum
nmero real positivo T, de modo que: f (x + T) = f (x). Mas, ento:
m + n.sen [a.(x + T) + b) = m + n.sen (a.x + b) sen [a.(x + T) + b) = .sen (a.x + b) (*). Como se sabe,
dois ngulos e tm o mesmo seno se, e somente se, = + 2k ou = ( ) + 2k, em que k
um nmero inteiro. Assim, a igualdade em (*) ocorre somente quando:
a.(x + T) + b = a.x + b + 2k (I) ou a.(x + T) + b = (a.x + b) + 2k (II). Portanto, a fim de que T seja
2 k
(por I) ou
um perodo de f, deve-se impor T =
a
2ax 2b+2k
T=
(por II). Neste ponto, deve-se notar que a segunda opo no convm, visto que T
a
2 k
dependeria do valor de x escolhido. Portanto, qualquer perodo de f deve assumir a forma T =
, em
a
2
que k 0. Mais ainda, o perodo fundamental de f igual a T =
.
a
OBSERVAO: Um argumento inteiramente anlogo prova que as funes f: R R, com f (x) = m +
2
n.cos (a.x + b) so tambm peridicas, de perodo fundamental T =
, bem como qualquer funo f:
a

R R de lei f (x) = m + n.tg (a.x + b) possui perodo fundamental T = .


a
3. Nenhuma funo afim f: R R, f (x) = ax + b, em que a 0, pode ser peridica. Em verdade, caso
exista algum nmero real para o qual f (x + T) = f (x), ento:
a(x + T) + b = ax + b a.T = 0 T = 0 (j que a 0).

94

Captulo 2. Funes

4. Analogamente, nenhuma funo quadrtica f: R R, f (x) = ax + bx + c, com a 0, pode ser


peridica. Com efeito:
f (x + T) = f (x) a(x + T)2 + b(x + T) + c = ax2 + bx + c 2axT + aT2 + bT = 0 2ax + aT + b = 0
2ax + b
(uma vez que a 0). Da, T deveria ser
, o que obviamente conveniente, visto que T deve
a
ser independente de x.
2

5. Denomina-se maior (ou mximo) inteiro que no supera x o (nico) nmero inteiro k, tal que:
x 1 < k x.
Utiliza-se a notao k = [x]. O inteiro k tambm recebe o nome de parte inteira de x.
Assim, por exemplo: [5,7] = 5; [6] = 6; [ 91] = 91; [ 713, 94] = 714.
fcil provar que, dentre vrias outras propriedades:
[x] = x x Z (pois no h mais que um inteiro no intervalo (x 1, x], de comprimento menor
que 1; e, quando isso ocorre, o inteiro deve ser x).
0 x [x] < 1. De fato: x 1 < k = [x] x x [x] < 1 x 0 x [x] < 1. O nmero x
[x] comumente representado por {x} e recebe a denominao parte fracionria de x.
(x + 1) [x + 1] = x [x]. Basta demonstrar que [x + 1] [x] = 1 = (x + 1) x. Com efeito,
sendo m = [x + 1] e k = [x]. Ento, por definio, m e k so os inteiros que satisfazem:
(x + 1) 1 < m x + 1 e x 1 < k x, do que segue x < m x + 1 e x k < 1 x. Somando:
0 < m k < 2. obvio que m k inteiro. Logo, m k = 1.
Defina-se, agora, a funo f: R R, por f (x) = x [x]. De acordo com a ltima propriedade
demonstrada acima, tem-se f (x + 1) = f (x), x R. Logo, f peridica e um perodo de f 1. Resta
indagar se o perodo de f, isto , se 1 representa o perodo fundamental de f. A resposta sim. Em
verdade, supondo que houvesse um perodo T0, tal que 0 < T0 < 1, ento: f (x + T0) = f (x), x R. Mas
j que 1 perodo de f: f [(x + T0) + 1] = f [(x + 1) + T0] = f (x), x R . Da: [(x + 1) + T0] = [x] (*).
Basta demonstrar que, se dois nmeros tm mesma parte inteira, ento eles diferem por um nmero
menor que 1. De fato: [a] = [b] = k a 1 < k a; b 1 < k b. Sem perda de generalidade, supondo b
a k < 1 a
a, vem que:
b a 1 < 0 < 1 + b a . Somando a b a todos os membros das
b 1 < k b
desigualdades: 1 < a b < 1. De b a, tem-se 0 a b < 1. Portanto, de (*), conclui-se que:
(x + 1) + T0 x < 1 T0 < 0, o que um absurdo.
y

2 1

A propsito, tal funo tambm conhecida como dente de serra, denominao facilmente
justificvel pelo grfico, ilustrado acima.
6. Como saber se a funo real de varivel real de lei f (x) = cos x (definida para todo real no negativo
x) peridica ou no? A idia , como j foi feito precedentemente, supor inicialmente que f seja
peridica com perodo T. Caso isto ocorresse:

95

Captulo 2. Funes

f (x + T) = f (x) cos (x + T ) = cos x . Lembrando que os ngulos e tm o mesmo co-seno se, e


somente se, = + 2k, em que k um nmero inteiro, dever-se-ia impor que:
x + T = x + 2k . Quadrando:
x + T = x + 4k22 2k x , de que:
T = 4k22 2k x , o que no convm (T no pode depender de x).
Por conseguinte, f no peridica.

2.12.2. INTERPRETAO GRFICA


Graficamente, uma funo peridica de perodo T se, e somente se, seu grfico pode ser
inteiramente obtido atravs de translaes horizontais de um trecho prprio qualquer, de comprimento T.
Noutros termos: fixando uma parte ininterrupta (no interrompida) qualquer do grfico de f, com
comprimento (projeo do grfico sobre o eixo x) T, todo o resto do grfico nada mais do que uma
mera cpia de tal parte, podendo ser traado a partir de translaes exclusivamente horizontais do
referido trecho.
Isto uma conseqncia direta da definio. De fato, um ponto (x, f(x)) pertence ao grfico de
uma funo peridica de perodo T se, e somente se, o ponto (x + T, f(x + T)) = (x + T, f(x)) tambm
est em G(f). s notar, ento, que a distancia entre tais pontos x + T x = T (> 0). Como este
raciocnio vlido para todo x do domnio de f
Para fixar idias, pode-se usar uma metfora bem til. Imagine-se que seja destacada uma parte
ininterrupta de G(f), com comprimento T. Caso se faa um carimbo de tal parte, para obter todo o
grfico de f suficiente carimb-lo, lado a lado, apenas deslizando esse carimbo na horizontal.
y
CARIMBO
x0 T

x0

x0 + T

x
y0
T
As imagens repetem-se de T em T:
y0 = f (x0) = f (x0 + T) = f (x0 T)

2.12.3. ALGUMAS PROPRIEDADES


I. Se f: A B peridica de perodo T, ento qualquer nmero da forma k.T, com k inteiro positivo,
tambm um perodo de f.

DEMONSTRAO
Por hiptese, f (x + T) = f (x), x A. Note-se que, como trivial, a propriedade vlida para
T =1. Suponha-se que ela tambm seja vlida para um inteiro positivo k, genrico, isto , que f (x + kT)
= f (x), x A. Da, como T um perodo de f:
f [(x + kT) + T] = f (x). Mas: f [(x + kT) + T] = f [x + (k + 1)T]. Logo, f [x + (k + 1)T] = f (x), x A,
ou seja, (k + 1)T tambm um perodo de f. Assim, por induo finita, a propriedade verdadeira para
qualquer valor de k inteiro e positivo.

96

Captulo 2. Funes

OBSERVAO: Deve-se notar que, mesmo quando k um inteiro negativo (e mesmo quando k = 0,
como bvio), a propriedade f (x + kT) = f (x), x A, continua vlida. Assim, por exemplo, se T
um perodo de f, pode-se garantir que f (x) = f (x + T) = f (x + 2T) = f (x + 2469T) = f (x T) = f (x
2T) = f (x 11T) = ...
Em verdade, supondo que k seja um inteiro negativo, tem-se que m = k > 0. Desse modo,
aplicando o resultado demonstrado anteriormente no ponto x + kT, tem-se que:
f (x + kT) = f [(x + kT) + mT] (pois mT um perodo de f). Desta feita:
f (x + kT) = f [x + (k + m)T] = f (x), x A.
Finalmente, conveniente observar que esta propriedade formaliza a idia do carimbo exposta
anteriormente: deslocando-se a partir de x0, de T em T, tanto para a direita (x0 + k.T, com k > 0), quanto
para a esquerda (x0 + k.T, com k < 0), obtm-se apenas repeties do que ocorre para x0.
Conforme j foi visto, uma funo constante sempre peridica, possuindo como perodo
qualquer nmero real positivo T. Conseqentemente, no h um perodo fundamental, e os perodos no
guardam relao especial entre si. possvel provar (usando argumentos de Anlise Matemtica, que
no cabem num curso deste nvel) que h trs grandes grupos de funes contnuas reais de varivel real,
mutuamente exclusivos, e que englobam todos os tipos dessas funes:
a) As funes no peridicas.
b) As funes constantes, que no tm um perodo fundamental.
c) As funes que possuem um (nico) perodo fundamental.
Ou seja, como evidente, ou uma funo peridica ou no peridica. O mais interessante (e
no bvio) que uma funo peridica ou possui um perodo fundamental ou constante.
O teorema seguinte uma espcie de recproca do teorema I, aplicvel s funes peridicas no
constantes.
II. Suponha-se que f: A B seja uma funo peridica no constante. Se T0 o perodo fundamental de
f, ento qualquer perodo de f deve ser um mltiplo inteiro de T0. Mais precisamente:
f (x + T0) = f (x + T) = f (x), x A, e 0 < T0 T, T k N*: T = k.T0.

DEMONSTRAO
Suponha-se, por absurdo, que, embora f (x + T0) = f (x + T) = f (x), x A, 0 < T0 T, T e T =
k.T0, no fosse k inteiro. Ento, embora k Z, [k] seria inteiro. Da, 0 < k [k] < 1 (ver item 1.1,
exemplo 5). Pela observao anterior, devido ao fato de que [k].T0 seria tambm um perodo de f,
deveria ocorrer:
f (x + k.T0) = f [(x + k.T0) [k].T0] = f [x + (k [k]).T0] = f(x), x A.
Mas ento o nmero real positivo (k [k]).T0 seria tambm um perodo de f, o que um absurdo,
pois (k [k]).T0 < T0, ou seja, haveria um perodo de f menor que o fundamental.
Portanto, qualquer que seja o perodo T de uma funo peridica no constante de perodo
fundamental T0, deve haver algum inteiro positivo k, de modo que:
T = k.T0.
Dessa forma, por exemplo, qualquer perodo da funo f: R R, de lei f (x) = sen x, deve possuir a
forma 2k, com k natural no nulo. Analogamente, a funo dente de serra no pode possuir um nmero
no inteiro como perodo. De acordo com o teorema em II, caso se deseje confeccionar um carimbo para
reproduzir todo o grfico f (x) = x [x], permitido construir carimbos de comprimentos 1 (o menor de
todos), 2, 7 ou 2005 unidades. Mas impossvel carimbar totalmente esse grfico com carimbos de
comprimento 5/3, 1/2 ou , por exemplo.
Um corolrio do teorema precedente consiste no seguinte resultado: Se T1 e T2 so perodos de uma
mesma funo peridica no constante, ento a razo entre eles um nmero racional.

97

Captulo 2. Funes

De fato, supondo que o perodo fundamental seja T0, existem naturais no nulos, p e q, de tal sorte
T
p.T0 p
que T1 = p.T0 e T2 = q.T0. Portanto: 1 =
= , o qual , obviamente, um racional.
T2 q.T0 q
III. Sejam f e g funes peridicas, reais de varivel real, de perodos fundamentais T1 e T2. Supondo
T
p
bem definidas as operaes funcionais a seguir, sendo 1 = , com p e q inteiros e mdc (p, q) = 1,
T2
q
ento as funes f g, f.g e f/g, caso no sejam constantes, so peridicas, com perodo T = q.T1 =
p.T2.

DEMONSTRAO
Suponha-se que f (x + T1) = f (x), e que g (x + T2) = g (x), para todos os valores de x no domnio
comum a f e a g.
Desse modo, por exemplo, tem-se que: (f.g)(x + T) = f (x + T).g (x + T) = [f (x + q.T1)].[g (x +
p.T2)]. Como p e q so inteiros, pelo teorema I conclui-se que: (f.g)(x + T) = f (x).g(x) = (f.g)(x), ou
seja, T = q.T1 = p.T2 um perodo de f.g..

2.13. ALGUMAS TRANSFORMAES GEOMTRICAS BSICAS


2.13.1. DEFINIES INICIAIS
Assunto de grande relevncia no estudo atual de geometria, as transformaes geomtricas
podem ser estudadas sob dois pontos de vista complementares: puramente geomtrico (sinttico) ou
algbrico (analtico). Nestas poucas linhas, procurar-se- resumir o melhor que os dois mtodos
possuem, atravs de uma aplicao particular de tais transformaes ao estudo das curvas planas, com
mais nfase nos grficos de funes reais de varivel real.
De um modo geral, qualquer transformao geomtrica uma funo que transforma um ponto
(do plano ou do espao) noutro ponto, mediante uma regra bem determinada. Considerem-se X e Y dois
conjuntos de pontos, que podem ser chamados genericamente de FIGURAS. Uma funo : X Y, que
leva o ponto P de X ao ponto P de Y admite, como qualquer funo, o conceito de imagem direta.
Assim, por exemplo, quando se escreve ( PQ ) = P' Q' , deve-se entender que o segmento PQ foi
transformado no segmento P' Q' .
Exemplo:

C = (A)

Homotetia de centro P e
PC PD
de razo
=
= k.
PA PB
B

M = (M) D = (B)

Sejam os segmentos paralelos e no congruentes AB


e CD , com AC BD = {P}. a transformao que
leva os pontos de AB aos de CD , tal que (X) =
PX CD . AB transformado em CD , por .

98

Captulo 2. Funes
3- Seja f: XY uma funo injetiva. Ento, para
cada subconjunto A de X, f(Ac) (f(A))c onde Ac
= {x X/ x A} e (f(A))c = {x Y/ x f(A)}.
Podemos afirmar que est (esto) correta(s):
a) as sentenas no 1 e no 2.
b) as sentenas no 2 e no 3.
c) Apenas a sentena no 1.
d) as sentenas no 1 e no 2.
e) Todas as sentenas.

b) ]0, 1[
c) ]e, e + 1[

e) ]1, +[

291) (ITA-90) Seja f: a funo definida por


x + 2, se x 1

f(x)= x 2 , se 1 < x 1
4, se x > 1

Lembrando que se A ento f- 1(A) = {x


:f(x) A} considere as afirmaes:
I- f no injetora e f 1 ([3 , 5]) = {4}
II- f no sobrejetora e f 1 ([3 , 5]) = f 1 ([2 , 6])
III- f injetora e f 1 ([0 , 4]) = [ 2 , +[
Ento podemos garantir que:
a) Apenas as afirmaes II e III so falsas;
b) As afirmaes I e III so verdadeiras;
c) Apenas a afirmao II verdadeira;
d) Apenas a afirmao III verdadeira;
e) Todas as afirmaes so falsas.

287) (ITA-85) Considere as seguintes funo: f(x)


= x 7/2 e g(x) = x2 1/4 definidas para todo x
real. Ento, a respeito da soluo da inequao
|(gof)(x)| > (gof)(x), podemos afirmar que:
a) Nenhum valor de x real soluo.
b) Se x < 3 ento x soluo.
c) Se x > 7/2 ento x soluo.
d) Se x > 4 ento x soluo.
e) Se 3 < x < 4 ento x soluo.
288) (ITA-86) Consideremos as seguintes
afirmaes sobre uma funo f: .
1. Se existe x tal que f(x) f( x) ento f no
par.
2. Se existe x tal que f( x) = f(x) ento f
impar.
3. Se f par e mpar ento existe x tal que
f(x) = 1.
4. Se f mpar ento fof (f composta com f)
mpar.
Podemos afirmar que esto corretas as afirmaes
de nmeros.
a) 1 e 4
b) 1, 2 e 4
c) 1 e 3
d) 3 e 4
e) 1, 2 e 3

292) (ITA-90) Seja a funo f: {2}


2x - 3
{3} definida por f(x) =
+ 1 . Sobre sua
x-2
inversa podemos garantir que:
a) no est definida pois f no injetora.
b) no est definida pois f no sobrejetora.
y-2
c) est definida por f 1 (y) =
, y 3.
y-3
y+5
1 , y 3.
d) est definida por f 1 (y) =
y-3
2y - 5
e) est definida por f 1 (y) =
, y 3.
y-3

289) (ITA-86) Sejam a, b e c nmeros reais dados


com a < 0. Suponha que x1 e x2 sejam as razes
reais da funo y = ax2 + bx + c e x1 < x2. Sejam

293) (ITA-90) Sejam as funes f e g dadas por:


1 se | x |< 1
f: , f(x) =
0 se | x | 1
2x 3
g: {1} , g(x) =
x 1
Sobre a composta (fog)(x) = f(g(x)) podemos
garantir que:

x3 = b/2a e x 4 = (2b + b 2 4ac ) / 4a . Sobre


o sinal de y podemos afirmar que:
a) y < 0, x , x1 < x < x3
b) y < 0, x , x4 < x < x2
c) y > 0, x , x1 < x < x4
d) y > 0, x , x > x4
e) y < 0, x , x < x3

a) se x

3
2

, f(g(x)) = 0

b) se 1 < x <
c) se

290) (ITA-88) Sejam f e g funes reais de


varivel real definidas por f(x) = ln (x2 x) e
1
g( x ) =
. Ento o domnio de fog :
1 x
a) ]0, e[
d) ] 1, 1[

4
3

160

, f(g(x)) = 1

< x < 2 , f(g(x)) = 1

d) se 1 < x
e) n.d.a

3
2

4
3

, f(g(x)) = 1

Captulo 2. Funes
294) (ITA-91) Considere as afirmaes:
I- Se f: uma funo par e g: uma
funo qualquer, ento a composio gof uma
funo par.
II- Se f: uma funo par e g: uma
funo mpar, ento a composio fog uma
funo par.
III- Se f: uma funo mpar e inversvel
ento f 1: uma funo mpar.
Ento:
a) Apenas a afirmao I falsa;
b) Apenas as afirmaes I e II so falsas;
c) Apenas a afirmao III verdadeira;
d) Todas as afirmaes so falsas;
e) n.d.a.

298) (ITA-94) Dadas as funes reais de varivel


real f(x) = mx + 1 e g(x) = x + m, onde m uma
constante real com 0 < m < 1, considere as
afirmaes:
I- (fog)(x) = (gof)(x), para algum x R.
II- f(m) = g(m)
III- Existe a R tal que (fog)(a) = f(a).
IV- Existe b R tal que (fog)(b) = mb.
V- 0 < (gog)(m) < 3
Podemos concluir
a) Todas so verdadeiras.
b) Apenas quatro so verdadeiras.
c) Apenas trs so verdadeiras.
d) Apenas duas so verdadeiras.
e) Apenas uma verdadeira.

295) (ITA-91) Se A = {x : |x2 + x + 1| |x2 +


2x 3|}, ento temos:

299) (ITA-96) Considere as funes reais f e g


definidas por:
1 + 2x
x
f (x ) =
, x R - { -1, 1} e g( x ) =
,
2
1 + 2x
1 x
x R - { -1/2}. O maior subconjunto de R onde
pode ser definida a composta fog, tal que
(fog)(x) < 0, :
a) ]-1, -1/2[ ]-1/3, -1/4[
b) ]- , -1[ ]-1/3, -1/4[
c) ]- , -1[ ]-1/2, 1[
d) ]1, [
e) ]-1/2, -1/3[

a) A = [ 2 ,

1
]
2

[4 , + [

b) A = [ 1 , 4]
2

c) A = [ 3 , 1]
d) A = ] , 3] [1, + [
e) n.d.a.

296) (ITA-92) Dadas as funes f: e g


:, ambas estritamente decrescentes e
sobrejetoras, considere h = fog. Ento podemos
afirmar que:
a) h estritamente crescente, inversvel e sua
inversa estritamente crescente.
b) h estritamente decrescente, inversvel e sua
inversa estritamente crescente.
c) h estritamente crescente, mas no
necessariamente inversvel.
d) h estritamente crescente, inversvel e sua
inversa estritamente decrescente.
e) nda

300) (ITA-96) Seja f : R R definida por:


3x + 3, x 0
f (x ) = 2
x + 4 x + 3, x > 0
a) f bijetora e (fof )(2 / 3) = f 1 (21) .
b) f bijetora e (fof )(2 / 3) = f 1 (99) .
c) f sobrejetora mas no injetora.
d) f injetora mas no sobrejetora.
e) f bijetora e (fof )(2 / 3) = f 1 (3) .

297) (ITA-93) Seja uma funo no nula,


mpar e peridica de perodo p. Considere as
seguintes afirmaes:
I. f(p) 0
II. f( x) = f(x + p), x R
III. f( x) = f(x p), x R
IV. f(x) = f( x) , x R
Podemos concluir que:
a) I e II so falsas.
b) I e III so falsas.
c) II e III so falsas. d) I e IV so falsas.
e) II e IV so falsas.

301) (ITA-97) Se Q e I representam,


respectivamente, o conjunto dos nmeros
racionais e o conjunto dos nmeros irracionais,
considere as funes . definidas por

0, se x Q
1, se x I

1, se x Q
g (x) =
0, se x I
Seja J a imagem da funo composta og :
. Podemos afirmar que:
a) J =
b) J = Q
c) J = {0}
d) J = {1}
e) J = {0,1}
(x) =

161

Captulo 2. Funes

ax + b
, - c < x < c, ento f(x), para c < x
x+b
< c, constante e igual a:
a) a + b b) a + c c) c d) b e) a

302) (ITA-97) Sejam f ,g : funes tais


que:
g(x) = 1 x
e
(x) + 2 (2 x ) = (x 1)3
para todo x . Ento [g(x)] igual a:
a) (x 1)3 b) (1 x)3 c) x3
d) x e) 2 x

f (x ) =

308) (ITA-02) Os valores de x R, para os quais


a funo real por f (x ) = 5 - | | 2x - 1 | - 6 | est
definida, formam o conjunto.
a) [0, 1]
d) [- 5, 6]
b) [- 5, 6]
e) (- , 0] [1, 6]
c) [- 5, 0] [1, )

303) (ITA-98) Sejam as funes f: e


g:A , tais que f(x) = x2 9 e (fog)(x)
= x - 6,
em seus respectivos domnios. Ento, o domnio
A da funo g :
a) [ 3, +[
b)
c) [ 5 , +[
d) ] , 1[[3 , + [
e) ] , 6 [

309) (ITA-03) Considere uma funo : IR IR


no-constante e tal que (x + y) = (x) (y), x,
y IR.
Das afirmaes:
I - (x) > 0, x IR.
II - (nx) = [(x)]n, x IR, n IN*.
III - par.
(so) verdadeira(s):
a) apenas I e II.
d) todas.
b) apenas II e III.
e) nenhuma.
c) apenas I e III.

304) (ITA-99) Sejam f, g, h: R R funes tais


que a funo composta h o g o f: R R a
funo identidade. Considere as afirmaes:
I - A funo h sobrejetora.
II- Se x0 R tal que f(x0) = 0, ento f(x) 0
para todo x R com x x0.
III- A equao h(x) = 0 tem soluo em R.
Ento:
a) Apenas a afirmao (I) verdadeira.
b) Apenas a afirmao (II) verdadeira.
c) Apenas a afirmao (III) verdadeira.
d) Todas as afirmaes so verdadeiras.
e) Todas as afirmaes so falsas.

310) (ITA-03) Mostre que toda funo : IR \


{0} IR, satisfazendo (xy) = (x) + (y) em
todo seu domnio, par.

305) (ITA-99) Considere as funes f e g


definidas por f(x) = x 2/x, para x 0 e g(x) =
x
, para x 1. O conjunto de todas a s
x +1
solues da inequao
(gof)(x)< g(x) :
a) [1, + [
b) ] , 2[
c) [ 2, 1[
d) ] 1, 1[
e) ] 2, 1[ ]1, + [

311) (ITA-04) Sejam as funes e g definidas

em por (x) = x2 + x e g(x) = - (x2 + x), em


que e so nmeros reais. Considere que estas
funes so tais que:
G

Valor
Ponto
Valor
Ponto
mnimo
de
mximo
de
mnimo
mximo

306) (ITA-01) O conjunto de todos os valores de


m para os quais a funo
f(x) =

-1

x 2 + (2m + 1) x + (m 2 + 2)
est definida e no negativa para todo x real :

d) ,

1
4

b) , +
4

9
4

>0

Ento, a soma de todos os valores de x para os


quais (fog) (x) = 0 igual a:
a) 0 b) 2 c) 4 d) 6 e) 8

x 2 + (2m + 3) x + (m 2 + 3)

1 7
a) ,
4 4

<0

312) ITA-05) Considere os conjuntos S = {0, 2, 4,


6}, T = {1, 3, 5} e U = {0, 1] e as afirmaes:
I {0} S e S U .
II {2} S \ U e S T U = {0, 1}.
III Existe uma funo f : S T injetiva.
IV Nenhuma funo g : T S sobrejetiva.
Ento, (so) verdadeira(s)
a) apenas I.
b) apenas IV.
c) apenas I e IV.
d) apenas II e III.

7
c) 0,
4

1 7

e) ,
4 4

307) (ITA-02) Sejam a, b, c reais no-nulos e


distintos, c > 0. Sendo par a funo dada por

162

Captulo 2. Funes
e) apenas III e IV.
318) (IME-96) Seja f uma funo real tal que x,
1
a : f(x + a) = + f(x) [f(x)]2 . f
2
peridica? Justifique.

313) (ITA-05) Seja D = IR \ {1} e f : D D uma


x +1
funo dada por f ( x ) =
x 1
Considere as afirmaes:
I f injetiva e sobrejetiva.
II f injetiva, mas no sobrejetiva.
1
III f ( x ) + f = 0 , para todo x D, x 0.
x
IV f(x) . f(x) = 1, para todo x D.
Ento, so verdadeiras:
a) apenas I e III.
b) apenas I e IV c) apenas
II e III
d) apenas I, III e IV e) apenas II, III e IV

319) (IME-96) Dados dois trinmios do segundo


grau:
y = ax2 + bx + c
(I)
2
y = ax +bx + c
(II)
Considere, sobre o eixo Ox, os pontos A e B cujas
abscissas so as razes do trinmio (I) e AB os
pontos cujas abscissas so as razes do trinmio
(II). Determine a relao que deve existir entre os
coeficientes a, b, c, a, b, c de modo que AB
divida o segmento AB harmonicamente.

314) (ITA-05) Determine todos os valores reais


de a para os quais a equao (x 1)2 = |x a|
admita exatamente trs solues distintas.

320) (IME-99) Sejam as funes g (x) e h (x)


assim definidas: g(x) = 3x 4 ; h (x) = f (g (x))
= 9x2 6x + 1. Determine a funo f(x) e faa seu
grfico.

315) (IME-90) Seja f uma funo definida nos


inteiros positivos satisfazendo:
f(1) = 1
f(2n) = 2.f(n) + 1
f(f(n)) = 4n + 3
Calcule f(1990).

321) (IME-04) Seja uma funo : - {0} ,


onde representa o conjunto dos nmeros reais,
tal que (a/b) = (a) - (b) para a e b pertencentes
ao domnio de . Demonstre que uma funo
par.

316) (IME-93) Considere uma funo L: +


que satisfaz:
1. L crescente, isto , para quaisquer 0 < x < y
tem-se L(x) < L(y);
2. L(x.y) = L(x) + L(y) para quaisquer x, y > 0.
Mostre que:
a) L(1) = 0;
b) L(1/x) = L(x), para todo x > 0;
c) L(x/y) = L(x) L(y) para quaisquer x, y > 0;
d) L(xn) = nL(x) para todo x > 0 e natural n;
e) L( n x ) = L(x)/n para todo x > 0 e natural n;
f) L(x) < 0 < L(y) sempre que 0 < x < 1 < y.

322) (IME) Sejam q e r funes cujos domnios


o conjunto dos inteiros maiores que zero. Sabe-se
que q(1) = 1, r(1) = 0 e: se r(n) < 2q(n) + 1, ento
r ( n + 1) = r ( n) + 1
;

q ( n + 1) = q ( n)
r ( n + 1) = 0
q ( n + 1) = q ( n) + 1

se r(n) = 2q(n) + 1, ento


Determine q(5) e r(5).

323) A funo f tal que f (2x + 3) = 3x + 2.


Nestas condies, f(3x + 2) igual a:
a) 2x + 3
b) 3x + 2
c) (2x + 3)/2
d) (9x + 1)/2 e) (9x 1)/3

317) (IME-94) Seja f: R R uma funo


quadrtica tal que f(x) = ax2 + bx + c, a 0, x
R. Sabendo que x1 = 1 e x2 = 5 so razes e que
f(1) = -8
Pede-se:
a) Determinar a, b, c
b) Calcular f(0)
c) Verificar se f(x) apresenta mximo ou mnimo,
justificando a resposta
d) As coordenadas do ponto extremo
e) O esboo do grfico

324) Sendo f uma funo real de varivel real tal


que f(x + 3) = 2x + 3 , prove que f(2x + 3) = 4x +
3.
325) Dadas as funes f(x) = 4x + 5 e g(x) = 2x
5k, ocorrer gof(x) = fog(x) se e somente se k for
igual a:
a) 1/3
b) 1/3
c) 0
d) 1
e) 1

163

Captulo 2. Funes
326) Se f(x) = 1 1/x , com x 0, ento
determine o valor de
R = 96.f(2).f(3).f(4). ... .f(14).f(15).f(16).

a) 1

c)1

d) 4

e) 16

335) Assuma que f uma funo real tal que f(x)


= f( x) e f(x + 2) = 2f(x) para todo x. Ento f(5)
igual a:

327) Sejam as funes reais f(x) = 3x 5 e


fog(x) = x2 3. Determinar a lei da funo g.

336) Seja f uma funo real tal que: f(2) = 3 e


f(a + b) = f(a) + f(b) + ab, para todo a e b.
Calcule f(11).

328) Sejam as funes reais g(x) = 3x 2 e


fog(x) = 9x2 3x + 1. Determinar a lei da funo
f.

337) Seja f uma funo definida em N0 = {0, 1, 2,


3, } e com valores em N0, tal que para n, m
N0 e m 9, f(10n + m) = f(n) + 11m e f(0) = 0.
Quantas solues existem para a equao f(x) =
1995?
a) Nenhuma b) 1
c) 2
d) 11
e) infinitas

329) Sejam f e g funes reais definidas por


x 2 + 2 x + 4 se x 1 e g(x) = x 3.
f ( x) =
se x < 1
3x + 4

Obter a lei que define fog.


330) Sejam as funes reais fog e g definidas por
4 x 2 6 x 1 se
( fog )( x) =
se
4x + 3

b) 0

x 1 e g(x) = 2x 3
x <1

x
e a um nmero real. Se
1 x
x0 = a, x1 = f(x0), x2 = f(x1), ..., x1996 = f(x1995), e
x1996 = 1, calcule a.
a) 0
b) 1/1997
c) 1995
d) 1995/1996
e) nda

338) Dados f ( x ) =

Obter a lei que define f.


331) Prove os seguintes teoremas:
a) Se f: AB e g: BC so funes injetoras,
ento a funo composta gof: AC tambm
injetora
b) Se f: AB e g: BC so funes
sobrejetoras, ento a funo composta gof: AC
tambm sobrejetora
c) Se f: AB e g: BA satisfazem gof = IA,
ento f injetora e g sobrejetora
d) Sejam f: AB e g: BA satisfazendo gof =
IA e fog = IB. Ento, f bijetora e g = f 1
e) Se as funes f de A em B e g de B em C so
bijetoras ento (gof) 1 = f 1og 1

339) Seja f uma funo de inteiros no-negativos


para inteiros no-negativos tal que: f(n.m) =
nf(m) + mf(n), f(10) = 19, f(12) = 52 e f(15) =
26. Determine f(8).
a) 12
b) 24
c) 36
d) 48
e) 60
340) Seja fi (x), i = 1, 2, 3, ... definida por
1
f1 =
e fi+1(x) = fi (f1(x)). Ento, f1998 (1998)
1 x
:
a) 0
b) 1998
c) 1/1997
d) 1997/1998 e) nda

332) Seja f: +: x x x . Ento f(+)


igual a:
a) +
b)
c) [ 1/4, +[
d) vazio
e) [ 1/2, + [

341) Assuma que f(1) = 0, e que para todos os


inteiros m e n, f(m + n) = f(m) + f(n) + 3(4mn
1). Determine f(19).

333) Seja f: N N definida por:


n 3 n 1000
f ( n) =
f ( f (n + 6)) n < 1000

1 1
= , ento f(x) :
x 1 x

342) Dado que f

a) (x 1) 1
c) (x + 1)/x

Ento o valor de f (1992) f (1) :


a) 989 b) 992 c) 1988 d) 1991 e)
indeterminado

b) x/(x + 1)
d) 1/x x

343) Suponha que f ( x ) = 1

334) Seja f: funo peridica com perodo


3, tal que
f(x) = x2 para cada 1 < x 2. Ento f( 4) vale:

1
. Determine
(1 x )

f(f(f(...f(3)...))), onde existem 1998 f's na


composio.
164

Captulo 2. Funes

a) 3

b) 3/2

c) 2/3

344) Se f ( x ) =

352) Suponha que o grfico de y = ax2 + bx + c


dado pela figura abaixo. Ento entre as
expresses: ab, ac, b, a + b + c, a b + c
quantas so positivas?

d) 1

3x
e f(g(x)) = x, ento g(x) = ?
3x + 4

a) (3x + 4)/(3x)
c) (4x)/(3 3x)
e) nda

b) (3x)/(3x + 4)
d) (3x + 4)/4

345) Seja f ( x ) =

2x + 3
. Sabe-se que a inversa de
5x 1

f uma funo que pode ser escrita da forma


f 1 ( x ) =

a) 6

a) 1

x+b
. Determine o valor de b + c + d.
cx + d

b) 7

c) 8

d) 9

c) 3

d) 4

e) 5

353) Suponha que f(x) uma funo com domnio


nos nmeros reais e contra-domnio nos nmeros
reais tal que f(x + f(x)) = 4.f(x) e f(1) = 4. Qual
o valor de f(5)?
a) 16 b) 18 c) 20 d) 22 e) 24

e) nda

346) Suponha que f(x + y) = f(x).f(y) para todos


os nmeros reais x e y. Se f(1) = 8, calcule
f(2/3).
a) 1/8 b) 2/3 c) 4 d) 8 e) nda

354) Suponha que f(x) uma funo tal que para


todo nmero real x:
(i) f(x) + f(1 x) = 11;
(ii) f(1 + x) = 3 + f(x).
Ento f(x) + f( x) deve ser igual a:
a) 8 b) 9 c) 10 d) 11 e) 12

347) Uma certa funo satisfaz f(x) + 2.f(6 x) =


x para todos os nmeros reais x. O valor de f(1)
:
a) 3 b) impossvel determinar c) 2
d) 1 e) 9

355) Se f(x) satisfaz 2.f(x) + f(1 x) = x2 para


todo x, ento f(x) =
a) (x2 3x + 1)/2
b) (x2 + 8x 3)/9
2
c) (4x + 3x 2)/6 d) (x2 + 2x 1)/3
e) (x2 + 9x 4)/9

348) Se f(x) uma funo que satisfaz f(2x + 1)


= 2f(x) + 1 para todo x, a se f(0) = 2, ento f(3)
=
a) 5 b) 9 c) 11 d) 13 e) 15
349) Se f(N + 1) = ( 1)N + 1.N 2.f(N) para os
inteiros N 1, e f(1) = f(1989), ento o valor de
f(1) + f(2) + f(3) + ... + f(1988) igual a:
a) 992/3 b) 993/3 c) 996/3
d) 995/3 e) 994/3

356) Seja f: , no identicamente nula, tal


que f(x).f(y) = [f(x + y) + f(x y)]/2, para todos
os nmeros reais x e y:
a) Mostre que f(0) = 1, f(2) = 1, f(3) = 0 e f(4) =
1.
b) Mostre que f(x + 4) = f(x), para todo x real.

350) Se as equaes (1) x2 + ax + b = 0 e (2) x2


+ cx + d = 0 possuem exatamente uma raiz em
comum, e abcd 0, ento a outra raiz da equao
(2) :
a) d.(c a)/(b d)
b) d.(a + c)/(b + d)
c) c.(b + c)/(a + d)
d) (a c)/(b d)
e) c.(a + c)/(b d)
351) Se f ( x ) =

b) 2

357) Seja f uma funo satisfazendo a equao


f(x) + 1999.f(2 x) = (x 1)3. Ento o valor de
f(0) :
358) Sejam a e b nmeros reais e seja f(x) = 1/(ax
+ b). Dado que existem trs nmeros reais
distintos x1, x2, x3 tais que f(x1) = x2, f(x2) = x3 e
f(x3) = x1, prove que a = b2 .

1+ x
, f1(x) = f(f(x)), f2(x) =
1 3x

f(f1(x)), e em geral fn(x) = f(fn 1(x)), ento


f1993(3) =
a) 3 b) 1993 c) 1/2 d) 1/5 e) 2 1993

359) Uma funo real f satisfaz, para todo x,


f ( x + 1) =

165

1 + f (x)
. Demonstre que f peridica.
1 f (x)

Captulo 3. Representao Decimal


6) (PUC/PR-2001) Sejam A e B dois nmeros de dois algarismos cada um e A < B. Sabendo-se que
cada um desses nmeros igual ao triplo do produto de seus algarismos, qual a razo A/B?
a) 3/8 b) 1/2 c) 3/4 d) 5/8 e) 5/7
Soluo:
Seja (ab)10 a forma geral destes dois nmeros. Assim:
10a + b = 3ab 9ab 30a 3b = 0 (3a 1)(3b 10) = 10
Existem duas possibilidades:
3a 1 = 2
i)
a=1eb=5
3b 10 = 5
3a 1 = 5
ii)
a=2eb=4
3b 10 = 2
A 15
A 5
=

= .
Temos ento A = 15 e B = 24
B 24
B 8
7) (PUC/PR-2004) Um nmero A formado por trs algarismos, abc: o algarismo das dezenas a
metade do das unidades, o das centenas o triplo do das unidades. Invertendo-se a ordem dos algarismos
daquele nmero, obtm-se um nmero B, cba, igual ao nmero A diminudo de 396. A soma A + B
800 igual a:
a) 22
b) 24
c) 26
d) 28
e) 30
Soluo:
Segundo o enunciado temos as seguintes relaes entre os dgitos de A: c = 2b e a = 3c.
Portanto: 100a + 10b + c (100c + 10b + a) = 396 99a 99c = 396 a c = 4
2c = 4 c = 2 a = 6 b = 1 A + B 800 = 612 + 216 800 = 28.
8) Determine a soma dos algarismos do nmero (999.......995)2, onde o nmero 999........995 tem 99
dgitos iguais a 9.
Soluo:
x = (10100 5)2 = 10200 10.10100 + 25 = 10200 10101 + 25.
Assim, x um nmero da forma: 9999...9990000...0025, contendo 99 dgitos 9 e 99 dgitos 0.
Portanto: S = 9.(99) + 2 + 5 = 898
9) (OBM-97) O nmero N tem trs algarismos. O produto dos algarismos de N 126 e a soma dos dois
ltimos algarismos de N 11. O algarismo das centenas de N :
A) 2 B) 3 C) 6 D) 7 E) 9
Soluo:
Seja N = (abc)10. Pelo Enunciado temos que: a.b.c = 126 = 2.32.7 e b + c = 11.
A expresso b + c = 9 possui as seguintes possibilidades de resposta:
2 + 9 = 11 3 + 8 = 11 4 + 7 = 11 5 + 6 = 11
Como a.b.c = 2.32.7, ento temos apenas duas possibilidades para os dgitos de N: {2, 9, 7} ou {3, 6, 7}
Assim, para que b + c = 11 temos que duas possibilidades:
i) b = 2 e c = 9 a = 7
ii) b = 9 e c = 2 a = 7
10) (OBM-2000) Um certo nmero N de dois algarismos o quadrado de um nmero natural.
Invertendo-se a ordem dos algarismos desse nmero, obtm-se um nmero mpar. A diferena entre os
dois nmeros o cubo de um nmero natural. Podemos afirmar que a soma dos algarismos de N :
A) 7 B) 10 C) 13 D) 9 E) 11
Soluo:

168

Captulo 3. Representao Decimal


Seja N = 10a + b. O nmero 10b + a (obtido invertendo-se os algarismos de N) mpar, logo a mpar.
Portanto N = 16 ou N = 36, pois 16 e 36 so os nicos quadrados perfeitos de dois dgitos cujo algarismo
das unidades mpar. Mas 61 16 = 45, que no um cubo perfeito, e 63 36 = 27 = 33 .
Ento N = 36 e 3 + 6 = 9.

11) Um nmero inteiro de seis dgitos inicia com 1. Se este dgito movido do extremo esquerdo para o
extremo esquerdo sem mudar a ordem dos outros dgitos, o novo nmero 3 vezes o original. A soma
dos dgitos dos nmeros :
Soluo:
Sejam n = (1abcde)10 e m = (abcde1)10 os nmeros. Assim: n = 105 + (abcde)10 e m = (abcde)10.10 + 1.
Substituindo: m = 10(n 105) + 1 m = 10n 106 + 1.
Como m = 3n 3n = 10n 106 + 1 7n = 106 1 = 999999 n = 142857.
Desta forma, a resposta 27.
12) Prove que o nmero 111...11 222...22 , para todo n natural, um quadrado perfeito.
2 n 1's

n 2 's

Soluo:
Inicialmente notemos que: 11...1 = (99...9)/9 = (102n 1)/9
222...22 = 2(111...11) = 2(999...99)/9 = 2(10n 1)/9
2

2
10 2 n 1 2.10 n 2 10 2 n 2.10 n + 1 10 n 1
= 333...33
Assim: 111...11 222...22 =

=
=

9
9
9
n 3's
3

13) (Olimpada da ndia-98) Existe algum inteiro positivo N tal que o nmero formado pelos ltimos
dois dgitos da soma 1 + 2 + 3 + ... + N 98?
Soluo:
N( N + 1)
Suponha que exista N tal que 1 + 2 + 3 + ... + N = ...98. Assim:
= ...98 N(N + 1) = ...96
2
Como o dgito das unidades de N(N + 1) 6, ento as nicas possibilidades para o algarismo das
unidades de N so 2 ou 7. Seja x o algarismo das dezenas de N. Assim, podemos armar o algoritmo para
multiplicar N e N + 1:
5

... a

...

x ... a

x ...

... 2a 6
... 3a

... 7a + 5 6

...

8a

... 5a 6
... 15a + 5 6
Note que nos dois casos o dgito das dezenas de N(N + 1) sempre mltiplo de 5, no podendo valer 9.
Assim, no existe nenhum inteiro N tal que o nmero formado pelos ltimos dois dgitos da soma 1 + 2
+ 3 + ... + N 98.

14) Mostre que 1 o nico inteiro positivo que igual a soma dos quadrados dos seus dgitos.
Soluo:
a) Como 92 + 92 + 92 + 92 = 324 o nmero deve possuir menos do que 4 dgitos, ou seja, n = (xyz)10
100x + 10y + z = x2 + y2 + z2 (100 x)x + (10 y)y = z(z 1)
Como o valor mximo de z 9, ento o valor mximo de z(z 1) 72. Como 100 x 91, temos que
x = 0 (10 y)y = z(z 1).
Os valores possveis de (10 y)y so: 0, 9, 16, 21, 24, 25
Os valores possveis de z(z 1) so: 0, 2, 6, 12, 20, 30, 42, 56, 72
Assim, (10 y)y = z(z 1) = 0 y = 0 e z = 1.

169

Captulo 3. Representao Decimal


15) (Seletiva Brasileira Olimpada do Cone Sul-96) Prove que toda progresso aritmtica de nmeros
naturais contm dois termos cuja soma dos algarismos so iguais.
Soluo:
Sabemos que toda progresso aritmtica pode ser expressa da forma an = a + nr, a o primeiro termo e
r a razo da PA. Sejam x o nmero de dgitos de a e y o nmero de dgitos de r.
Calculemos agora os termos de ordem 10x e 10x +1: a 10 x = (10 x )r + a e a 10 x +1 = (10 x +1 )r + a .
Desde que a possui x dgitos e os ltimos x dgitos de (10x)r so iguais a zero, ento o nmero
a10 x = (10 x )r + a da forma [ra]10, ou seja, os primeiros y dgitos so os dgitos de r e os ltimos x dgitos
so os dgitos de a.
Da mesma forma, como a possui x dgitos e os ltimos x + 1 dgitos de (10x + 1)r so iguais a zero, ento
o nmero a 10 x +1 = (10 x +1 )r + a da forma [r0a]10, ou seja, os primeiros y dgitos so os dgitos de r,
depois temos um zero e os ltimos x dgitos so os dgitos de a.
Evidentemente, a soma dos dgitos de a 10 x e a 10 x +1 so iguais, uma vez que, com exceo de um dgitos
zero em a 10 x +1 , todos os outros dgitos destes nmeros so todos iguais.
Na verdade, em uma PA de termos naturais existem infinitos termos com a mesma soma dos dgitos,
uma vez que todos os termos da forma a 10 y = (10 y )r + a , com y x (x o nmero de dgitos de a),
possuem a mesma soma dos dgitos.

16) (Olimpada de Maio-98) Ins escolheu quatro dgitos distintos do conjunto {1, 2, 3, 4, 5, 6, 7, 8, 9}.
Formou com eles todos os possveis nmeros de quatro cifras distintas e somou todos esses nmeros de
quatro cifras. O resultado 193314. Encontre os quatro dgitos que Ins escolheu.
Soluo:
Digamos que os nmeros escolhidos so a, b, c e d.
Sabemos que todas as permutaes destes 4 nmeros igual a 4! = 24.
Fixando um nmero por vez em cada posio, temos ento a permutaes dos outros 3, que igual a 3!
= 6, ou seja, existem 6 nmeros iniciando com 1, 6 terminando em b, 6 com c como segundo dgito e
assim por diante.
Deste modo, podemos escrever a soma dos nmeros da seguinte forma:
S = abcd + abdc + acbd + acdb + ... + dcba S = 6(103 + 102 + 10 + 1)(a + b + c + d)
S = 6666(a + b + c + d) = 193314 a + b + c + d = 29
Como o maior valor da soma dos 4 nmeros 30 (9 + 8 + 7 + 6 = 30), devemos subtrair 1 de algum dos
nmeros, sem que seja obtido dois nmeros iguais. Notamos que isto s possvel quando escolhemos 5
no lugar de 6, implicando que os nmeros escolhidos so {5, 7, 8, 9}.
17) (IMO-68) Determine todos os nmeros naturais n cujo produto dos seus dgitos decimais igual a
n2 10n 22.
Soluo:
Suponha que n possui m > 1 dgitos e que seu primeiro dgito (mais significativo) d.
Inicialmente vamos provar que o produto dos dgitos decimais de n (Pn) menor ou igual a n.
Note que Pn mximo quando todos os dgitos de n (exceto o primeiro, que vale d) so iguais a 9
Pn d.9m 1. Por outro lado, n ser mnimo se depois de seu primeiro dgito (que d) existam m 1
zeros n d.10m 1. Assim: n d.10m 1 d.9m 1 Pn, para m > 1.
Deste modo, provamos que para todos os nmeros naturais n com mais de um dgito, o produto dos
dgitos decimais de n sempre menor ou igual a n.
Assim: Pn n n2 10n 22 n n2 11n 22 0 0 n 12.
Entretanto sabemos que Pn > 0 n2 10n 22 > 0 n 12.
Assim, n = 12 a nica possibilidade de soluo.
Conferindo, vemos realmente que P12 = (12)2 (10)(12) 22 = 12.

170

Captulo 3. Representao Decimal


9) Determine a soma de todos os nmeros pares
de 4 dgitos que podem ser escritos usando 0, 1, 2,
3, 4, 5 (e onde os dgitos podem ser repetidos).

Exerccios
1) Achar um inteiro positivo de dois algarismos
que seja igual ao qudruplo da soma dos seus
algarismos.

10)

Sejam

os

inteiros

a = 111
...
11




m 1's

2) Consideramos os nmeros inteiros de 1 a 1000


inclusive. Somemos entre si todos os que tem
todos seus dgitos pares e somemos entre si todos
os que tem todos seus dgitos mpares. Qual soma
maior?

b = 1000
...
005 . Prove que ab + 1 um quadrado



m 1 1's

perfeito. Expresse a raiz quadrada de ab + 1 da


mesma forma como a e b foram expressos.
11) Mostrar que o produto 12345679 x 9 x k,
sendo k 0 um algarismo, kkk.kkk.kkk

3) O inteiro A consiste de 666 nmeros 3, e o


inteiro B possui 666 nmeros 6. Quantos dgitos
aparecem no produto A.B?

12) O produto de um inteiro positivo de trs


algarismos por 7 termina direita por 638.
Determinar esse inteiro.

4) Quando um nmero de dois dgitos somado a


outro nmero de dois dgitos possuindo os mesmo
dgitos em ordem inversa, a soma um quadrado
perfeito. Determine todos estes nmeros de 2
dgitos.

13) Se a = 3.643.712.546.890.623.517 e
b = 179.563.128,
determine o nmero de
algarismos do produto a.b.

5) A soma de um nmero de dois algarismos com


outro que possui os mesmos dgitos, porm na
ordem inversa, 55. Achar o nmero sabendo que
a diferena entre os algarismos das dezenas e das
unidades igual a 3.

14) Achar o menor inteiro cujo produto por 21


um inteiro formado apenas com algarismos 4.
15) Achar o menor inteiro positivo que
multiplicado por 33 d um produto cujos
algarismos so todos 7.

6) A subtrao de um nmero de dois algarismos


com outro que possui os mesmos dgitos, porm
na ordem inversa, 36. Achar o nmero sabendo
que a soma dos quadrados dos seus algarismos
igual 40.

16) O nmero de 4 dgitos 2pqr multiplicado


por 4 e o resultado um nmero de 4 dgitos
rqp2. Pode-se afirmar que p + q = ?
a) 8 b) 7 c) 6 d) 5 e) nda

7) Um nmero de trs algarismos (xyz)10 igual a


cinco vezes o nmero formado apenas pelos
algarismos das unidades e dezenas (yz)10.
Determine este nmero sabendo que a soma dos
seus algarismos 8 (x + y + z = 8) e que a
subtrao entre os algarismos das unidades e das
dezenas 3 (z y = 3).

17) Seja s(n) a soma dos dgitos de n. Por


exemplo, s(197) = 1 + 9 + 7 = 17. Seja s2(n) =
s(s(n)), s3(n) = s(s(s(n))), e assim por diante. Qual
o valor de s1999(1999)?
a) 28 b) 10 c) 1 d) 8 e) nda
18) possvel encontrar um nmero inteiro cujo
produto dos dgitos seja igual a 66?

8) Um nmero a de trs algarismos (xyz)10


somado com outros dois nmeros formados pelos
mesmos algarismos, porm em outra ordem b =
(zxy)10 e c = (yzx)10, e o resultado 999. Sabendo
que o algarismo das unidades igual ao algarismo
das centenas em a (x = z) e que y + z = 7,
determine o nmero a.

19) Mostrar que o produto de dois fatores entre 10


e 20 o dcuplo da soma do primeiro com as
unidades do segundo, mais o produto das
unidades dos dois.
20) Determine o ltimo dgito da seguinte soma:
S = 1! + 2! + 3! + ... + 1995! + 1996!
a) 9 b) 7 c) 5 d) 3 e) 1

171

Captulo 3. Representao Decimal


iii) se deste nmero n subtrairmos o nmero 3816,
obteremos um nmero formado pelos mesmos
algarismos do nmero n, mas na ordem contrria.
Qual esse nmero?

21) (Unifor-99) Seja n a diferena entre o maior


nmero inteiro com 6 algarismos distintos e o
maior nmero inteiro com 5 algarismos distintos.
A soma dos algarismos de n um nmero
a) primo. b) par. c) divisvel por 11.
d) quadrado perfeito. e) mltiplo de 5.

29) (Fuvest-2005) O menor nmero inteiro


positivo que devemos adicionar a 987 para que a
soma seja o quadrado de um nmero inteiro
positivo
a) 37 b) 36 c) 35 d) 34 e) 33

22) (Mackenzie-2000) O nmero de algarismos


do produto 49.513 :
a) 20 b) 22 c) 18 d) 15 e) 17

30) (Unicamp-2000) Um determinado ano da ltima


dcada do sculo XX representado, na base 10,
pelo nmero abba e um outro, da primeira dcada do
sculo XXI, representado, tambm na base 10,
pelo nmero cddc.
a) Escreva esses dois nmeros.
b) A que sculo pertencer o ano representado
pela soma abba + cddc ?

23) (UERJ-2002) Analise a expresso abaixo, na


qual n um nmero natural: N = 10n n.
a) Se n um nmero par, ento N tambm um
nmero par. Justifique esta afirmativa.
b) Determine o valor da soma dos algarismos de
N quando n = 92.
24) (UFU-2000) Desenvolvendo o nmero 1065 92
iremos encontrar todos os algarismos que o compe.
Assim, podemos afirmar que a soma desses
algarismos igual a:
a) 575 b) 573 c) 566 d) 585
25) (Covest-2002)
A tabela ao lado ilustra uma operao
correta de adio, onde as parcelas e a
soma esto expressas no sistema de
numerao decimal e x, y e z so
dgitos entre 0 e 9. Quanto vale x + y +
z?
a) 17 b) 18 c) 19 d) 20 e) 21

31) (ESA-94) Um nmero formado por trs


algarismos, cuja soma 15. O algarismo das
dezenas o triplo do algarismo das unidades e o
algarismo das centenas o sucessor do algarismo
das dezenas. Esse nmero :
a) 276 b) 267 c) 726 d) 762 e) 627

8x3
y87
+
57z
2296

32) (Colgio Naval-80) Um nmero natural de 6


algarismos comea, esquerda, pelo algarismo 1.
Levando-se este algarismo 1, para o ltimo lugar,
direita, conservando a seqncia dos demais
algarismos, o novo nmero o triplo do nmero
primitivo. O nmero primitivo :
a) 100.006
d) maior que 180.000
b) mltiplo de 11
e) divisvel por 5
c) mltiplo de 4

26) (Fuvest-88)
1 a b c
x
3
a b c 4
Acima est representada uma multiplicao, onde os
algarismos a, b e c so desconhecidos. Qual o valor
da soma a + b + c?
a) 5 b) 8 c) 11 d) 14 e) 17

33) (Colgio Naval-96) Os nmeros naturais M e


N so formados por dois algarismos no nulos. Se
os algarismos de M so os mesmos algarismos de
N, na ordem inversa, ento M + N,
necessariamente mltiplo de:
a) 2 b) 3 c) 5 d) 7 e) 11

27) (Fuvest-92) Seja x = 21000. Sabendo que log10 2


aproximadamente igual a 0,30103 pode-se afirmar
que o nmero de algarismos de x :
a) 300 b) 301 c) 302 d) 1030 e) 2000

34) (Colgio Naval-2000) Considere um sistema


de numerao, que usa os algarismos indoarbicos e o valor posicional do algarismo no
numeral, mas numera as ordens da esquerda para
a direita . Por exemplo: no nmero 3452 tem-se:
1 Ordem : 3
2 Ordem : 4
3 Ordem : 5

28) (Fuvest-2000) Um nmero inteiro positivo n de


4 algarismos decimais satisfaz s seguintes
condies:
i) a soma dos quadrados dos 1o e 4o algarismos 58;
ii) a soma dos quadrados dos 2o e 3o algarismos 52;
172

Captulo 3. Representao Decimal


4 Ordem : 2
Alm disso, cada 7 unidades de uma ordem forma
1 unidade da ordem registrada imediatamente
direita.
Com base nesse sistema, coloque (E) quando a
operao for efetuada erradamente e (C) quando
efetuada corretamente. Lendo o resultado final da
esquerda para a direita, encontramos

39) (Gois-97) Determine a soma dos algarismos


do nmero (999.......995)2, onde o nmero
999........995 tem 99 dgitos iguais a 9.

(A) (E) (E) (E)


(C) (C) (E) (C)
(E) (C) (C) (C)

(B) (E) (C) (C)


(D) (C) (C) (E)

41) (Campina Grande-2000) Encontre um nmero


de 3 dgitos tal que a soma dele com o produto de
seus dgitos d 586.

35) (Colgio Naval-2003) Se a, b e c so


algarismos distintos, no sistema de numerao
decimal existe um nico nmero de dois
algarismos (ab) tal que (ab) 2 (ba)2 = (cc)2. O
valor de (a + b + c) igual a:
a) 11 b) 12 c) 13 d) 14 e) 15

42) (Campina Grande-2003) Na equao


(XY).(ZY) = TTT, XY representa um nmero de
algarismos distintos, o mesmo acontecendo com
ZY, enquanto que TTT representa um nmero de
3 algarismos iguais. A soma X + Y + Z + T
igual a:
a) 21 b) 20 c) 22 d) 19 e) 23

40) (Campina Grande-99) Na subtrao a seguir,


A, B e C so algarismos. Quais so os valores de
A, B e C?
8 A31
C 4 0 A
B1 C 6

36) (IME-86) No produto abaixo, o * substitui


algarismos
diferentes
de
3
e
no
necessariamente
iguais.
Determine
o
multiplicando e o multiplicador.

43) (Rio Grande do Norte-95) A soma dos


algarismos de um nmero natural N, de trs
dgitos, 21. Formamos um novo nmero
mudando a posio do algarismo das unidades
com o das dezenas. O novo nmero 45 unidades
maior que N.
Ento, o produto dos algarismos de N :

* * 3 *
x

* * 3
3 * * *
* * * 3 3
* * * *

44) (Rio Grande do Sul-2001) Um nmero inteiro


positivo dito tetra-legal se a soma dos seus
algarismos for mltipla de quatro (4). A
quantidade de nmeros tetra-legais maiores que
zero (0) e menores que cem (100) igual a:
a) 24 b) 21 c) 20 d) 22 e) 25

* * * * * * *

Questes de Olimpadas Nvel Intermedirio


37) (Braslia-86) Determine um nmero de 4
dgitos, sabendo que seus dois primeiros dgitos
so iguais, que seus dois ltimos dgitos tambm
so iguais e que o nmero um quadrado perfeito.

45) (Rio Grande do Norte-2001) Existem quantos


inteiros positivos de dois algarismos tais que a
diferena entre o inteiro e o produto de seus
algarismos seja 12?

38) (Minas Gerais-86) Suponhamos que voc


pedisse a algum escrever dois nmeros
quaisquer, um embaixo do outro e, em seguida,
escrever abaixo a soma destes dois nmeros, e
continuar assim escrevendo a soma dos dois
nmeros imediatamente superiores, at completar
10 linhas. Voc poderia ento adivinhar a soma
dos dez nmeros, olhando rapidamente a coluna e
multiplicando o stimo nmero por 11. Explique
por que isso sempre acontece.

46) (Rio Grande do Sul-2002) Determine todos os


nmeros naturais que possuem trs algarismos
no nulos e distintos e que so iguais a soma de
todos os nmeros de dois dgitos que podem ser
formados a partir de seus algarismos resulta no
prprio nmero.
47) (So Paulo-97) Prove que se o quadrado de
um nmero de dois algarismos, escrito na base 10,
173

Captulo 4. Critrios de Divisibilidade


Divisibilidade por 8: um nmero inteiro divisvel por 8 se o nmero formado por seus trs ltimos
algarismos for divisvel por 8.
Demonstrao:
N = (anan 1...a3a2a1a0) = (anan 1...a3000) + (a2a1a0) = 1000(anan 1...a3) + (a2a1a0)
Como 1000(anan 1...a3) divisvel por 8, N divisvel por 8 se e somente se (a2a1a0) (que o nmero
formado pelos trs ltimos algarismos de N) divisvel por 8.
Divisibilidade por 9: um nmero inteiro divisvel por 9 se a soma dos seus algarismos for divisvel
por 9.
Demonstrao:
N = (anan 1...a2a1a0) = 10nan + 10n 1an 1 + ... + 10a1 + a0
N = (99
...
...

9 + 1)a n + (99

9 + 1)a n 1 + ... + (9 + 1)a 1 + a 0
n

n 1

N = 99
...
...

9a n + 99

9a n 1 + ... + 9a 1 + a n + a n 1 + ... + a 1 + a 0
n

n 1

N = 9[11
...1a n + 11
...1a n 1 + ... + a 1 ] + a n + a n 1 + ... + a 1 + a 0


n

n 1

Como o primeiro termo mltiplo de 9, para que N seja mltiplo de 9 devemos ter que a expresso
an + an 1 + ... + a1 + a0 (que a soma dos dgitos de n) seja mltiplo de 9.

Divisibilidade por 10: um nmero inteiro divisvel por 10 se seu ltimo algarismo for igual a 0.
Demonstrao:
N = (anan 1...a2a1a0) = (anan 1...a2a10) + a0 = 10(anan 1...a2a1) + a0
Como 10(anan 1...a2a1) divisvel por 10, N divisvel por 10 se e somente se a0 (que o algarismo das
unidades de N) divisvel por 10, ou seja, se igual a 0.
Divisibilidade por 11: um nmero inteiro N = anan 1...a2a1a0 divisvel por 11 quando o inteiro
a0 a1 + a2 a3 + a4 a5 + ... + ( 1)nan for divisvel por 11.
Demonstrao:
N = (anan 1...a2a1a0) = a0 + 10a1 + 102a2 + 103a3 + 104a4 + 105a5 + ... + 10nan
N = a0 + (11 1)a1 + (99 + 1)a2 + (1001 1)a3 + (9999 + 1)a4 + (100001 1)a5 + ... + (100...00 1)nan
N = 11[a1 + 9a2 + 91a3 + 999a4 + 9091a5 + ... + ] + a0 a1 + a2 a3 + a4 a5 + ... + ( 1)nan
Como o primeiro termo mltiplo de 11, para que N seja mltiplo de 11 devemos ter que a expresso
a0 a1 + a2 a3 + a4 a5 + ... + ( 1)nan seja mltiplo de 11.
Divisibilidade por 2m ou 5m, m 1: um nmero inteiro de n algarismos divisvel por 2m ou 5m,
n m 1, se o nmero formado por seus ltimos m algarismos for divisvel por 2m ou 5m,
respectivamente.
Demonstrao:
N = (anan 1...a3a2a1a0) = (anan 1...an m00...0) + (an m 1...a1a0) = 10m(anan 1...an m) + (an m 1...a1a0)
N = 2m5m(anan 1...an m) + (an m 1...a1a0)
Como 2m5m(anan 1...an m) divisvel por 2m ou 5m, N divisvel por 2m ou 5m se e somente se
(an m 1...a1a0) (que o nmero formado pelos ltimos m algarismos de N) divisvel por 2m ou 5m,
respectivamente.

178

Captulo 4. Critrios de Divisibilidade


Exemplos:
1) (UFRJ-98) Determine um nmero inteiro cujo produto por 9 seja um nmero natural composto apenas
pelo algarismo 1.
Soluo:
Como um nmero divisvel por 9 possui a soma dos algarismos divisvel por 9, ento o menor nmero
divisvel por 9 formado apenas por 1s 111111111. Logo: 9.x = 11111111 x = 12345679.
3) (Olimpada do Par-2001) Determinar todos os nmeros de quatro dgitos n = 1a7b que so mltiplos
de 15. (a e b so dgitos no necessariamente distintos)
Soluo:
Desde que 15 = 3.5, temos aplicar os critrios de divisibilidade por 3 e 5. Para que n = 1a7b seja
divisvel por 5, b deve ser igual a 0 ou 5:
i) b = 0 n = 1a70
Para que n seja divisvel por 3 temos que a soma dos dgitos de n deve ser divisvel por 3:
1 + a + 7 + 0 = 8 + a = 3k
se k 2 8 + a 6 a 2, que no dgito
se k = 3 8 + a = 9 a = 1 n = 1170
se k = 4 8 + a = 12 a = 4 n = 1470
se k = 5 8 + a = 15 a = 7 n = 1770
se k 6 8 + a 18 a 10, que no dgito
ii) b = 5 n = 1a75
Para que n seja divisvel por 3 temos que a soma dos dgitos de n deve ser divisvel por 3:
1 + a + 7 + 5 = 13 + a = 3k
se k 4 13 + a 12 a 1, que no dgito
se k = 5 13 + a = 15 a = 2 n = 1275
se k = 6 13 + a = 18 a = 5 n = 1575
se k = 7 13 + a = 21 a = 8 n = 1875
se k 8 13 + a 24 a 11, que no dgito
Portanto, todos os nmeros so 1170, 1470, 1770, 1275, 1575, 1875.
3) (PUC/PR-2001) O nmero de trs algarismos abc, menor que 500, tal que a, b e c formam uma
progresso aritmtica e que divisvel por 45, est contido no intervalo:
a) [0, 100)
b) [100, 200)
c) [200, 300)
d) [300, 400)
e) [400, 500)
Soluo:
Se a, b e c formam uma PA ento a = b r e c = b + r. Para que (abc)10 seja mltiplo de 9:
i) a + b + c = 9 b r + b + b + r = 9 b = 3.
Se c = 5 temos a = 1. Se c = 0 temos a = 6.
ii) a + b + c = 18 b r + b + b + r = 18 b = 6.
Se c = 5 temos a = 7. Se c = 0 temos a = 12 (no convm).
Os nmeros que satisfazem o enunciado so 135, 630 e 765. Destes, somente 135 menor que 500.
4) O nmero de 8 algarismos, 1x9y9z55 divisvel por 33. Se x < y < z, quantos h de tais nmeros?
a) Nenhum b) 05 c) 10 d) 45 e) 30
Soluo:
Se 1x9y9z55 divisvel por 33, devemos aplicar a este nmero os critrios de divisibilidade por 3 e 11.
Assim, aplicando o critrio de divisibilidade por 3:
(*)
3k1 = 1 + x + 9 + y + 9 + z + 5 + 5 = 29 + x + y + z x + y + z = 3(k1 10) + 1 = 3k2 + 1
Aplicando agora o critrio de divisibilidade por 11:
11k3 = (5 + z + y + x) (5 + 9 + 9 + 1) = 5 + x + y + z 24 = 11k3
x + y + z = 11(k3 + 1) + 8 = 11k4 + 8
(**)
179

Captulo 4. Critrios de Divisibilidade


Como z > y > x e z 9 ento x + y + z 7 + 8 + 9 = 24.
Os nmeros naturais menores ou iguais a 24 que deixam resto 8 na diviso por 11 (**) so 8 e 19.
Destes dois somente 19 da forma 3k + 1 (*), ou seja, necessariamente devemos ter x + y + z = 19.
Analisemos todas as solues naturais da equao x + y + z = 19 com a restrio x < y < z:
2 + 8 + 9 = 3 + 7 + 9 = 4 + 6 + 9 = 4 + 7 + 8 = 5 + 6 + 8.
Portanto, temos exatamente 5 solues naturais possveis, produzindo os seguintes nmeros que so
divisveis por 33: 12989955, 13979955, 14969955, 14979855, 15969855.
Deste modo, existem 5 nmeros.

5) Determine todos os inteiros positivos N de trs dgitos tais que N e a soma dos seus dgitos seja
divisvel por 11.
Soluo:
Seja N = [abc]. Como N divisvel por 11 ento a b + c = 11.k1 (*)
Pelo enunciado a soma dos algarismos tambm deve ser divisvel por 11: a + b + c = 11.k2 (**)
Subtraindo (**) de (*): 2b = 11(k2 k1) 11 | b
b = 0 uma vez que 0 b 9.
(*) + (**): 2(a + c) = 11(k1 + k2) 11 | a + c
(a, c) = {(2, 9), (3, 8), (4, 7), (5, 6), (6, 5), (7, 4), (8, 3), (9, 2)}
N = {209, 308, 407, 506, 605, 704, 803, 902}
6) (Olimpada da Rssia-80) Todos os nmeros de dois dgitos de 19 80 so escritos em linha reta sem
espaos. obtido o nmero 192021....7980. Este nmero divisvel por 1980?
Soluo:
Fatorando 1980 temos: 1980 = 22.32.5.11
I) Como x = 192021...7980 termina em 80 e 80 divisvel por 4, ento x divisvel por 4.
II) 1 + 9 + 2 + 0 + 2 + 1 + 2 + 2 + ... + 7 + 9 + 8 + 0 =
= 1 + 10(2 + 3 + 4 + 5 + 6 + 7) + 8 + 9 + 6(1 + 2 + 3 + 4 + 5 + 6 + 7 + 8 + 9) =
= 1 + 270 + 17 + 270 = 558
Como 5 + 5 + 8 = 18 = 9.2 ento x divisvel por 9
III) Como x termina em 0, ento x divisvel por 5.
IV) Notamos que os dgitos de ordem mpar so os dgitos das unidades de cada par, ento:
a = 9 + 6(1 + 2 + 3 + 4 + 5 + 6 + 7 + 8 + 9) = 279
Notamos que os dgitos de ordem par so os dgitos das dezenas de cada par, ento:
b = 1 + 10(2 + 3 + 4 + 5 + 6 + 7) + 8 = 279
Como a b = 0, ento x tambm divisvel por 11.
Desta forma 192021...7980 divisvel por 1980.
7) Os inteiros de dois dgitos de 19 a 92 so escritos consecutivamente para formar o inteiro
N = 19202122...909192. Se 3k a maior potncia de 3 que fator de N, ento k =
a) 0 b) 1 c) 2 d) 3 e) mais de 3
Soluo:
Calculemos a soma dos dgitos de N:
S(N) = 1 + 9 + 10(2 + 3 + 4 + 5 + 6 + 7 + 8) + 3.9 + 7(1 + 2 + 3 + 4 + 5 + 6 + 7 + 8 + 9) + 1 + 2
S(N) = 705.
Como 3 | S(N) ento N divisvel por 3. Como 9 no divide S(N) ento 9 no divide N.
Desta forma, k = 1.

180

Captulo 4. Critrios de Divisibilidade


d) vale 9

Exerccios

11) (EPCAr-2001) Seja o nmero m = 488a9b,


onde b o algarismo das unidades e a o
algarismo das centenas. Sabendo-se que m
divisvel por 45, ento a + b igual a
a) 1 b) 7 c) 9 d) 16

1) Determinar os algarismos x e y de modo que


o inteiro:
a) 67xy seja divisvel por 5 e por 11.
b) 5x6y seja divisvel por 3 e por 4.
c) 56x21y seja divisvel por 9 e por 10.
d) 34xx58y seja divisvel por 9 e por 11.
e) 231xy seja divisvel por 5 e por 9.
f) 514xy seja divisvel por 8 e por 9.

12) (EPCAr-2003) Seja um nmero m = 488a9b,


onde b o algarismo das unidades e "a", o
algarismo das centenas. Sabe-se que m divisvel
por 55, ento o menor valor de a + b igual a
a) 2 b) 7 c) 10 d) 13

2) Determinar os algarismos x e y de modo que


o inteiro:
a) x0y seja divisvel por 12.
b) 5x2y seja divisvel por 15.
c) 28x75y seja divisvel por 33.
d) 1234xy seja divisvel por 72.
e) 3262xy seja divisvel por 36.
f) 7x36y5 seja divisvel por 1375.

13) (Colgio Naval-87) O nmero 583ab


divisvel por 9. O valor mximo da soma dos
algarismos a e b, :
a) indeterminado c) 18
e) 2
b) 20
d) 11
14) (Colgio Naval-2001) Seja N = xyzzyx um
nmero natural escrito na base dez, onde x, y e z
so algarismos distintos. Se N1 e N2 so os dois
maiores nmeros divisveis por 3 e 25, obtidos a
partir de N pela substituio de x, y e z, ento N1
+ N2 igual a:
A) 1008800 B) 1108800 C) 1106650
D) 1157000 E) 1209800

3) Determine todos os valores possveis para n =


1a3b sabendo que n divisvel por 12.
4) Demonstrar que um inteiro divisvel por 4 se
e somente se a soma do algarismo das unidades
com o dobro do algarismo das dezenas divisvel
por 4.
5) Demonstrar que um inteiro divisvel por 8 se
e somente se a soma do algarismo das unidades,
mais o dobro do algarismo das dezenas, mais o
qudruplo do algarismo das centenas divisvel
por 8.

15) (Colgio Naval-2003) Justapondo-se os


nmeros naturais conforme a representao
abaixo, onde o sinal * indica o ltimo algarismo,
forma-se um nmero de 1002 algarismos.
12345678910
1112131415161718192021..........*
O resto da diviso do nmero formado por 16
igual a.
a) 2
c) 6
e) 10
b) 4
d) 8

6) Demonstrar que um inteiro divisvel por 6 se


e somente se a soma do algarismo das unidades
com o qudruplo da soma de todos os outros
algarismos divisvel por 6.
7) O inteiro xy243z divisvel por
Determinar os algarismos x, y e z.

e) vale 0

396.
Questes de Olimpadas Nvel Intermedirio
16) (Rio Grande do Norte-97) O nmero 1234
no divisvel por 11, mas um nmero formado
por uma permutao de seus algarismos pode ser
divisvel por 11. Por exemplo, 1243 divisvel
por 11. Qual nmero total de permutaes do
nmero 1234 que divisvel por 11?:
a) 11 b) 1 c) 15 d) 8 e) 10

8) Determinar x, y, z no nmero 33xy49z para


que seja mltiplo de 693.
9) Sejam A e B dois nmeros distintos de sete
dgitos, cada um deles contendo todos os dgitos
de 1 at 7. Prove que A no divisvel por B.
10) (ESA-94) Se o nmero 7x4 divisvel por 18
ento o algarismo x:
a) no existe b) vale 4 c) vale 7

17)
(Cear-99)
Azambuja
escreveu
__4__1__6__3__ no quadro de sua sala de aula.

181

Captulo 4. Critrios de Divisibilidade


25) (Ahsme-92) Os inteiros de dois dgitos de 19
a 92 so escritos consecutivamente para formar o
inteiro N = 19202122...909192.
Se 3k a maior potncia de 3 que fator de N,
ento k =
a) 0 b) 1 c) 2 d) 3 e) mais de 3

Disse para seus colegas que eles dispunham dos


algarismos 9, 8 e 5 para colocar dois deles em
dois espaos vazios, apagar os espaos no
preenchidos e assim obter um nmero de seis
algarismos diferentes. Quais algarismos devem
ser escolhidos e onde coloc-los para formar o
maior nmero possvel que seja divisvel por 6?

26) (Aime-84) Determine o menor inteiro positivo


n tal que todo dgito de 15n 0 ou 8.

18) (Par-2001) Determinar todos os nmeros de


quatro dgitos n = 1a7b que so mltiplos de 15.
(a e b so dgitos no necessariamente distintos)

27) (Mxico) Quantos nmeros mltiplos de 6


menores que 1000 tem a propriedade de que a
soma de suas cifras 21?
(a) 6 (b) 9 (c) 12 (d) 15 (e) 18

19) (OBM-98) O nmero 1234a6 divisvel por


7. O algarismo a vale:
A) 0 B) 2 C) 5 D) 6 E) 8

28) (Mxico) Ao dividir qualquer potncia de 10


por 45, o resto sempre 10. Com base nisto,
descreva um critrio (distinto da divisibilidade
simultnea por 9 e por 5) para que um nmero
anan 1an 2a0 seja divisvel por 45.

20) (OBM-98) Coloque em cada quadradinho, no


desenho a seguir, os algarismos 1, 2, 3, 4 ou 5, de
forma que cada um deles aparea pelo menos uma
vez e que o nmero formado seja o maior possvel
e mltiplo de 9.

29) (Furman University-99) Um nmero


chamado palndromo se lido da esquerda para a
direita igual ao nmero lido da direita para a
esquerda. Qual o maior inteiro k que verdade
afirmar que todos os nmeros palndromos de 4
dgitos so divisveis por k?
a) 8 b) 9 c) 10 d) 11 e) nda

No nmero que voc construiu, o algarismo mais


repetido apareceu:
a) 6 vezes b) 5 vezes c) 4 vezes
d) 3 vezes e) 2 vezes
21) (OBM-98 Nvel 1) Encontre uma maneira de
se escrever os algarismos de 1 a 9 em seqncia,
de forma que os nmeros determinados por
quaisquer dois algarismos consecutivos sejam
divisveis ou por 7 ou por 13.

30) (University of South Carolina-87) Em base


10, o valor do dgito d para o qual o nmero
d456d seja divisvel por 18 :
a) 0 b) 2 c) 4 d) 6 e) 8
31) (Canad-80) Se a679b um nmero de 5
dgitos (em base 10) que divisvel por 72,
determine a e b.

22) (OBM-2002) N = 539984 um nmero


inteiro positivo com oito algarismos, sendo o
primeiro e o ltimo desconhecidos. Sabendo que
N um mltiplo de 198, encontre o algarismo das
unidades de N / 198.

32) (Sucia-93) O inteiro x tal que a soma dos


dgitos de 3x igual a soma dos dgitos de x.
Prove que x divisvel por 9.

23) (Argentina) Utilizando exclusivamente dois


dgitos distintos, 2 e a, forma-se o seguinte
nmero de 90 cifras: 2a22a222a2222a...
Se este nmero de 90 cifras mltiplo de 9,
determinar todos os valores possveis do dgito a.

33) (Blgica-99) Se os nmeros de 1 a 6 so


escritos em uma ordem qualquer, um nmero
consistindo de 6 dgitos obtido. Qual a
probabilidade que este nmero seja divisvel por
6?
a) 1/6 b) 1/3 c) 1/2 d) 2/3 e) 5/6

24) (Argentina-98) Determinar os dgitos a e b


tais que o nmero de 7 cifras 6a74b14 seja
mltiplo de 9 e de 11. Dar todas as possibilidades.

34) (Hong Kong-90) O nmero de 6 dgitos


a1989b divisvel por 72. Determine a e b.

182

Captulo 4. Critrios de Divisibilidade


35) (Hong Kong-98) Um inteiro positivo N
composto somente dos dgitos 0 e 1, e divisvel
por 2475. Determine o menor nmero possvel de
dgitos de N.

Qual a tecla defeituosa? Qual o nmero que


apareceu na tela?
44) (Irlanda-98) Mostre que nenhum inteiro da
forma xyxy em base 10 (onde x e y so dgitos)
podem ser o cubo de um inteiro.

36) (Leningrado-90) Existe um nmero de 6


dgitos divisvel por 11, cujos dgitos so 1, 2, 3,
4, 5, 6 escritos em alguma ordem sem repeties?

45) (Rssia-62) dado um nmero de 1962


dgitos, que divisvel por 9. Seja x a soma dos
seus dgitos. Seja y a soma dos dgitos de x. Seja z
a soma dos dgitos de y. Calcule z.

37) (Moldvia-98) Sejam A = (a1a2..an-1an)10 e


B = (a1a2..an 1)10 + 4an onde a1, a2, ..., an so os
dgitos do nmero A. Prove que A divisvel por
13 se e somente se B divisvel por 13.

46) (Rssia-98) Existem nmeros de 5 algarismos


M e N onde todos os algarismos de M sejam
pares, todos os algarismos de N sejam mpares,
cada um dos algarismos de 0 a 9 ocorrendo
exatamente uma vez entre M e N e tais que N
divide M?

Questes de Olimpadas Nvel Avanado


38) (Rio de Janeiro-2000) Determine o nico
nmero inteiro N de nove algarismos que satisfaz
s seguintes condies:
(1) seus algarismos so todos distintos e
diferentes de zero.
(2) para todo inteiro positivo n = 2, 3, 4, ..., 9, o
nmero formado pelos n primeiros algarismos de
N (da esquerda para a direita) divisvel por n.
39) (Argentina) Com os dgitos 1, 2, 3, 4, 5, 6,
formar um nmero de seis cifras distintas abcdef
tal que o nmero de trs cifras abc seja mltiplo
de 4, o nmero de trs cifras bcd seja mltiplo de
5, o nmero de trs cifras cde seja mltiplo de 3 e
o nmero de trs cifras def seja mltiplo de 11.
40) (Argentina-96) Quantos nmeros de 15
dgitos que utilizam exclusivamente os dgitos 3 e
8 so mltiplos de 11?
41) (Argentina-2002) Achar o menor mltiplo de
84 formado exclusivamente por dgitos 6 e 7.
42) (Manhattan-97) Considere um inteiro positivo
que, quando escrita a sua representao decimal,
somente foram usados dgitos 0 e 1. Suponha que
exatamente 300 1's so usados, e o resto dos
dgitos so 0's. Pode este inteiro ser o quadrado de
outro inteiro?
43) (Maio-2001) Na minha calculadora, uma das
teclas de 1 a 9 est com defeito: ao pression-la
aparece na tela um dgito entre 1 e 9 que no o
correspondente. Quando tentei escrever o nmero
987654321, apareceu na tela um nmero divisvel
por 11 e que deixa resto 3 ao ser dividido por 9.

183

Captulo 5. Propriedades da Divisibilidade


A) 10

B) 11

C) 12

D) 13

79) (Canad-78) Determine todos os pares a e b


de inteiros positivos satisfazendo 2a2 = 3b3.

E) 14

69) (OBM-2002) O resto da diviso por 9 de


1111111111 22222 :
A) 0 B) 1 C) 3 D) 6 E) 8

80) (Inglaterra-71) Mostre que 2n3 + 2n2 + 2n + 1


nunca mltiplo de 3.
81) (Noruega-94) Se a e b so nmeros naturais
(a, b {1, 2, 3, ...}) e a + b + ab = 54, ento a
+ b igual a:
a) 12 b) 14 c) 15 d) 16 e) 17

70) (Argentina) Sejam p = 2.3.5.7.11.13..... o


produto de todos os nmeros primos at 1997 e q
= 3.5.7.9.11.13..... o produto de todos os nmeros
impares at 1997. Achar a penltima cifra da
direita do produto pq.

82) (Blgica-93) Para quantos valores inteiros de


71) (Argentina) Um nmero a de trs cifras raro
se existe um nmero b de duas cifras tal que ao
dividir a por b, o resto igual ao cubo do
quociente. Por exemplo, 100 raro porque ao
dividi-lo por 46, o quociente 2 e o resto 8 = 23.
Quantos nmeros raros de trs cifras existem?

n (com 100 n 200) a frao


irredutvel?
a) 0 b) 25

c) 50

d) 75

n2 3
n2 1

e) 101

83) (Blgica-97) Sejam a, b, c N0. Dividindo a


por b encontramos o quociente q e o resto r, e
dividindo q por c encontramos o quociente q e o
resto r. A diviso de a por bc possui o resto:
a) r
b) r
c) rr
d) br + r
e) nda

72) (Argentina-96) Determinar o maior nmero


natural de 6 dgitos, todos distintos de zero, que
mltiplo do nmero que resulta ao apagar o
primeiro dgito da esquerda.

84) (Blgica-2003) O ltimo dgito da soma 62003


+ 62002 + ... + 6 + 1 igual a:
a) 5 b) 6 c) 7 d) 8 e) 9

73) (Argentina-2001) Determinar o menor


nmero natural que satisfaz as seguintes trs
condies simultaneamente:
i) tem resto 24 na diviso por 57;
ii) tem resto 73 na diviso por 106;
iii) tem resto 126 na diviso por 159.

85) (Rssia-92) Existe algum inteiro positivo de 4


dgitos tal que no possvel fazer nenhuma troca
de qualquer conjunto de 3 dos seus dgitos de
modo a formar um nmero que mltiplo de
1992.

74) (Chile-91) Demonstre que as expresses 2x +


3y, 9x + 5y so ambas divisveis por 17 para os
mesmos valores inteiros de x e y.

86) (OBM-2003) Para quantos inteiros positivos


2004
um inteiro positivo?
m o nmero 2
m 2
a) um
b) dois c) trs
d) quatro e) mais que quatro

75) (Mxico) Encontre todos os inteiros positivos


n tais que n2 + 1 um mltiplo de n + 1.
76) (Wisconsin-94) Determine todos os nmeros
de 3 dgitos que so iguais mdia aritmtica dos
6 nmeros que podem ser obtidos pela
permutao dos dgitos de m de todas as formas
possveis.

Questes de Olimpadas Nvel Avanado


87) (Cear-96) Os lados de um tringulo so
expressos, em cm, por trs inteiros consecutivos e
sua rea, em cm2, dada por um inteiro. Prove
que o menor lado do tringulo mpar.

77) (Wisconsin-98) Prove que nenhuma potncia


de 2 pode ser escrita como soma de dois ou mais
inteiros positivos consecutivos.

88) (Rio Grande do Sul-2002) Dizemos que um


natural n olmpico se nenhum de seus
algarismos zero e a soma deles divide o seu
produto. Por exemplo, 257 olmpico pois 2 + 5
+ 7 = 14 divide 2 x 5 x 7 = 70, mas 89 no
olmpico porque 8 + 9 = 17 no divide 8 x 9 = 72.

78) (Duke-98) Determine todos os inteiros n tais


que n 3 divide n2 + 2.

195

Captulo 5. Propriedades da Divisibilidade


Mostre que para todo inteiro k > 0 existe um
olmpico de k algarismos.

98) (Argentina) Prove que 7 | a2 + b2 somente


quando 7 | a e 7 | b.

89) (OBM-89) Seja k um inteiro positivo tal que


k (k + 1)
um quadrado perfeito. Prove que k e
3
3
(k + 1) so quadrados perfeitos.

99) (Argentina) Consideramos os nmeros


naturais N menores que 10000 que tem o dgito 2
no lugar das dezenas. Quantos destes nmeros N
deixam resto 5 na diviso por 12?

90) (OBM-94) Quantos nmeros n do conjunto


{1, 2, 3, ..., 100} existem de tal forma que o
algarismo das dezenas de n2 seja um nmero
mpar?
a) 10 b) 20 c) 30 d) 40 e) 50

100) (Argentina) Sejam x e d nmeros naturais


tais que o resto de dividir x por d igual a 4 e o
resto de dividir 14x por d 17. Achar o resto de
dividir 210x por d.

91) (OBM-95) Quantos inteiros no-negativos


menores que 61 no podem ser escritos da forma
2a + 2b + ab, onde a, b so inteiros positivos?
a) 09 b) 10 c) 16 d) 26 e) 28

101) (Argentina) Encontrar dois dgitos distintos


entre si A e B tais que o nmero da forma
BABABA seja mltiplo de AAA, de BBB e de
AB, e, entretanto, BA no seja mltiplo de B.

92) (OBM-97) O nmero de valores inteiros de m


para os quais as razes da equao x2 (m + m2)x
+ m3 1 = 0 so inteiras igual a:
a) 0 b) 1 c) 2 d) 3 e) 4

102) (Argentina) Prove que no existe nenhum


inteiro n tal que n2 + 3n + 4 seja divisvel por 49.
103) (Argentina-97) Achar todos os nmeros
naturais n menores que 1000 tais que n2 termina
em 44, ou seja, n2 tem seus dois ltimos dgitos
iguais a 4.

93) (OBM-2002) Determine o maior natural k


para o qual existe um inteiro n tal que 3k divide n3
3n2 + 22.

104) (Argentina-98) Determinar o maior natural n


tal que 419 + 498 + 4n um quadrado perfeito.

94) (Seletiva Brasileira Cone Sul-99) Sejam p, q,


r, s inteiros no negativos tais que (p + q)2 + q = (r
+ s)2 + s. Prove que p = r e q = s.

105) (Nmero de Ouro-97) Existem quatro


nmeros naturais consecutivos cujo produto seja
um quadrado perfeito?

95) (Seletiva Brasileira Cone Sul-2002) Uma


progresso aritmtica infinita, formada por
inteiros positivos dois a dois distintos, tal que
um de seus termos um quadrado perfeito. Prove
que tal seqncia contm infinitos termos que so
quadrados perfeitos.

106) (Olimpada Provincial-97) Achar todos os


quadrados perfeitos que tem o primeiro dgito (da
esquerda) igual a 1 e todos os restantes dgitos
iguais a 4.
107) (Rioplatense-98) Prove que se so dados 101
nmeros inteiros positivos quaisquer, possvel
escolher 11 deles cuja soma seja divisvel por 11.

96) (Seletiva Brasileira Cone Sul-2003) Seja T o


conjunto de todos as ternas (a, b, c) de inteiros tais
que 1 a < b < c 2002. Para cada terna (a, b, c)
em T, considere o produto abc. Some todos estes
produtos correspondentes a todas as ternas em T.
Prove que a soma divisvel por 2003.

108) (Rioplatense-2000) Existe um nmero


natural n tal que a soma dos dgitos de n seja
divisvel por 23 e a soma dos dgitos de (n + 1)
tambm seja divisvel por 23? Se a resposta sim,
determine o menor nmero n. Se no, explicar
por que.

97) (Argentina) Colocar nmeros naturais


distintos e maiores que 1 nas casas de maneira
que sempre o nmero de uma casa seja mltiplo
do que est na casa anterior e que a soma dos
cinco nmeros seja 517.

109) (Rioplatense-2002) Encontre todos os


nmeros de dois algarismos que so mltiplos da
soma de seus algarismos.
196

Captulo 5. Propriedades da Divisibilidade


121) (Espanha-85) Seja n um nmero natural.
Prove que a expresso (n + 1)(n + 2)(2n
1)(2n) divisvel por 2n.

110) (Chile-95) Existem dois inteiros positivos a e


b tais que sua soma seja 1995 e que seu produto
seja um mltiplo de 1995?

122) (Inglaterra-2001) Determine todos os


nmeros naturais N de dois algarismos para os
quais a soma dos algarismos de 10N N
divisvel por 170.

111) (Chile-97) Para cada nmero inteiro positivo


n, forma-se o nmero Kn = n2 + n + 1. Prove que
nenhum dos nmeros Kn um quadrado perfeito.
112) (Mxico) Encontre todos os nmeros de 4
dgitos com as seguintes propriedades:
a) A primeira e a terceira cifras so iguais;
b) A segunda e a quarta so iguais e o nmero
mais 1 um quadrado perfeito.

123) (Alemanha-96) Determine o conjunto de


todos os inteiros positivos n para os quais n.2n 1
um quadrado perfeito.
124) (Repblicas Tcheca e Eslovaca-99) Mostre
que para todo nmero natural n o produto
2
2
2
2

4 4 4 4 um inteiro.
1
2
3
n

113) (Wisconsin-94) Suponha que n um inteiro


positivo. Determine o menor inteiro positivo x tal
que 2n divide x1995 + 1.

125) (Polnia-94) Determine todos os pares (x, y)

114) (Wisconsin-97) Suponha que a e b so


inteiros tais que a + 2b e b + 2a so quadrados.
Prove que a e b so mltiplos de 3.

de nmeros naturais tais que os nmeros

x +1
y

y +1
so naturais.
x

115) (Wisconsin-2003) Suponha que a2 + b2 + c2


um mltiplo de 16, onde a, b e c so inteiros.
Mostre que a3 + b3 + c3 um mltiplo de 64.

126) (Hong Kong-91) Seja an um nmero de 3n


dgitos iguais a 1: an = 111...11. Mostre que an
divisvel por 3n.

116) (Manhattan-98) John um menino de 10


anos. Ele somente sabe escrever o dgito 1. Prove
que, usando somente o dgito 1, John pode
escrever um mltiplo de 1999. possvel
caracterizar todos os inteiros n para os quais,
usando somente o dgito 1, pode-se escrever um
mltiplo de n?

127) (Hong Kong-97) Determine o nmero de


inteiros positivos a < 100 tal que a3 + 23
divisvel por 24.
128) (ndia-96) Se x e y so inteiros positivos tais
que xy + 1 divisvel por 24, mostre que x + y
divisvel por 24.

117) (Putnam-56) Prove que todo inteiro positivo


possui um mltiplo cuja representao decimal
envolve todos os dez dgitos.

129) (Rssia-67) a) Os dgitos de nmero natural


so rearranjados. Prove que a soma do nmero
dado com o obtido no pode ser igual a 99999
(1967 noves).
b) Os dgitos de um nmero natural so
rearranjados. Prove que se a soma do nmero
dado com o obtido igual a 1010, ento o nmero
dado era divisvel por 10.

118) (Canad-71) Mostre que, para todos os


inteiros n, n2 + 2n + 12 no mltiplo de 121.
119) (Canad-78) Seja n um inteiro. Se o dgito
das dezenas de n2 7, qual o dgito das unidades
de n2?

130) (Rssia-70) Todos os nmeros de 5 dgitos


desde 11111 at 99999 so escritos em cartes.
Estes cartes so postos em linha em uma ordem
arbitrria, resultando em um nmero de 444445
dgitos. Prove que este nmero no uma
potncia de 2.

120) (Canad Preparao IMO-2000) possvel


dividir os nmeros naturais 1, 2, ..., n em dois
grupos disjuntos, tais que os quadrados dos
membros em cada grupo possuem a mesma soma
se (a) n = 40000; (b) n = 40002?

197

Captulo 5. Propriedades da Divisibilidade


131) (Rssia-74) Entre todos os nmeros
representados como 36m 5n (m e n so nmeros
naturais), determine o menor. Prove que ele
realmente o menor.

142) (Torneio das Cidades-95) Prove que 4009


(com uma quantidade arbitrria de zeros) no
um quadrado perfeito.
143) (Sucia-91) Determine todos os inteiros
1 1 1
2
+
= .
positivos m e n tais que
m n mn 5

132) (Rssia-80) Um nmero natural contem seis


dgitos distintos no-nulos e divisvel por 37.
Prove que, rearranjando a ordem dos dgitos,
possvel obter pelo menos mais 23 nmeros que
so divisveis por 37.

144) (Hungria-53) Os inteiros positivos n e d so


tais que d divide 2n2. Prove que n2 + d no um
quadrado perfeito.

133) (Rssia-83) Dados os nmeros naturais n,


m, k. Sabe-se que mn divisvel por nm; e nk
divisvel por kn. Prove que mk divisvel por km.

145) (Seletiva Brasileira Cone Sul-2003)


Encontre o menor inteiro positivo n tal que 32003
um divisor de (n + 1)(n + 2)...(3n).

134) (Rssia-97) Os nmeros de 1 a 37 so


escritos em uma linha de modo que cada nmero
divide a soma dos nmeros anteriores. Se o
primeiro nmero 37 e o segundo 1, qual o
terceiro nmero?

146) (Vietn-74) Determine todos os inteiros n e


b, com 0 < b < 10, tais que se an um inteiro
positivo com n dgitos, todos iguais a 1, ento
a2n b.an um quadrado perfeito.

135) (International Talent Search) Note que se ao


produto de dois membros distintos de {1, 16, 27}
acrescido 9, o resultado o quadrado perfeito de
um inteiro. Determine o nico inteiro positivo n
para o qual n + 9, 16n + 9 e 27n + 9 so
tambm quadrados perfeitos.

147) (Vietn-76) Determine todas as solues


inteiras positivas do sistema mm + n = n12, nm + n =
m3 .

136) (Bltica-99) Determine todos os inteiros


positivos n com a propriedade que raiz cbica de
n obtida pela remoo dos seus ltimos 3
dgitos.

149) (Inglaterra-2002) Determine todos os


inteiros positivos m e n, onde n mpar, que
1 4 1
+ = .
satisfazem
m n 12

148) (Vietn-80) Determine todas as solues


inteiras positivas de 2a + 2b + 2c = 2336.

137) (Torneio das Cidades-86) Pode 1986 ser


representado como a soma de 6 quadrados
perfeitos?

150) (Rssia-84) Os dgitos x e y satisfazem a


condio: para todo n 1 o nmero xx...x6yy...y4
(n vezes x e n vezes y) um quadrado perfeito.
Determine todos os possveis valores de x e y.

138) (Torneio das Cidades-98) Sabe-se que o


quadrado de um nmero inteiro termina em 09.
Demonstrar que, em tal quadrado, o dgito das
centenas par.

151) (Rssia-77) Chamamos de "fino" o nmero


de 2n dgitos que um quadrado perfeito e que os
dois nmeros representados pelos seus primeiros
n dgitos (primeiro dgito diferente de zero) e
ltimos n dgitos (primeiro dgito pode ser zero
mas todos os dgitos no podem ser nulos) so
tambm quadrados perfeitos.
a) Determine todos os nmeros "finos" de dois e
quatro algarismos.
b) Existe algum nmero "fino" de seis
algarismos?
c) Prove que existe um nmero "fino" de vinte
dgitos.

139) (Cone Sul-banco) Mostre que qualquer


nmero inteiro a soma de 5 cubos.
140) (IMO-76) Determine o maior nmero que
produto de inteiros positivos cuja soma 1976.
141) (IMO-88 banco) Se r o resto quando cada
um dos nmeros 1059, 1417 e 2312 dividido
por d, onde d um inteiro maior que 1, determine
o valor de d r.

198

Captulo 6. Nmeros Primos

NMEROS PRIMOS
6.1. DEFINIO:
Seja p > 1 um inteiro positivo. Diz-se que p um nmero primo (ou apenas primo) se e somente
se p apresenta como seus nicos divisores 1 e p. Se um inteiro positivo n maior que 1 no primo ento
chama-se n de composto.
6.2. PROPRIEDADES:
(1) Se p um primo tal que p | ab, ento p | a ou p | b.
(2) Se p um primo tal que p | a1a2...an, ento existe um ndice k, com 1 k n, tal que p | ak.
(3) Se os inteiros p, q1, q2, ..., qn so todos nmeros primos e se p | q1q2...qn, ento existe um ndice k,
com 1 k n, tal que p = ak.
(4) Todo inteiro composto possui um divisor primo.
Exemplos:
1) Determinar todos os primos p tais que 3p + 1 um quadrado perfeito.
Soluo:
3p + 1 = x2 3p = x2 1 3p = (x + 1)(x 1)
Como p um nmero primo, ento as possibilidades so:
i) x + 1 = 3 e x 1 = p x = 2 e p = 1 impossvel pois 1 no primo
ii) x + 1 = p e x 1 = 3 x = 4 e p = 5
2) O menor nmero primo que divide 311 + 512 :
a) 2 b) 3 c) 5 d) 311 + 512 e) nda
Soluo:
Como 311 e 512 so nmeros mpares, ento a sua soma vai ser um nmero par. Assim sendo, 311 + 512
divisvel por 2, que o menor nmero primo.
3) (OBM-95) Quantos so os nmeros primos p, para os quais p1994 + p1995 um quadrado perfeito?
Soluo:
a) 0 b) 1 c) 22 d) 3 e) 4
Soluo:
Notemos que p1994 + p1995 = p1994(p + 1)
Desde que p1994 = (p997)2 um quadrado perfeito, ento para que p1994 + p1995 seja um quadrado
perfeito basta que p + 1 seja um quadrado perfeito:
p + 1 = x2 p = x2 1 p = (x 1)(x + 1)
Como p primo, a nica possibilidade x = 2 p = 3
4) Determine o menor inteiro positivo k tal que (k + 1) + (k + 2) + ... + (k + 19) seja um quadrado
perfeito.
Soluo:
Calculando (k + 1) + (k + 2) + ... + (k + 19) = 19k + 190 = 19(k + 10).
Desde que 19 primo, para que 19(k + 10) seja quadrado perfeito ento k + 10 deve ser divisvel por
19. O menor valor ocorre quando k + 10 = 19 k = 9.
Assim, para k = 9, temos (k + 1) + (k + 2) + ... + (k + 19) = 192.
No caso geral, k + 10 = 19x2 k = 19x2 10.
5) (Competio Descartes-99) Se p1 e p2 so nmeros primos distintos e A = (p1p2 + 1)4 1, mostre que
A possui ao menos 4 divisores primos distintos.
Soluo:
Seja p3 = p1p2 + 2.
199

Captulo 6. Nmeros Primos


Observe que: A = (p1 p2 + 2p1p2 + 1) 12 = (p1 p2 + 2p1p2 + 1
+ 2p1p2 + 1 + 1)
2 2
A = p1p2(p1p2 + 2)(p1 p2 + 2p1p2 + 2) A = p1p2p3(p1p2p3 + 2)
Suponhamos que p3 | p1. Desde que p3 = p1p2 + 2 teremos p3 | 2, que falso por p3 mpar e > 1.
Assim, p3 no divide p1. Analogamente temos que p3 no divide p2. Temos agora dois casos:
i) p3 composto: p1p2p3 possui ao menos 4 divisores primos A possui ao menos 4 divisores primos.
ii) p3 primo: Seja p4 = p1p2p3 + 2. Novamente teremos que p4 no divide nenhum dos nmeros p1, p2 ou
p3, pois se dividisse ento teramos p4 | 2, que falso uma vez que p4 impar e maior que 1.
Logo p4 contribui com pelo menos mais um divisor primo, implicando que A possua ao menos 4
divisores primos.
2

1)(p12p22

6) (Olimpada da Hungria-1923) Prove que, se os termos de uma progresso aritmtica infinita de


nmeros naturais no so todos iguais, ento no podem ser todos primos.
Soluo:
Seja an = a1 + (n 1)r, onde a1, n e r so todos nmeros naturais, n = 1, 2, 3, ...
Em algum momento teremos n = a1 + 1 = x ax = a1 + (a1 + 1 1)r = a1 + a1r = (r + 1)a1
Se r + 1 2 e a1 2 teremos que ax no primo.
Para que ax seja primo teremos que impor que r = 0 e a1 sendo primo ou a1 = 1 e r + 1 sendo primo.
Entretanto, quando n = 1 an = 1 e 1 no primo.
Portanto, a nica possibilidade para r = 0 e a1 primo.
7) Mostre que o numerador de 1 +

1 2
1995
+ + ... +
divisvel por 1997.
2 3
1996

Soluo:
x 1
1
= 1 , temos:
x
x
p
p
1
1
1
1
1
1
1
1 1 1
= 1 + 1 + 1 + 1 + ... + 1
+1

= 1997 1 + + + + ... +
+

q
1995 1996
q
2
3
4
1995
1996
2 3 4
Pode-se agora agrupar os termos eqidistantes dos extremos, de modo que:

p
1 1
1 1
1
1
1
= 1997 1 +
+
+ +
+ +
+ ... +

q
998 999
1996 2 1995 3 1994
p
1997
1997
1997
1997

= 1997
+
+
+ ... +
q
998.999
1.1996 2.1995 3.1994
1
p
1
1
1
p
m

= 1997 1
+
+
+ ... +
= 1997 , onde n a multiplicao

q
998.999
q
n
1.1996 2.1995 3.1994
de 1 at 1996, sendo que nenhum destes termos divide 1997, pois 1997 um nmero primo. Assim,
temos que p.n = 1997.q.m, e como n no divide 1997, ento 1997 divide p.

Fazendo

8) Seja n um inteiro positivo maior ou igual a 5. Mostre que no mximo 8 membros do conjunto {n + 1,
n + 2, ... , n + 30} podem ser primos.
Soluo:
Observe que todos os membros do conjunto Sn = {n + 1, n + 2, ... , n + 30} podem ser expressos na
forma 30N + i , onde N um inteiro no negativo e i varia de 0 at 29. Note que 30N + i mltiplo de 2
para i = 0, 2, 4, ... , 28; mltiplo de 3 para i = 0, 3, 6, ... , 27; mltiplo de 5 para i = 0, 5, 10, ... , 25.
Ou seja, para n > 5, os nicos valores de i para os quais 30N + i pode ser primo so os inteiros positivos
menores que trinta e tais que mdc (30, i) = 1, que so os valores 1, 7, 11, 13, 17, 19, 23 e 29. Portanto,
para n > 5, existem no mximo 8 primos em Sn .
9) Seja p um primo, p > 3. Provar que se existe um inteiro a tal que p divide (a2 a + 3), ento existe um
inteiro b tal que p divide (b2 b + 25).
Soluo:
200

Captulo 6. Nmeros Primos


2

Portanto, todo quadrado perfeito de quatro dgitos (< 6): n = x 77 x 31.


i) n = abcd = 1000a + 100b + 10c + d
ii) m = (a + 1)(b + 1)(c + 1)(d + 1) = 1000(a + 1) + 100(b + 1) + 10(c + 1) + (d + 1)
m = 1111 + 1000a + 100b + 10c + d m = 1111 + n m n = 1111 y2 x2 = 1111
(y x)(y + x) = (11)(101) y + x = 101 e y x = 11 y = 56 e x = 45
Como n = x2 = 452 n = 2025. Conferindo: n = 2025 m = 3136 = (56)2

5) Determine todos os inteiros n tais que n2 11n + 63 um quadrado perfeito.


Soluo:
Seja n2 11n + 63 = k2 4n2 44n + 252 = 4k2 (2n 11)2 + 131 = (2k)2 (2k)2 (2n
11)2 = 131 (2k + 2n 11)(2k 2n + 11) = 131.
Como 131 primo temo somente duas possibilidades:
i) 2k + 2n 11 = 131 e 2k 2n + 11 = 1 n = 38
ii) 2k + 2n 11 = 1
e 2k 2n + 11 = 131 n = 27
6) (Olimpada de Portugal-2001) O nmero de NOMEKOP o menor nmero tal que seu dobro um
quadrado perfeito, o seu triplo um cubo perfeito e seu quntuplo uma potncia quinta perfeita.
Determine o nmero de NOMEKOP.
Soluo:
Seja n o nmero de NOMEKOP. A decomposio em fatores primos tem que ser da forma n = 2i3j5k,
com i, j, k nmeros inteiros no negativos. Ento n o menor nmero tal que:
(a) 2n = 2i + 13j5k um quadrado perfeito, isto , tal que os expoentes i + 1, j e k so todos pares;
(b) 3n = 2i3j + 15k um cubo perfeito, isto , tal que os expoentes i, j + 1 e k so todos mltiplos de 3;
(c) 5n = 2i3j5k + 1 uma potncia quinta perfeita, isto , tal que os expoentes i, j e k + 1 so todos
mltiplos de 5.
Assim, para que n verifique as condies anteriores:
(i) i tem que o menor mltiplo de 3.5 = 15 tal que i + 1 seja par, ou seja, i = 15;
(ii) j tem que o menor mltiplo de 2.5 = 10 tal que j + 1 seja mltiplo de 3, ou seja, j = 20;
(iii) k tem que o menor mltiplo de 2.3 = 6 tal que k + 1 seja mltiplo de 5, ou seja, k = 24.
Portanto n = 215320524 o nmero de NOMEKOP.
7) (Olimpada da Irlanda-97) Determine (com prova) todos os pares de inteiros (x, y) satisfazendo a
equao: 1 + 1996x + 1998y = xy
Soluo:
Podemos escrever a equao da seguinte forma:
xy 1996x 1998y = 1 (x 1996)(y 1998) 1996.1998 = 1 (x 1996)(x 1998) = 1 +
1996.1998
(x 1996)(x 1998) = 1 + (1997 1)(1997 + 1) = 1 + 19972 1 (x 1996)(x 1998) = 19972
Assim temos as possibilidades:
i) x 1996 = 1997 e y 1998 = 1997 x = 3993 e y = 3995
ii) x 1996 = 1997 e y 1998 = 1997 x = 1 e y = 1
iii) x 1996 = 19972 e y 1998 = 1 x = 19972 1996 e y = 1999
iv) x 1996 = 19972 e y 1998 = 1 x = 1996 19972 e y = 1997
v) x 1996 = 1 e y 1998 = 19972 x = 1997 e y = 19972 + 1998
vi) x 1996 = 1 e y 1998 = 19972 x = 1996 e y = 1998 19972

203

Captulo 6. Nmeros Primos


6.4. DOIS TEOREMAS CLSSICOS SOBRE NMEROS PRIMOS
Teorema: Sejam a 2 e k 2 nmeros inteiros. Se ak 1 primo, ento k tambm primo.
Demonstrao:
Suponhamos que o inteiro ak 1 (k 2) seja primo.
Se o inteiro k fosse composto, ento teramos k = r.s, com r > 1 e s > 1, o que implica que:
ak 1 = ars 1 = (ar)s 1 ou seja: ak 1 = (ar 1)(ar(s 1) + ar(s 2) + ... + ar + 1)
Como r > 1, os dois fatores do segundo membro so ambos maiores que 1, isto , ak 1 um inteiro
composto, o que contraria a hiptese. Logo, k primo.
Teorema: Sejam a > 1 e n > 0 dois nmeros inteiros. Se an + 1 primo, ento n uma potncia de 2.
Demonstrao:
Todo nmero inteiro pode decomposto na forma n = 2b.(2c + 1), onde (2c + 1) conhecida como a parte
mpar do nmero n. Evidentemente, para provar que um nmero inteiro n igual a uma potncia de 2,
temos que provar que a parte mpar de n (que vale 2c + 1) igual a 1.
b

Assim temos que: a n + 1 = a 2 .(2c+1) + 1 = (a 2 ) 2c+1 (1) 2c+1 = X 2c+1 Y 2c+1 , que divisvel por X Y, de
forma que an + 1 no primo se 2c + 1 > 1.
Deste modo, conclumos que 2c + 1 = 1, e que se an + 1 primo ento n uma potncia de 2.

Exemplos:
1) Dado um nmero primo cujos dgitos so todos iguais a 1 (em expanso decimal), prove que o
nmero de dgitos deve ser um nmero primo.
Soluo:
Seja n o nmero primo dado, possuindo os dgitos iguais a 1
Suponhamos, por absurdo, que o nmero de dgitos s, seja um nmero composto s = ab
n = 111...111 9n = 999...999 = 10s 1 n = (10ab 1)/9
Como 10a 1 | 10ab 1 (10a 1)/9 | (10ab 1)/9 (10a 1)/9 | n n no primo,
contrariando o enunciado do exerccio. Portanto s deve ser primo.
2) Seja n um nmero natural consistindo de 1991 uns: n = 1111.....111 . Prove que n no um nmero
1991 1's

primo.
Soluo:
Como 1991 = 11.181, ento: n = 111...11 9n = 999...99 = 101991 1 = 1011.181 1
Assim, 9n = 1011.181 1 = (1011 1)(101980 101969 + 101958 101947 + 101936 ... + 1022 1011 + 1)
Desta forma, 9n divisvel por 1011 1, ou seja, 9n divisvel por 9999999999, implicando que n
divisvel por 1111111111.

3) (IMO-92 banco) Prove que N =

5125 1
um nmero composto.
5 25 1

Soluo:
x 5 1 4
= x + x3 + x2 + x + 1.
x 1
Ento: N = x4 + x3 + x2 + x + 1 = (x2 + 3x + 1)2 5x(x + 1)2 =
= [( x 2 + 3x + 1) 5x ( x + 1)][( x 2 + 3x + 1) + 5x ( x + 1)] .
Como x = 525 temos: N = [(550 + 3.525 + 1) 513(525 + 1)][(550 + 3.525 + 1) + 513(525 + 1)], ou seja, N
a multiplicao de dois inteiros maiores que 1, implicando que N composto.

Inicialmente notemos que fazendo x = 525 temos N =

204

Captulo 6. Nmeros Primos


6.5. A INFINITUDE DOS NMEROS PRIMOS
Teorema (devido Euclides): H um nmero infinito de primos.
Demonstrao:
Suponhamos, por hiptese, que exista um primo pn maior que todos os outros primos:
p1 = 2, p2 = 3, p3 = 5, p4 = 7, ..., e analisemos o nmero inteiro positivo P tal que: P = p1p2p3...pn + 1
Como P > 1, do Teorema Fundamental da Aritmtica pode-se concluir que P possui pelo menos um
divisor primo p. Contudo, p1, p2, p3, ..., pn so os nicos primos existentes, implicando que p deve,
necessariamente, ser igual a um desses n primos. Desta forma: p | P e p | p1p2p3...pn
implicando
que: p | P p1p2p3...pn ou p | 1
O que um absurdo, pois p > 1 e o nico divisor positivo de 1 o prprio 1. Portanto, qualquer que
seja o primo Pn, sempre existe um primo maior que Pn, isto , o conjunto {2, 3, 5, 7, 11, 13, ...} dos
primos infinito.
Exemplo:
1) (Olimpada da Espanha-92) Seja a seqncia 3, 7, 11, 15, ... (progresso aritmtica). Provar que em
tal seqncia existem infinitos nmeros primos.
Resoluo:
PA: {3, 7, 11, 15, ...} an = 3 + 4(n 1) an = 4n 1
Suponhamos, por absurdo, que exista um nmero finito de primos da forma pi = 4n 1.
Seja o nmero N = 4p1p2p3pn 1, onde pi so todos os primos da forma 4n 1.
Notemos que N tambm da forma 4n 1 e mpar.
Fatorando N, temos que os primos que dividem N devem ser da forma 4n 1 e 4n + 1.
Como (4n1 1)(4n2 1) = 4(4n1n2 n1 n2) + 1 = 4k + 1
(4n1 1)(4n2 + 1) = 4(4n1n2 + n1 n2) 1 = 4k 1
(4n1 + 1)(4n2 + 1) = 4(4n1n2 + n1 + n2) + 1 = 4k + 1
Como mdc (N, pi) = 1, ento cada pi no divide N
Entretanto, na fatorao de N temos que ter fatores primos da forma 4n 1, pois somente multiplicando
um termo da forma 4n1 1 com outro da forma 4n2 + 1 conseguimos um nmero da forma 4k 1, que
a forma de N. Assim, este fator primo de N da forma 4n 1 deve ser distinto dos outros primos pi da
forma 4n 1, que um absurdo.
6.6. DIVISORES PRIMOS DE UM INTEIRO COMPOSTO
Teorema: Se um inteiro positivo a > 1 composto, ento a possui um divisor primo p a .
Demonstrao:
Se o inteiro positivo a > 1 composto, ento existem inteiros b e c, onde 1 < b < a e 1 < c < a, tais que
a = bc. Supondo que b c, temos: b2 bc = a b a
Sendo b > 1, o Teorema Fundamental da Aritmtica afirma que b tem pelo menos um divisor primo p,
de modo que p b a . Como p | b e b | a, implica que p | a, isto , o inteiro primo p a um
divisor de a.
Exemplos:
1) Prove que 1997 um nmero primo.
Soluo:
Observe que 1997 44,687 . Como 1997 no divisvel por nenhum primo que seja menor ou igual a
44 (2, 3, 5, 7, 11, 13, 17, 19, 23, 29, 31, 37, 41, 43) ento podemos afirmar que 1997 um primo.
2) (OBM-98) So dados 15 nmeros naturais maiores que 1 e menores que 1998 tais que dois quaisquer
so primos entre si. Mostre que pelo menos um desses 15 nmeros primo.
Soluo:
205

Captulo 6. Nmeros Primos


Dado 1 < n < 1998, se ele no for primo, ele tem que ter um fator primo menor que 1998 , ou seja, um
fator primo, menor que 45. Como s existem 14 primos menores que 45 (2, 3, 5, 7, 11, 13, 17, 19, 23,
29, 31, 37, 41, 43), e so dados 15 nmeros, ento um desses no ter fator primo menor que 45,
implicando que seja primo.

6.7. CRIVO DE ERATSTENES


Crivo de Erattenes uma representao de nmeros primos, em forma de tabela, de modo que
no excedam um dado inteiro n. Para constru-la deve-se escrever, na ordem natural, todos os inteiros
desde 2 at n e, em seguida, eliminam-se todos os inteiros compostos que so mltiplos dos primos p tais
que p n , isto , 2p, 3p, 4p, ...
Exemplo: Construir a tabela de todos os primos menores que 100
2 3 4 5 6 7 8 9 10
11 12 13 14 15 16 17 18 19 20
21 22 23 24 25 26 27 28 29 30
31 32 33 34 35 36 37 38 39 40
41 42 43 44 45 46 47 48 49 50
51 52 53 54 55 56 57 58 59 60
61 62 63 64 65 66 67 68 69 70
71 72 73 74 75 76 77 78 79 80
81 82 83 84 85 86 87 88 89 90
91 92 93 94 95 96 97 98 99 100
Os inteiros positivos que no foram eliminados so:
2, 3, 5, 7, 11, 13, 17, 19, 23, 29, 31, 37, 41, 43, 47, 53, 59, 61, 67, 71, 73, 79, 83, 89, 97,
que so todos os primos menores que 100.

6.8. PRIMOS GMEOS


Sendo a e b dois inteiros positivos, denomina-se a e b de primos gmeos se os dois so primos,
positivos, mpares e consecutivos. Por exemplo, so pares de primos gmeos: 3 e 5, 5 e 7, 11 e 13, 17
e 19, 29 e 31. At hoje no sabido se existe um nmero infinito de pares de primos gmeos.
Um fato interessante a existncia de apenas um terno de inteiros positivos mpares e consecutivos que
so todos primos: 3, 5 e 7.
Exemplos:
1) (Olimpada do Canad-73) Prove que se p e p + 2 so ambos nmeros inteiros primos maiores que 3,
ento 6 divide p + 1.
Soluo:
Como p e p + 2 so mpares e primos, ento nenhum deles divisvel por 2 ou 3. Desde que p e p + 2
so primos mpares, temos que p + 1 par.
Como p, p + 1, p + 2 so trs nmeros consecutivos, ento um deles divisvel por 3.
Como p e p + 2 no so divisveis por 3, ento p + 1 divisvel por 3, implicando que p + 1
divisvel por 6.
2) Mostrar que, se n > 3, os inteiros n, n + 2 e n + 4 no podem ser todos primos.
Soluo:
Sejam os inteiros consecutivos: n, n + 1, n + 2, n + 3, n + 4, n + 5
Suponhamos que n, n + 2, n + 4 sejam todos primos.
Como n e n + 2 so primos, e entre 3 nmeros inteiros consecutivos sempre um deles divisvel por
3, ento n + 1 divisvel por 3. Se n + 1 divisvel por 3 ento n + 3 divisvel por 3, que um
absurdo, pois n + 4 primo.

206

Captulo 6. Nmeros Primos


6.9. SEQNCIAS DE INTEIROS CONSECUTIVOS COMPOSTOS
Teorema: Para qualquer valor do inteiro positivo n, existem seqncias de n inteiros positivos
consecutivos e compostos.
Demonstrao:
Analisando a seguinte seqncia: (n + 1)! + 2, (n + 1)! + 3, (n + 1)! + 4, ..., (n + 1)! + (n + 1) vemos
que os todos seus n termos so inteiros positivos consecutivos, e tambm cada um deles um nmero
composto, pois (n + 1)! + j sempre divisvel por j se 2 j n + 1. Fazendo n = 4 temos a seguinte
seqncia: 5! + 2, 5! + 3, 5! + 4, 5! + 5, cujos 4 termos so inteiros positivos consecutivos, sendo cada
elemento um nmero composto, pois:
5! + 2 = 122 = 2.61
5! + 3 = 123 = 3.41
5! + 4 = 124 = 4.31
5! + 5 = 125 = 53
Exemplos:
1) (Olimpada da Blgica-90) Defini-se n! = 1.2.3...n. Ento o nmero de primos p tais que:
77! + 1 < p < 77! + 77 dado por:
a) 0 b) 1 c) 7 d) 11 e) 17
Soluo:
O conjunto dos inteiros p tais que 77! + 1 < p < 77! + 77 equivale ao conjunto 77! + 2 p 77! + 76,
ou seja, pn = 77! + n, onde 2 n 76. Notemos que pn sempre divisvel por n, pois como 2 n 76,
na fatorao de 77! certamente existe o fator n. Assim, nenhum pn primo.
2) (IMO-89) Prove que, para cada inteiro positivo n, existem n inteiros positivos consecutivos nenhum
dos quais uma potncia inteira de um nmero primo.
Soluo:
Sabemos que existe uma seqncia de n inteiros positivos compostos:
(n + 1)! + 2, (n + 1)! + 3, (n + 1)! + 4, , (n + 1)! + r, , (n + 1)! + n + 1
Cada um destes nmeros divisvel por r.
Se formarmos a seqncia:
(n + 1)!2 + 2, (n + 1)!2 + 3, (n + 1)!2 + 4, , (n + 1)!2 + r, , (n + 1)!2 + n + 1
Cada um destes nmeros tambm divisvel por r
(n + 1)!2 + r
(n + 1)!
= (n + 1)!
Notemos que:
+1
r
r
(n + 1)!
(n + 1)!
Como (n + 1)!
inteiro, e (n + 1)! divisvel por r, ento r no divide (n + 1)!
+1 ,
r
r
pois se r dividisse este valor, ento r deveria dividir 1, e somente 1 e 1 dividem 1.
Como para cada r podemos escolher um primo p que divide r, ento (n + 1)!2 + r divisvel por p, mas
no por uma potncia de p.
6.10. POSTULADO DE BERTRAND:
Para todo inteiro positivo n, existe um primo p tal que n p 2n.
Exemplo:
1) Prove que o produto de n inteiros positivos consecutivos (n 2) no uma potncia de algum inteiro
(expoente maior ou igual a 2).
Soluo:
Seja P = k(k + 1)(k + 2)...(M 1)M.
Ento, o maior primo p M tem expoente 1 na decomposio do produto em fatores primos de P
(implicando que P no uma potncia de um inteiro) pois, caso contrrio, teramos 2p M e, portanto,
pelo postulado de Bertrand, existiria um primo q tal que p < q < 2p M, contrariamente ao fato de p ser
o maior primo que menor ou igual a M.
207

Captulo 6. Nmeros Primos


14) (UECE-2001) Se a , b e c so dgitos
escolhidos, aleatoriamente, no conjunto {2, 3, 4,
5, 6, 7, 8, 9}, ento o nmero de 6 dgitos abcabc:
a) possui pelo menos 3 fatores primos
b) possui somente 2 fatores primos
c) mltiplo de 3, obrigatoriamente
d) no divisvel por 11

Exerccios
1) Achar as solues inteiras e positivas da
equao x2 y2 = 499.
2) Mostrar que todo inteiro da forma n4 + 4, com
n > 1, composto.

15) (Unifor-99) Trs nmeros primos, a,b,c so


tais que a < b < c e a.b.c = 1001. verdade que
a) a + b = 18 b) a + c = 24 c) b + c = 28
d) c b = b a e) a . b = 55

3) Determine todos os nmeros primos p para os


quais a equao x3 + y3 = p possui soluo nos
inteiros positivos.
4) Mostrar que o nico primo da forma n3 1
7.

16) (Unifor-99) O produto de dois nmeros


naturais mpares e consecutivos 483. Nessas
condies, verdade que o
a) maior deles um quadrado perfeito.
b) menor deles menor que 18.
c) maior deles um nmero primo.
d) menor deles mltiplo de 6.
e) maior deles mltiplo de 7.

5) Mostrar que todo inteiro da forma 8n + 1, com


n 1, composto.
6) Mostrar que, se n2 + 2 primo, ento 3 | n.
7) Prove que se 2n + 1 e 3n + 1 so quadrados
perfeitos, ento 5n + 3 no primo.

17) (Unifor-2000) A soma de todos os nmeros


primos que so divisores de 30! :
a) 129 b) 130 c) 132 d) 139 e) 140

8) Prove que se n 10, n + 10 e n + 60 so


primos, ento n + 90 tambm .
9) Uma velhinha pergunta a um matemtico quais
so os nmeros das linhas de nibus que passam
pelo Instituto de Matemtica. Este responde que
se lembra apenas que os nmeros so de trs
algarismos distintos, cada um dos algarismos
representando um nmero primo. Alm disso, os
nmeros das linhas no so divisveis por 2, por 3
ou por 5. A velha senhora conclui prontamente
que o nmero de linhas de nibus que passam
pelo Instituto :
(a) 4
(b) 3
(c) 2
(d) 1
(e) 0

18) (UFU-98) Se p um nmero natural primo e a


soma de todos os divisores positivos de p2 igual a
31, ento p igual a:
a) 5 b) 7 c) 3 d) 2 e) 11
19) (Fuvest-96) Qual, dos cinco nmeros
relacionados abaixo, no um divisor de 1015?
a) 25 b) 50 c) 64 d) 75 e) 250

10) Prove que o quadrado de todo nmero primo


maior que 3 deixa resto 1 quando dividido por 12.

20) (Fuvest-98) A diferena entre os quadrados de


dois nmeros naturais 21. Um dos possveis
valores da soma dos quadrados desses dois nmeros
:
a) 29 b) 97 c) 132 d) 184 e) 252

11) Determine todos os primos p tais que 17p + 1


um quadrado perfeito.

21) (UFC-2000) Se

p
, onde p e q so
q

1 1
+
3 4
nmeros inteiros positivos relativamente primos,
determine p + q.

12) Prove que todo inteiro positivo igual a


diferena entre dois inteiros positivos compostos,
porm primos entre si.

Questes de Olimpadas Nvel Intermedirio

13) Prove que se 3 nmeros primos, todos


maiores que 3, formam uma progresso
aritmtica, ento a razo da PA divisvel por 6.

22) (So Jos dos Campos-96) Ache, se possvel,


trs nmeros inteiros em progresso aritmtica
cujo produto um nmero primo.
208

Captulo 6. Nmeros Primos


33) (Canad-92) Prove que o produto dos
primeiros n nmeros naturais divisvel pela
soma dos primeiros n nmeros naturais se e
somente se n + 1 no um nmero primo mpar.

23) (OBM-81 banco) Se n > 4 um nmero no


primo, prove que (n 1)! mltiplo de n.
24) (OBM-88) Determine todos os primos que so
soma e diferena de dois primos.

34) (ProMath Competition) Considere a equao


quadrtica x2 + ax + b + 1 = 0. Mostre que se as
razes desta equao so inteiros no nulos, ento
a2 + b2 um nmero composto.

25) (OBM-2000) O nmero 10 pode ser escrito de


duas formas como soma de dois nmeros primos:
10 = 5 + 5 e 10 = 7 + 3. De quantas maneiras
podemos expressar o nmero 25 como uma soma
de dois nmeros primos?
A) 4 B) 1 C) 2 D) 3 E) nenhuma

35) (Blgica-2001) Se x um nmero primo e


x2 + y2 = z2, onde x, y, z N0, ento y =
a) (x2 1)/2 b) (x2 + 1)/2 c) x
d) x2 1
e) x2 + 1

26) (OBM-2001) Quantos nmeros de dois


algarismos no so primos nem mltiplos de 2, 3
ou 5 ?
A) 1 B) 3 C) 2 D) 4 E) mais de 4

36) (University of South Carolina-90) Determine


o nmero de pares ordenados (x, y), com x e y
ambos inteiros, que satisfazem a equao x2 4y2
= 3.
a) 0 b) 2 c) 3 d) 4 e) 6

27) (OBM-2001) No conjunto {101, 1 001, 10


001, ..., 1 000 000 000 001} cada elemento um
nmero formado pelo algarismo 1 nas
extremidades e por algarismos 0 entre eles.
Alguns desses elementos so nmeros primos e
outros so compostos. Sobre a quantidade de
nmeros compostos podemos afirmar que:
A) igual 11
B) igual a 4
C) menor do que 3
D) maior do que 4 e menor do que 11
E) 3

37) (Esccia-2001) Os inteiros positivos p e q so


tais que p, p + q e p + 2q so primos. Prove
que pq mltiplo de 6.
38) (Hungria-1923) Prove que, se os termos de
uma progresso aritmtica infinita de nmeros
naturais no so todos iguais, ento no podem
ser todos primos.
39) (Hungria-1931) Seja p um primo maior que 2.
2
pode expresso em somente uma
Prove que
p
1 1
forma como
+
onde x e y so inteiros
x y
positivos com x > y.

28) (Argentina-95) possvel escrever os 11


nmeros desde 1985 at 1995 em alguma ordem
de modo que o nmero de 44 dgitos obtido seja
um nmero primo?
29) (Manhattan-98) Determine todos os nmeros
primos p para os quais p + 10 e p + 14 so
tambm primos.

40) (Noruega-97) Sejam x e y inteiros positivos.


O menor valor possvel de |11x5 7y3| :
a) 1 b) 2 c) 3 d) 4 e) nenhum destes

30) (University of South Carolina-93) Suponha


que x e y so inteiros tais que y > x > 1 e
y2 x2 = 187. Ento um valor possvel de x.y :
a) 30 b) 36 c) 40 d) 42 e) 54

41) (Torneio das Cidades-2004) Encontre todos os


inteiros positivos n para os quais h n sucessivos
inteiros positivos cuja soma seja um nmero
primo.

31) (British Columbia Colleges-2000) Determine


o menor inteiro positivo k tal que (k + 1) + (k +
2) + ... + (k + 19) seja um quadrado perfeito.

Questes de Olimpadas Nvel Avanado


42) (Mathematical Excalibur) Vinte oito inteiros
so escolhidos no intervalo [104, 208]. Mostre

32) (Alberta Competition-98) Seja S = 1 + 2 + 3 +


... + 10n. Quantos fatores de 2 aparecem da
fatorao de S?
209

Captulo 6. Nmeros Primos


que existem dois deles possuindo um mesmo
divisor primo.

inteiro, ento ambas fraes tem o mesmo


denominador.

43) (Brasil-2002) Mostre que existe um conjunto


A formado por inteiros positivos tendo as
seguintes propriedades:
a) A tem 2002 elementos.
b) A soma de qualquer quantidade de elementos
distintos de A (pelo menos um) nunca uma
potncia perfeita.
Obs: Uma potncia perfeita um nmero da
forma ab, onde a e b so inteiros positivos e b
2.

51) (Manhattan-97) Suponha que p um nmero


1 1
1
primo. Mostre que o nmero: 1 + + + ... +
2 3
p
no um inteiro.
52) (Descartes-99) Se p1 e p2 so nmeros primos
distintos e A = (p1p2 + 1)4 1, mostre que A
possui ao menos 4 divisores primos distintos.
53) (Wisconsin-98) Determine todos os nmeros
primos p para os quais possvel escrever
1
1
1
= 2 + 2 com inteiros positivos a e b.
p a
b

44) (Brasil-2003) Determine o menor nmero


primo positivo que divide x2 + 5x + 23 para
algum inteiro x.

54) (AIME-99) Determine o menor valor de a5, tal


que a1, a2, a3, a4, a5 uma progresso aritmtica
crescente com todos os termos primos.

45) (Brasil Preparao Cone Sul-99) Prove que,


ao expressarmos a soma
1 1 1
1
1
1 + + + + ... +
+
como uma frao
2 3 4
109 110
irredutvel, o numerador um mltiplo de 11.

55) (Putnam-88) Se n > 3 no primo, mostre que


possvel encontrar inteiros positivos a, b, c tais
que n = ab + bc + ca + 1.

46) (Irlanda-2001) Mostre que se um nmero


primo mpar p pode ser colocado sob a forma x5
y5 para alguns inteiros x e y ento
4 p + 1 v2 + 1
=
5
2

56) (Iuguslvia-80) Determine todos os inteiros x


para os quais x2 + 3x + 24 um quadrado
perfeito.

para algum inteiro mpar v.

57) (Espanha-87) Seja C o conjunto dos nmeros


naturais C = {1, 5, 9, 13, 17, 21, ...}. Dizemos que
um nmero primo relativo a C se ele no pode
ser escrito como um produto de nmeros menores
de C.
a) Mostre que 4389 um membro de C que no
pode ser representado em ao menos duas maneiras
distintas como um produto de dois nmeros
primos relativos a C.
b) Determine outro membro de C com a mesma
propriedade.

47) (Brasil Seleo Cone Sul-2002) a) Prove que,


para n > 1 inteiro, 13 + 23 + ... + n3 = (1 + 2 + ... +
n)2.
b) Seja p > 3 primo e k > 1 inteiro. Mostre que
no possvel escrevermos pk como soma dos
cubos de dois ou mais inteiros positivos e
consecutivos.
48) (Argentina-99) Sejam a, b, c, d, e, nmeros
naturais consecutivos tais que a + b + c + d + e
um cubo perfeito e b + c + d um quadrado
perfeito. Achar o mnimo valor possvel de c.
49) (Argentina-99) Seja d = a47 + b47 + c47, com a,
b, c nmeros inteiros tais que a + b + c = 0.
a) Decidir se possvel que d seja igual a 2.
b) Decidir se possvel que d seja um nmero
primo.

58) (Irlanda-2002) Suponha que n seja o produto


de quatro nmeros primos distintos a, b, c, d tais
que :
(i) a + c = d
(ii) a(a + b + c + d) = c (d b)
(iii) 1 + bc + d = bd
Determine n.

50) (Mxico-87) Demonstre que se duas fraes


so irredutveis (simplificadas) e sua soma um

59) (Sucia-77) Seja p um primo. Determine o


maior inteiro d tal que pd divide p4!

210

Captulo 7. MDC e MMC


mltiplo comum entre A e B igual a 3720.
Determinar A e B.

48) (Portugal-95) Os cantores Luciano Pavo,


cido Domingos e Jos Camionetes so muito
supersticiosos: Luciano Pavo s d concertos de
10 em 10 dias, cido Domingos s d concertos
de 6 em 6 dias, enquanto que Jos Camionetes s
d concertos de 11 em 11 dias. Sabendo que no
dia 29 de Fevereiro de 1996 os trs daro um
concerto, em quantos concertos podero os trs
cantar em conjunto no perodo de quatro anos que
se segue? Em que datas se podero realizar tais
concertos?

54) (Argentina-2000) Determinar a quantidade de


pares de nmeros naturais (a, b) que verificam
simultaneamente as seguintes duas condies: o
mximo divisor comum entre a e b igual ao
produto dos 5 primeiros nmeros naturais; o
mnimo mltiplo comum entre a e b igual ao
produto dos 15 primeiros nmeros naturais. Ou
seja, mdc (a, b) = 1.2.3.4.5 e mmc (a, b) =
1.2.3.4.5.6.7.8.9.10.11.12.13.14.15.

49) (Portugal-2002) O Antnio e a Catarina


comearam a trabalhar no mesmo dia. O horrio
do Antnio consiste em 3 dias de trabalho e
depois um dia de descanso, enquanto que a
Catarina trabalha 7 dias seguidos e descansa nos
trs dias seguintes. Quantos dias de descanso
tiveram em comum nos primeiros 1000 dias?

55) (Aime-87) Quantos termos ordenados (a, b, c)


existem de modo que mmc (a, b) = 1000, mmc (b,
c) = 2000 e mmc (c, a) = 2000?
56) (Canad-97) Quantos pares de inteiros
positivos x, y existem, com x y, e tais que mdc
(x, y) = 5! e mmc (x, y) = 50!

50) (Portugal-2002) Luke Skywalker e Han Solo


defrontam-se numa corrida com as suas naves
espaciais mais potentes. Luke d cada volta na
pista em 45 segundos e Han em 48 segundos. As
naves espaciais de Luke e Han s se cruzam no
momento em que Luke Skywalker termina a
corrida. Quantas voltas tm a corrida?

57) (USAMO-72) Os smbolos (a, b, ..., g) e [a, b,


..., g] significam o mximo divisor comum e o
mnimo mltiplo comum, respectivamente, dos
inteiros positivos a, b, ..., g. Prove que:
[a , b, c]2 = (a, b, c) 2
[a , b][a , c][b, c] (a , b)(a , c)(b, c)

51) (Blgica-90) Seja n o nmero de inteiros ( 0)


menores ou iguais a 10.000, que so divisveis por
todos os inteiros positivos menores ou iguais a 10.
Ento:
a) n = 0 b) 1 n 5 c) 5 n 10
d) 10 n 15 e) 15 n

58) (Catalunha) Caracterize todos os inteiros


positivos a e b tais que a + b + mdc (a, b) mmc
(a, b), e determine quando vale a igualdade.
59) (Hungria-98) Para quais inteiros positivos n
existem os inteiros positivos x, y tais que
mmc (x, y) = n! e mdc (x, y) = 1998?

Questes de Olimpadas Nvel Avanado

60) (ndia-98) Determine o menor valor possvel


do mmc de vinte (no necessariamente distintos)
nmeros naturais cuja soma 801.

52) (OBM-99) Um professor de matemtica


passou aos seus alunos a adio

A C
+
onde A,
B D

B, C e D so inteiros positivos, as fraes esto


simplificadas ao mximo e os denominadores so
nmeros primos entre si. Os alunos adicionaram
as fraes tirando o mnimo mltiplo comum dos
denominadores das parcelas e escrevendo este
como o denominador do resultado. Mostre que a
frao que os alunos encontraram como resultado
est simplificada.

61) (Austrlia-91) Seja Mn o mnimo mltiplo


comum dos nmeros 1, 2, 3, ..., n; isto , M1 = 1,
M2 = 2, M3 = 6, M4 = 12, M5 = 60, M6 = 60. Para
quais inteiros positivos Mn 1 = Mn vlido?
62) (Austrlia-2001) Seja L(n) o menor inteiro
positivo divisvel por 2, 3, ... e n. Determine todos
os nmeros primos p e q tais que q = p + 2 e
L(q) > q.L(p).

53) (Argentina-2000) Dos nmeros naturais A e B


sabe-se que B = (A2 1)/8 e que o mnimo

227

Captulo 9. Congruncias

CONGRUNCIAS
Sejam a e b dois inteiros quaisquer e seja m um inteiro positivo fixo. Diz-se que a congruente a
b mdulo m se e somente se m divide a diferena a b. Em outros termos, a congruente a b mdulo m
se e somente se existe um inteiro k tal que a b = km. Com a notao a b (mod. m) indica-se que a
congruente a b mdulo m. Portanto, simbolicamente:
a b (mod. m) m | (a b)
ou seja:
a b (mod. m) k Z / a b = km
8.1. PROPRIEDADES
Seja m um inteiro positivo fixo (m > 0) e sejam a, b, c e d inteiros quaisquer. So vlidas as
seguintes propriedades:
(1) a a (mod. m)
Demonstrao:
Como 0 divisvel por qualquer inteiro m 0, ento: 0 = a a = qm a = qm + a a a (mod. m)
(2) a b (mod. m) b a (mod. m)
Demonstrao:
Se a b (mod. m) a = qm + b

b = ( q)m + a b a (mod. m)

(3) a b (mod. m) e b c (mod. m) a c (mod. m)


Demonstrao:
Se b c (mod. m) b = q2m + c
Se a b (mod. m) a = q1m + b.
Assim, a = m(q1 + q2) + c a c (mod.m)
(4) a b (mod. m) e n | m, com n > 0, a b (mod. n)
Demonstrao:
Se a b (mod. m) a = q1m + b.
Se n | m m = nq2 a = q1q2n + b a b (mod. n)
(5) a b (mod. m) e c > 0, ac bc (mod. mc)
Demonstrao:
Se a b (mod. m) a = qm + b. Multiplicando por c > 0 ac = q(mc) + bc ac bc (mod. mc)
(6) a b (mod. m) e a, b, m so todos divisveis pelo inteiro d > 0 a/d b/d (mod. m/d)
Demonstrao:
Se a b (mod. m) a = qm + b (i). Se d | a, d | b e d | m a/d, b/d e m/d so todos inteiros.
Dividindo (i) por d temos que: (a/d) = q(m/d) + (b/d) a/d b/d (mod. m/d)
(7) a b (mod. m) e c d (mod. m) a + c b + d (mod. m) e ac bd (mod. m)
Demonstrao:
Se a b (mod. m) a = q1m + b (i). Se c d (mod. m) c = q2m + d (ii).
(i) + (ii) a + c = (q1 + q2)m + b + d a + c b + d (mod. m)
(i) x (ii) ac = q1q2m2 + q1dm + q2bm + bd ac = (q1q2m + q1d + q2b)m + bd ac bd (mod. m)
(8) a b (mod. m) a + c b + c (mod. m) e ac bc (mod. m)
Demonstrao:
Se a b (mod. m) a = qm + b (i). Somando c aos 2 lados de (i) a + c = qm + b + c
a + c b + c (mod. m)
Multiplicando (i) por c ac + (qc)m + bc ac bc (mod. m)
(9) a b (mod. m) an bn (mod. m) para todo inteiro positivo n
241

Captulo 9. Congruncias
Demonstrao:
Se a b (mod. m) a = qm + b (i). Elevando a n os dois lados de (i) temos que:
n
n n n p p p 1
n np
n
n
n
n
p
n
n

a = (b + qm) a = b + b (qm)
a = b + m b q m
p
p =1
i =1 p

an = bn + mk an bn (mod. m)
Exemplos:
1) (Mackenzie-2003) Ao ser dividido por 5, o nmero 4758 + 118a x 25847 deixa resto 1. Um possvel
valor do algarismo a, das unidades, :
a) 4 b) 5 c) 6 d) 7 e) 8
Soluo:
Note que: 118a = 1180 + a 118a a (mod. 5)
Como 25847 2 (mod. 5) (118a)(25847) 2a (mod. 5)
Como 4758 3 (mod. 5) 4758 + (118a)(25847) 2a + 3 (mod. 5)
Logo: 2a + 3 1 (mod. 5) 2a 4 (mod. 5) a = 2 ou a = 7.
2) (Colgio Naval-2005) Um nmero natural N deixa: resto 2 quando dividido por 3; resto 3 quando
dividido por 7; e resto 19 quando dividido por 41.
Qual o resto da diviso do nmero k = (N + 1).(N + 4).(N + 22) por 861?
a) 0 b) 13 c) 19 d) 33 e) 43
Soluo:
Pelo enunciado:
i) N 2 (mod. 3) N + 1 0 (mod. 3)
ii) N 3 (mod. 7) N + 4 0 (mod. 7)
iii) N 19 (mod. 41) N + 22 0 (mod. 41)
Logo: (N + 1)(N + 4)(N + 22) 0 (mod. mmc (3.7.41)) (N + 1)(N + 4)(N + 22) 0 (mod. 861)
3) (Colgio Naval-2004) O resto da diviso de 5131 + 7131 + 9131 + 15131 por 12 igual a.
a) 0 b) 2 c) 7 d) 9 e) 11
Soluo:
i) 7 5 (mod. 12) 7131 ( 5)131 (mod. 12) 5131 + 7131 0 (mod. 12) (1)
ii) 15 9 (mod. 12) 15131 ( 9)131 (mod. 12) 9131 + 15131 0 (mod. 12) (2)
Somando as congruncias (1) e (2): 5131 + 7131 + 9131 + 15131 0 (mod. 12)
4) (OBM-98) Qual o dgito das unidades do nmero 31998?
a) 1 b) 3 c) 5 d) 7 e) 9
Soluo:
32 1 (mod. 10) (32)999 ( 1)999 (mod. 10) 31998 1 (mod. 10) 31998 9 (mod. 10)
o dgito das unidades de 31998 9.
5) Demonstrar que 270 + 370 divisvel por 13.
Soluo:
22 + 32 0 (mod. 13) 22 32 (mod. 13) (22)35 ( 32)35 (mod. 13)
270 370 (mod. 13) 270 + 370 0 (mod. 13)
6) Mostrar que 1110 1 (mod. 100)
Soluo:
1110 1 = (11 1)(119 + 118 + 117 + ... + 112 + 11 + 1)
1110 1 = 10.(119 + 118 + 117 + ... + 112 + 11 + 1)
Basta agora provar que (119 + 118 + 117 + ... + 112 + 11 + 1) divisvel por 10.
242

Captulo 9. Congruncias
Como 11 1 (mod. 10) 1 11 11 11 11 ... 11 11 1 (mod. 10)
Somando temos: 119 + 118 + 117 + ... + 112 + 11 + 1 1 + 1 + 1 + ... + 1 (mod. 10)
119 + 118 + 117 + ... + 112 + 11 + 1 10 (mod. 10) 119 + 118 + 117 + ... + 112 + 11 + 1 0 (mod. 10)
2

7) (Olimpada do Rio de Janeiro-98) Mostre que o nmero N = 7601998 201998 + 19101998 6521998
divisvel por 1998.
Soluo:
Notemos inicialmente que 1998 = 2.33.37
i) 760 20 = 740 = 22.5.37 760 20 (mod. 2.37) 7601998 201998 (mod. 2.37)
7601998 201998 0 (mod. 2.37)
ii) 1910 652 = 1258 = 2.17.37 1910 652 (mod. 2.37) 19101998 6521998 (mod. 2.37)
19101998 6521998 0 (mod. 2.37)
Assim: 7601998 201998 + 19101998 6521998 0 (mod. 2.37)
2.37 | 7601998 201998 + 19101998 6521998
iii) 760 652 = 108 = 22.33 760 652 (mod. 33) 7601998 6521998 (mod. 33)
7601998 6521998 0 (mod.33)
iv) 1910 20 = 1890 = 2.33.5.7 1910 20 (mod. 33) 19101998 201998 (mod. 33)
19101998 201998 0 (mod.33)
Assim: 7601998 201998 + 19101998 6521998 0 (mod. 33) 33 | 7601998 201998 + 19101998 6521998
Como 2.37 | 7601998 201998 + 19101998 6521998 e 33 | 7601998 201998 + 19101998 6521998
2.33.37 | 7601998 201998 + 19101998 6521998 1998 | 7601998 201998 + 19101998 6521998
8) (Olimpada da Bulgria-2000) Determine o dgito das centenas de 21999 + 22000 + 22001.
Soluo:
Notemos que 21999 + 22000 + 22001 = 21999(1 + 2 + 4) = 21999.7
210 = 1024 210 24 (mod. 100) 220 76 (mod. 100) (220)99 (76)99 (mod. 100)
21980 76 (mod. 100) (210)(21980) (24)(76) (mod. 100) 21990 24 (mod. 100)
(29)(21990) (512)(24) (mod. 100) 21999 88 (mod. 100) 21999.7 16 (mod. 100)
Desde modo conclumos que os dois ltimos dgitos de 21999.7 so 16. Como 21999.7 divisvel por 8, e
um nmero divisvel por 8 se e somente se o nmero formado pelos seus trs ltimos algarismos
divisvel por 8, os ltimos 3 dgitos de 21999.7 podem ser 216, 416, 616 ou 816, ou seja, o algarismo das
centenas par.
9) (Olimpada do Mxico-87) Demonstre que para qualquer inteiro positivo n, o nmero
(n3 n)(58n + 4 + 34n + 2) mltiplo de 3804.
Soluo:
Inicialmente observemos que 3840 = 22.3.317
Como n3 n = (n 1)n(n + 1), ento 6 | n3 n, faltando provar que 2.317 | 58n + 4 + 34n + 2
Como 58n + 4 + 34n + 2 a soma de dois nmeros mpares, ento 2 | 58n + 4 + 34n + 2
Como 54 + 32 = 634 = 2.317 54 + 32 0 (mod. 317) 54 32 (mod. 317)
(54)2n + 1 ( 32)2n + 1 (mod. 317) 58n + 4 34n + 2 (mod. 317) 58n + 4 + 34n + 2 0 (mod. 317)
10) (Olimpada do Mxico-90) Prove que nn 1 1 divisvel por (n 1)2 para todo inteiro n > 2.
Soluo:
Notemos que nn 1 1 = (n 1)(nn 2 + nn 3 + nn 4 + ... + n2 + n + 1)
Basta provar agora que nn 2 + nn 3 + nn 4 + ... + n2 + n + 1 divisvel por n 1.
Notemos que:
i) n 1 (mod. n 1) n2 1 (mod. n 1) n3 1 (mod. n 1) n4 1 (mod. n 1) ...
nn 3 1 (mod. n 1) nn 2 1 (mod. n 1)
Ento, somando estas congruncias: nn 2 + nn 3 + ... + n + 1 1 + 1 + ... + 1 + 1 (mod. n 1)
nn 2 + nn 3 + nn 4 + ... + n2 + n + 1 n 1 (mod. n 1)
243

Captulo 9. Congruncias
n

n2

+n

n3

+n

n4

+ ... + n + n + 1 0 (mod. n 1)
2

11) (Olimpada da Inglaterra-92) Prove que 3n + 2.17n, onde n um inteiro no nulo, nunca um
quadrado perfeito.
Soluo:
i) 32 1 (mod. 10) 32k ( 1)k (mod. 10) 32k + 1 3( 1)k (mod. 10)
ii) 172 1 (mod. 10) 172k ( 1)k (mod. 10) 172k + 1 17( 1)k (mod. 10)
I) 32k + 2.172k ( 1)k + 2.( 1)k (mod. 10) 32k + 2.172k 3( 1)k (mod. 10)
Como no existe quadrado perfeito que termine em 3 ou 7, ento 32k + 2.172k nunca vai ser um
quadrado
II) 32k + 1 + 2.172k + 1 3( 1)k + 34( 1)k (mod. 10) 32k + 1 + 2.172k + 1 37( 1)k (mod. 10)
32k + 1 + 2.172k + 1 3( 1)k (mod. 10)
Analogamente ao caso anterior, temos que no existe quadrado perfeito da forma 32k + 1 + 2.172k + 1.
12) (IMO-64) (a) Determine todos os nmeros naturais n para os quais 7 divide 2n 1.
(b) Prove que no existe um nmero natural n para o qual 7 divide 2n + 1.
Soluo:
a) Notemos que 23 = 8 = 7 1 23 1 (mod. 7) 23k 1 (mod. 7) 7 | (23k 1)
Assim, se n divisvel por 3, ento 2n 1 divisvel por 7.
II) 23k 1 = 7x 2(23k 1) = 2(7x) (23k + 1 1) 1 = 7y 23k + 1 1 = 7y + 1
se n da forma n = 3k + 1, ento 7 no divide 2n 1.
III) 2(23k + 1 1) = 2(7y + 1) 33k + 2 2 = 14y + 2 33k + 2 1 = 7z + 3.
Logo, se n da forma n = 3k + 2, ento 7 no divide 2n 1.
b) I) 23k 1 = 7x 23k + 1 = 7x + 2
II) 23k + 1 1 = 7y + 1 23k + 1 + 1 = 7y + 3
III) 33k + 2 1 = 7z + 3 33k + 2 + 1 = 7z + 5
Assim, 7 nunca divide 2n + 1.
13) (Olimpada da ustria-Polnia-93) Determine todos os nmeros naturais x, y 1 tais que 2x 3y =
7.
Soluo:
Analisemos a expresso mdulo 3.
i) 2 1 (mod. 3) 2x ( 1)x (mod. 3)
ii) 3 0 (mod. 3) 3y 0 (mod. 3)
Assim, 2x 3y = 7 2x 3y 7 (mod. 3) ( 1)x 1 (mod. 3) x par
Analisemos agora a expresso mdulo 8.
i) 32 1 (mod. 8) 32k 1 (mod. 8) 32k + 1 3 (mod.8)
Como 2x 3y = 7 2x 3y 1 (mod. 8)
Se x = 0 2x 1 (mod. 8) 3y 2 (mod. 8) impossvel
Se x = 2 2x 4 (mod. 8) 3y 5 (mod. 8) impossvel
Se x 3 2x 0 (mod. 8) 3y 1 (mod. 8) y par
Deste modo conclumos que x e y so pares, x 4. Portanto, x = 2n e y = 2m, n e m nmeros naturais.
Assim: 2x 3y = 7 22n 32m = 7 (2n 3m)(2n + 3m) = 7 2n 3m = 1 e 2n + 3m = 7
i) 2.2n = 8 2n + 1 = 8 n = 2 x = 4.
ii) 2.3m = 6 3m = 3 m = 1 y = 2
Assim, x = 4 e y = 2 a nica soluo.
14) (Olimpada da Espanha-2004) Determinar os quatro ltimos dgitos de 32004.
Soluo:
Temos que 32 = 9 = 10 1. Graas a esta expresso, a frmula do binmio de Newton nos permite
simplificar os clculos:
244

Captulo 9. Congruncias
Exerccios

16) Demonstrar que 64 divide 7 2n + 16n 1 para


todo n inteiro positivo.

1) O ltimo dgito de 32001 22001 vale:


a) 1 b) 3 c) 5 d) 7 e) 9

17) Prove que o nmero 55k + 1 + 45k + 2 + 35k


divisvel por 11, para todo nmero natural k.

2) Mostre que 22225555 + 55552222 divisvel por


7.

18) O nmero 3105 + 4105 divisvel por 13, 49,


181 e 379, e no divisvel por 5 ou 11. Como
este resultado pode ser confirmado?

3) Demonstre que 1492n 1770n 1863n + 2141n


divisvel por 1946.

19) Determine todos os membros da seqncia an


= 32n 1 + 2n 1 (n N) que so quadrados
perfeitos de algum inteiro positivo.

4) Demostrar que para todo n natural verifica-se:


32n + 2 + 26n + 1 0 (mod. 11).
5) Demonstrar que An = 5n + 2.3n 1 + 1,
mltiplo de 8 para todo inteiro positivo n.

20) (Colgio Naval-83) O resto da diviso por 11


do resultado da expresso:
121120 + 911932 x 34326,
(A) 9
(B) 1
(C) 10
(D) 6
(E) 7

6) Prove que se n um inteiro no negativo,


ento: 17 | 27n + 1 + 32n + 1 + 510n + 1 + 76n + 1.

21) (Colgio Naval-84) Sendo x2 = 343, y3 = 492 e


z6 = 75, o algarismo das unidades simples do

7) Prove que para todos os inteiros positivos n,


1n + 8n 3n 6n divisvel por 10.

xy
resultado de
z
(A) 1
(B) 3

8) Seja n um inteiro no negativo. Prove que o


nmero formado colocando 2n e 2n + 1 lado a lado
em qualquer ordem um mltiplo de 3.

24

(C) 5

(D) 7

(E) 9

22) (Colgio Naval-95) Sabendo que o resultado


de 12 x 11 x 10 x ... x 3 x 2 x 1 + 14 divisvel
por 13. Qual o resto da diviso do nmero 13 x 12
x .... x 3 x 2 x 1 por 169?
a) 143 b) 149 c) 153 d) 156 e) 162

9) Achar os restos das divises de 250 e 4165 por 7.


10) Demonstrar que, se n > 6 um nmero
perfeito par, ento n congruente a 1 (mod. 6).
11) Calcular o resto da diviso por 8 de 436543 x
793767.

23) (IME-87) Sejam a, b e c nmeros inteiros tais


que 100a + 10b + c seja divisvel por 109. Mostre
que (9a c)2 + 9b2 tambm divisvel por 109.

12) Mostre que:


(i) 22x + 1 + 1 divisvel por 3;
(ii) Se n > 0, 7 no divide 2n + 1;
(iii) 36n 26n divisvel por 35;
(iv) (2903)n (803)n (464)n + (261)n divisvel
por 1897.

Questes de Olimpadas

13) Mostre que

24) (Problem Solving-Strategies) Prove que se m


termina em 5 ento 1991 | 12m + 9m + 8m + 6m.
25) (500 Mathematical Challenges) Quais so os
ltimos 3 dgitos de 79999?

377 1
mpar e composto.
2

26) (Rssia) Determine os dois ltimos dgitos de


14
1414 .

14) Prove que 3636 + 4141 divisvel por 77.

27) (Rssia) Qual o ltimo dgito do nmero:


((((77)7)7)7), onde existem 1000 potncias de
7? Quais so os dois ltimos dgitos?

15) Prove que:


a) 1919 + 6969 divisvel por 44;
b) 270 + 370 divisvel por 13.

28) (Rio Grande do Norte-95) O algarismo das


unidades do nmero 19931995 :
247

Captulo 10. Funo Mximo Inteiro

FUNO MXIMO INTEIRO


Seja x um nmero real. Denota-se por [x] o maior inteiro menor ou igual a x. Esta funo [x]
chamada de funo mximo inteiro ou funo degrau.
Exemplos: [2] = 2, [3,14] = 3, [ 3,14] = 4, [ 3 ] = 2
10.1. PROPRIEDADES:
(1) x = [x] + ; 0 < 1
Demonstrao:
Se [x] o maior inteiro menor ou igual a x ento 0 x [x] < 1.
(2) x 1 < [x] x < [x] + 1, x
Demonstrao:
Se 0 x [x] < 1 ento temos que [x] x < [x] + 1.
Por outro lado: 0 x [x] < 1 1 < [x] x 0 x 1 < [x] x x 1 < [x] x < [x] + 1.
(3) [n + ] = n, n inteiro e 0 1
Demonstrao:
Se n inteiro 0 < 1 ento n n + < n + 1 [n + ] = n.
x

[x]

(4) = , x , m Z*+
m m
Demonstrao:
x [ x ] + [ x ]
Sabemos que x = [x] + , com 0 < 1. Assim: =
.
=
+
m m m m

x [ x ]
Como 0 < 1 e m Z*+ ento 0 < 1 . Pela propriedade 3: = .
m
m m

(5) [n + x] = n + [x], n Z, x
Demonstrao:
Pela propriedade 1: x = [x] + (0 x < 1) [n + x] = [n + [x] + ]
Como [x] Z, ento pela propriedade 3: [n + [x] + ] = n + [x]
Exemplos:

[ ] [ ]

1) Prove que k x = k [ x ] x 0, k Z *+ .
Soluo:
Seja m um nmero inteiro tal que: mk [x] x < (m + 1)k m k [ x ] k x < m + 1

[ x ] = [ [x] ]
k

2) (Olimpada da Blgica-99) [x] designa o maior inteiro menor ou igual a x. O conjunto de solues em
x + 2
:
2

da equao [x + 1] =

a) um intervalo aberto
b) um intervalo meio aberto
c) um intervalo fechado
d) unitrio
e) vazio
Soluo:
Notemos que: [x + 1] = [x/2 + 1] [x] + 1 = [x/2] + 1 [x] = [x/2] 1 x < 1 [ 1, 1[
Ou seja, temos um intervalo meio aberto.

250

Captulo 10. Funo Mximo Inteiro


3) (OBM-2000) A notao x significa o maior inteiro que no supera x. Por exemplo, 3,5 = 3
1
1
e 5 = 5. O nmero de inteiros positivos x para os quais x 2 + x 3 = 10 :

a) 11 b) 12 c) 13 d) 14 e) 15
Soluo:
Para x 48, temos x1/2 6 e x1/3 3. Para 49 x 63, temos x1/2 = 7 e x1/3 = 3. Para x 64,
temos x1/2 8 e x1/3 4. Assim, as solues so todos os inteiros entre 49 e 63, que so 15 ao todo

1
1
1
4) Calcule o valor de 1 +
+
+ ... +
.
2
3
1.000.000

Soluo:
k +1 k k +1 + k
1
k +1 k =
=
k +1 + k
k +1 + k
1
1
Como para k = 1, 2, 3, ... temos k + 1 > k
< k +1 k <
.
2 k +1
2 k
Aplicando, para a inequao da esquerda, k = 0, 1, 2, ..., n 1, e somando todas estas inequaes, temos:
1
1
1
1+
+
+ ... +
< 2 n 1
2
3
n
Aplicando, para a inequao da direita, k = 1, 2, ..., n, e somando todas estas inequaes, temos:
1
1
1
1+
+
+ ... +
> 2 n +1 2
2
3
n
1
1
1
Ou seja: 2 n + 1 2 < 1 +
+
+ ... +
< 2 n 1
2
3
n
Aplicando n = 1.000.000, e sabendo que 1.000.001 1.000.000 = 1.000 , temos:
1
1
1
1998 < 1 +
+
+ ... +
< 1999
2
3
n

1
1
1
+
+ ... +
Assim, podemos afirmar que 1 +
= 1998
2
3
1.000.000

)(

[ ] [ ]

5) (Furman University-96) Calcule: 3 1 + 3 2 + ... + 3 124 .


a) 401 b) 402 c) 403 d) 404 e) nda
Soluo:
se 1 x 7 [x1/3] = 1; se 8 x 26 [x1/3] = 2; se 27 x 64 [x1/3] = 3.
se 64 x 124 [x1/3] = 4.
Assim: S = 1.(7 1+ 1) + 2.(26 8 + 1) + 3.(64 27 + 1) + 4.(124 64 + 1)
S = 7 + 38 + 114 + 244 S = 403.
6) (Olimpada da Argentina-97) Achar todos os nmeros naturais n tais que [n2/5] um nmero primo.
Observao: Os colchetes indicam a parte inteira do nmero que encerram. Por exemplo, [100/5] = 20,
[121/5] = 4, etc.
Soluo:
i) n = 5k [n2/5] = [25k2/5] = [5k2] = 5k2 [n2/5] primo somente quando k = 1 [n2/5] = 5
ii) n = 5k 1 [n2/5] = [(25k2 10k + 1)/5] = [5k2 2k + 1/5] = 5k2 2k = k(5k 2)
[n2/5] primo quando k = 1 [n2/5] = 3 ou [n2/5] = 3 n = 6 ou n = 4
iii) n = 5k 2 [n2/5] = [(25k2 20k + 4)/5] = [5k2 4k + 4/5] = 5k2 4k = k(5k 4) nunca
primo
Assim temos somente as solues n = 4 ou n = 6.
251

Captulo 11. Equaes Diofantinas Lineares

EQUAES DIAFANTINAS LINEARES


11.1. DEFINIO:
Uma equao diofantina linear uma equao da forma a1x1 + a2x2 + ... + anxn = c, onde x1, x2,
..., xn so as incgnitas e a1, a2, ..., an so inteiros dados. O tipo mais simples de equao diofantina a
equao diofantina linear de duas incgnitas x e y:
ax + by = c onde a, b e c so inteiros dados, sendo ab 0.
Se um par de inteiros x0, y0 satisfaz ax0 + by0 = c ento denomina-se que x0, y0 uma soluo
inteira ou apenas soluo da equao ax + by = c.
Por exemplo, consideremos a equao diofantina linear com duas incgnitas: 3x + 6y = 18.
Observemos que: 3.4 + 6.1 = 18
3( 6) + 6.6 = 18
3.10 + 6( 2) = 18 Desta forma, os pares de
inteiros: 4 e 1, 6 e 6, 10 e 2 so solues da equao 3x + 6y = 18
Existem equaes diofantinas lineares com duas incgnitas que no tem soluo. Assim, por
exemplo, a equao diofantina 2x + 4y = 7 no tem soluo, porque 2x + 4y um inteiro par para
quaisquer que sejam os valores inteiros de x e y, enquanto que 7 um inteiro mpar.
11.2. CONDIO DE EXISTNCIA DE SOLUO
A equao diofantina linear ax + by = c tem soluo se e somente se d divide c, sendo d = mdc (a, b).
Demonstrao:
() Suponhamos que ax + by = c tem uma soluo, isto , que existem inteiros x0, y0 tais que ax0 +
by0 = c. Por ser o mdc (a, b) = d, existem inteiros r e s tais que a = dr e b = ds, e temos:
c = ax0 + by0 = drx0 + dsy0 = d(rx0 + sy0), e como rx0 + sy0 um inteiro, segue-se que d divide c.
() Suponhamos que d divide c, isto , que c = dt, onde t um inteiro.
Por ser o mdc (a, b) = d, existem inteiros x0 e y0 tais que d = ax0 + by0, o que implica:
c = dt = (ax0 + by0)t = a(tx0) + b(ty0), isto , o par de inteiros: x = tx0 = (c/d)x0, u = ty0 = (c/d)y0 uma
soluo da equao ax + by = c.
11.3. SOLUES DA EQUAO ax + by = c.
Teorema 7.2: Se d divide c (d | c), sendo d = mdc (a, b), e se o par de inteiros x0, y0 uma soluo
particular da equao diofantina linear ax + by = c, ento todas as outras solues desta equao so
b
a
dadas pelas frmulas: x = x 0 + t ,
y = y 0 t , onde t um inteiro arbitrrio.
d
d
Demonstrao:
Suponhamos que o par de inteiros x0, y0 uma soluo particular da equao ax + by = c, e seja x1, y1
uma soluo qualquer desta equao. Ento, temos:
ax0 + by0 = c = ax1 + by1 a(x1 x0) = b(y0 y1)
Como mdc (a, b) = d, ento existem inteiros r e s tais que a = dr e b = ds, com r e s primos entre si.
Substituindo estes valores de a e b na igualdade anterior e cancelando o fator comum d, obtemos:
r(x1 x0) = s(y0 y1)
Assim sendo, r | s(y0 y1), e como o mdc (r, s) = 1, segue-se que r | (y0 y1), isto :
y0 y1 = rt e x1 x0 = st
onde t um inteiro. Portanto temos as frmulas:
x1 = x0 + st = x0 + (b/d)t
y1 = y0 rt = y0 (a/d)t

258

Captulo 11. Equaes Diofantinas Lineares


Exemplos:
1) Resolver a equao diofantina linear 14x + 22y = 50
Soluo:
Como o mdc (14, 22) = 2 e 2 | 50, ento a equao dada tem soluo, e por simples inspeo logo se v
que 14.2 + 22.1 = 50, de modo que o par de inteiros x0 = 2, y0 = 1 uma soluo particular, e por
conseguinte todas as demais solues so dadas pelas frmulas: x = 2 + 11t
y = 1 7t
onde t
inteiro arbitrrio.
2) (OBM-98) Quantos so os pares (x, y) de inteiros positivos que satisfazem a equao 2x + 3y = 101?
A) 13 B) 14 C) 15 D) 16 E) 17
Soluo:
Analisando a equao, notamos que a soluo com menor valor positivo para x x0 = 1 e y0 = 33.
x = x0 + (b/d)t y = y0 (a/d)t x = 1 + 3t y = 33 2t, t inteiro.
Evidentemente devemos aplicar t 0, pois se t < 0 teremos x < 0.
Assim, o problema saber at quando 33 2t > 0, pois se t > 0 1 + 37 > 0.
33 2t > 0 2t < 33 t < 16,5.
Como t inteiro 0 t 16 existem 17 pares (x, y) de inteiros positivos que satisfazem a
equao 2x + 3y = 101.
3) (British Columbia Colleges-98) Determine um conjunto de 3 inteiros positivos consecutivos tais que o
menor deles mltiplo de 5, o segundo mltiplo de 7 e o maior mltiplo de 9.
Soluo:
Sejam x, x + 1, x + 2 os inteiros, sendo que x = 5a x + 1 = 7b x + 2 = 9c.
Assim:
(x + 1) x = 1 7b 5a = 1, onde uma soluo inicial b0 = 3 a0 = 4 b = 3 + 5r a = 4 + 7r
Desta forma x = 5a = 5(4 + 7r) x = 20 + 35r
(x + 2) x = 2 9c 20 35r = 2 9c 35r = 22, onde c0 = 18 r0 = 4
c = 18 + 35t r = 4 + 9t
Portanto: x = 5a = 5(4 + 7r) = 5(4 + 7(4 + 9t)) = 20 + 35(4 + 9t) x = 160 + 315t.
Deste modo os inteiros 160 + 315t, 161 + 315t, 162 + 315t formam a soluo geral do problema.
4) (Olimpada da Noruega-99) Assuma que m e n so inteiros tais que 5m + 6n = 100. Ento, o maior
valor possvel de m.n :
a) 60 b) 70 c) 80 d) 90 e) nda
Soluo:
Como mdc (5, 6) = 1, e m0 = 20 e n0 = 0 uma soluo, temos que todas as solues so dadas por:
m = 20 + 6t e n = 5t
Assim, m.n = (20 + 6t)( 5t) m.n = 30t2 100t 30t2 + 100t + m.n = 0
m.nmax = (10000)/(4( 30)) m.nmax = 83,33333
que no inteiro, mais j d uma dica do maior valor inteiro de m.n, pois m.n 83.
Para que t seja inteiro, devemos ter o discriminante igual a um quadrado perfeito: 1002 120mn = x2
Para m.n = 83 temos que 1002 120mn no um quadrado perfeito. O mesmo ocorre para m.n igual a
82 e 81. Para m.n = 80 temos 1002 120(80) = 202. Assim: mnmax = 80.
Conferindo: 30t2 + 100t + 80 = 0 3t2 + 10t + 8 = 0 (3t + 4)(t + 2) = 0 t = 2.

259

Captulo 11. Equaes Diofantinas Lineares


evidentemente, (x0,y0) = (100,1). Alm desse, h
apenas mais um par de nmeros inteiros e
positivos, (x1, y1), satisfazendo a equao. O valor
de x1 + y1 :
A) 23 B) 52 C) 54 D) 101 E) 1997

Exerccios
1) Exprimir 100 como soma de dois inteiros
positivos de modo que o primeiro seja divisvel
por 7 e o segundo seja divisvel por 11.

10) (OBM-98) No planeta Z todos os habitantes


possuem 3 pernas e cada carro possui 5 rodas. Em
uma pequena cidade desse planeta, existem ao
todo 97 pernas e rodas. Ento podemos afirmar:
A) possvel que existam 19 carros nessa cidade
B) Existem no mximo 16 carros nessa cidade
C) Essa cidade tem 9 habitantes e 14 carros
D) Essa cidade possui no mximo 17 carros
E) Nessa cidade existem mais carros do que
pessoas

2) Determinar as duas menores fraes positivas


que tenham 13 e 17 para denominadores e cuja
soma seja igual a 305/221.
3) Demonstrar que, se a e b so inteiros positivos
primos entre si, ento a equao diofantina ax
by = c tem um nmero infinito de solues
inteiras e positivas.
4) Se x e y so inteiros positivos, determine o
nmero de solues de 2x + 3y = 100.

11) (OBM-99) Quantos so os pares (x, y) de


inteiros positivos que satisfazem a equao 2x +
3y = 101 ?
A) 13 B) 14 C) 15 D) 16 E) 17

5) (Epcar-2003) Um aluno da EPCAR, indagado


sobre o nmero de exerccios de matemtica que
havia resolvido naquele dia respondeu: "No sei,
mas contando de 2 em 2 sobra um; contando de 3
em 3 sobra um; contando de 5 em 5 tambm sobra
um; mas contando de 7 em 7 no sobra nenhum.
O total de exerccios no chega a uma centena".
Ento, o nmero de exerccios resolvidos tal que
a soma de seus algarismos igual a
a) 8 b) 10 c) 9 d) 11

12) (USA Talent Search-99) Seja C o conjunto


dos inteiros no-negativos que podem ser
expressos como 1999s + 2000t, onde s e t so
tambm inteiros no-negativos.
a) Mostre que 3.994.001 no pertence a C.
b) Mostre que se 0 n 3.994.001 e n um
inteiro no pertencente a C, ento 3.994.001 n
pertence a C.

Questes de Olimpadas
6) (Rio Grande do Norte-95) Uma caixa
automtica de banco s trabalha com notas de 5 e
10 cruzeiros. Um usurio deseja fazer um saque
de 100 cruzeiros. De quantas maneiras distintas a
caixa eletrnica poder fazer o pagamento?

13) (Canad-97) Determine todos os pontos no


segmento de reta que liga ( 4, 11) a (16, 1) e
cujas coordenadas so nmeros inteiros positivos.
14) (Argentina-97) Quantos nmeros entre 1 e
1000 inclusive podem decompor-se em soma de
um mltiplo positivo de 7 mais um mltiplo
positivo de 4?

7) (So Paulo-98) Encontre o menor inteiro


positivo a para o qual a equao 1001x + 770y =
106 + a tem soluo inteira. Neste caso, quantas
solues inteiras positivas (x > 0 e y > 0)
existem?

15) (frica do Sul-94) Qual o maior inteiro


positivo que no pode ser expresso na forma 5x +
7y, com x e y inteiros positivos?

8) (OBM-79) Mostre que o nmero de solues


inteiras positivas da equao x1 + 8x2 + 27x3 +
+ 100x10 = 3025 (*) igual ao nmero de
solues inteiras no negativas de y1 + 8y2 + 27y3
+ + 100y10 = 0. Usando este fato, conclua que a
equao (*) tem uma nica soluo inteira
positiva. Determine esta soluo.

16) (Blgica-90) Determine o nmero de solues


(x, y) da equao y + 3x = 100 com inteiros x e y,
ambos maiores ou iguais a 0.
17) (Blgica-96) Quantos pares de inteiros (n, k)
possuem a propriedade que 1 = 3n + 5k?
a) 0 b) 7 c) 8 d) 15 e) infinitos

9) (OBM-97) Uma das solues inteiras e


positivas da equao 19x + 97y = 1997 ,
260

Das könnte Ihnen auch gefallen